You are on page 1of 108

60

IAS BABA
TEST SERIES
TEST-NO-16
�हंद� माध्यम
2020
AIPTS/ILP VETERANS-
Exam Title :
2020 TE...
Email : yadavanurag075@gmail.com
Contact : 8882839768

Review in Hindi
QUESTION 1.
73 व��संवैधा�नक�संशोधन�अ�ध�नयम�, 1992 के�तहत���तुत��कए�गए�पंचायती�राज�म���मुखता�से�संचा�लत�होता�है:

a) म�यवत��और��जला��तर।�
b) स�म�त�और��जला��तर।�
c) स�म�त�और�म�यवत���तर।�
d) �ाम�, म�यवत��और��जला��तर�
QUESTION 2.
�न�न�ल�खत�म��से�कौन�सा�कथन�सही�है�/ ह��?

1. पंचायत��के�चुनाव�लड़ने�के��लए��कसी�भी�����के��लए��नधा��रत��यूनतम�आयु�25 वष��है।�
2. समय�से�पहले��वघटन�के�बाद�एक�पंचायत�का�पुनग�ठन�केवल�शेष�अव�ध�के��लए�जारी�रहता�है।�

नीचे��दए�गए�कूट�का�उपयोग�करके�सही�उ�र�चुन�:

a) केवल�1
b) केवल�2
c) दोन��1 और�2
d) न�तो�1 और�न�ही�2
QUESTION 3.
�न�न�ल�खत�म��से�कौन�से�रा�य�पंचायती�राज��ारा�कवर�नह���कये�गये�ह��?

a) ��पुरा�और��स��कम।�
b) म�णपुर�और�असम।�
c) मेघालय�और�नागाल�ड।�
d) उपरो��सभी�
QUESTION 4.
73 वां�संवैधा�नक�संशोधन�अ�ध�नयम�, 1992, �जसका�उ�े �य�दे श�म��पंचायती�राज�सं�थान��को�बढ़ावा�दे ना�है�, �न�न�ल�खत�म��से��कसके
�लए��ावधान�करता�है�?

1. �जला�योजना�स�म�तय��का�गठन।�
2. पंचायत��म��चुनाव�कराने�के��लए�रा�य�चुनाव�आयोग।�
3. रा�य��व��आयोग�का�गठन।�

उपरो��कथन��म��से�कौन�सा�सही�है�/ ह��?

a) केवल�1
b) केवल�1 और�2
c) केवल�2 और�3
d) 1, 2 और�3
QUESTION 5.
�न�न�ल�खत�म��से�कौन�पंचायती�राज�के��े��म��73 व��संवैधा�नक�संशोधन��ारा���ता�वत�नह��था�?

a) सभी��नवा��चत��ामीण��थानीय��नकाय��म��, जो�कुल�सीट��क��एक��तहाई�से�कम�नह��होगी�, सभी��तर��पर�म�हला�उ�मीदवार��के��लए


आर��त�होगी।�

IASbaba
Web: http://ilp.iasbaba.com/
Email: ilp@iasbaba.com
Page 35
AIPTS/ILP VETERANS-
Exam Title :
2020 TE...
Email : yadavanurag075@gmail.com
Contact : 8882839768

b) रा�य�पंचायती�राज�सं�था��को�संसाधन�आवं�टत�करने�के��लए�अपने��व��आयोग��का�गठन�कर�गे।�
c) य�द�दो�से�अ�धक�ब�चे�ह��तो�पंचायती�राज�पदा�धका�रय��को�अपने�काया�लय�म��बने�रहने�के��लए�अयो�य�घो�षत�कर��दया�जाएगा।�
d) पंचायती�राज��नकाय��को�रा�य�सरकार��ारा�अ�धगृहीत�या�भंग�कर��दए�जाने�पर�छह�महीने�के�अंदर�चुनाव�ह�गे।�
QUESTION 6.
�न�न�ल�खत�म��से�कौन�सी�स�म�त�पंचायती�राज�क���थापना�से�संबं�धत�है�?

1. थुंगन�स�म�त�
2. अशोक�मेहता�स�म�त�
3. गाड�गल�स�म�त�
4. बलवंत�राय�मेहता�स�म�त�

नीचे��दए�गए�कूट�का�उपयोग�करके�सही�उ�र�चुन�:

a) केवल�1 और�2
b) केवल�2 और�3
c) केवल�1, 2, और�3
d) 1, 2, 3 और�4
QUESTION 7.
�न�न�ल�खत�म��से�कौन�सा�कथन�गलत�है�?

a) 73 व��संवैधा�नक�संशोधन�अ�ध�नयम�, 1992 को�पी. वी. नर�स�हा�राव�के��धानमं���व�काल�म��अ�ध�नय�मत��कया�गया�था।�


b) पंचायत��का�गठन�रा�य�सरकार�क��इ�छा�पर��नभ�र�करता�है।�
c) पंचायती�राज�सं�थान�सं�वधान�के��यायो�चत��ह�से�के�दायरे�म��ह�।�
d) इनम��से�कोई�भी�नह�।�
QUESTION 8.
�न�न�ल�खत�कथन��पर��वचार�कर�:

1. �ाम�, म�यवत��और��जला��तर�पर�पंचायत��के�सभी�सद�य�सीधे�लोग���ारा�चुने�जाएंगे।�
2. �ाम�, म�यवत��और��ज़ला��तर�पर�एक�पंचायत�के�अ�य��का�चुनाव�इस�तरह�से��कया�जाएगा�जैसा�रा�य��वधानमंडल��नधा��रत
करता�है।�

ऊपर��दए�गए�कथन��म��से�कौन�सा�सही�है�/ ह��?

a) केवल�1
b) केवल�2
c) दोन��1 और�2
d) न�तो�1 और�न�ही�2
QUESTION 9.
73 व��संवैधा�नक�संशोधन�अ�ध�नयम�, 1992 के�अनुसार�सीट��के�आर�ण�के�संदभ��म���न�न�ल�खत�कथन��पर��वचार�कर�:

1. ��येक�पंचायत�म��अनुसू�चत�जा�त�और�अनुसू�चत�जनजा�त�के��लए�सीट��का�आर�ण�पंचायत��े��म��उनक��जनसँ�या�के�अनुपात�म�
है।�
2. अ�ध�नयम�म��म�हला��के��लए�सीट��के�आर�ण�का��ावधान�है�, जो�कुल�सं�या�के�आधे�से�कम�नह��होना�चा�हए।�

ऊपर��दए�गए�कथन��म��से�कौन�सा�सही�है�/ ह��?

a) केवल�1

IASbaba
Web: http://ilp.iasbaba.com/
Email: ilp@iasbaba.com
Page 36
AIPTS/ILP VETERANS-
Exam Title :
2020 TE...
Email : yadavanurag075@gmail.com
Contact : 8882839768

b) केवल�2
c) दोन��1 और�2
d) न�तो�1 और�न�ही�2
QUESTION 10.
रा�य�चुनाव�आयोग�के�संदभ��म���न�न�ल�खत�कथन��पर��वचार�कर�:

1. रा�य��नवा�चन�आयु��को�भारत�के�मु�य�चुनाव�आयु��के�परामश��से�रा�यपाल��ारा��नयु���कया�जाता�है।�
2. रा�य��नवा�चन�आयु��के�काया�लय�क��सेवा�और�काय�काल�क��शत��रा�य��वधा�यका��ारा��नधा��रत�क��जाती�ह�।�

उपरो��कथन��म��से�कौन�सा�सही�है�/ ह��?

a) केवल�1
b) केवल�2
c) दोन��1 और�2
d) न�तो�1 और�न�ही�2
QUESTION 11.
पंचायती�राज�के��न�न�ल�खत��ावधान��पर��वचार�कर�:

1. �कसी�भी��तर�पर�पंचायत��म���पछड़े�वग��के��लए�सीट��(दोन��सद�य��और�अ�य��) का�आर�ण��दान�करना।�
2. आ�थ�क��वकास�और�सामा�जक��याय�क��योजना�तैयार�करने�के��लए�पंचायत��को�श��य��और�उ�रदा�य�व��का�ह�तांतरण�
3. पंचायत��को��व�ीय�श��यां��दान�करना�, अथा�त्�उ�ह��उ�चत�कर�, शु�क�और�टोल�, फ�स�को�आरो�पत�और�एक��त�करने�के��लए
अ�धकृत�करना।�

उपरो���ावधान��म��से�कौन�सा�/ से��वै��छक��ावधान�( voluntary provisions) ह��/ ह��?

a) केवल�1
b) केवल�1 और�2
c) केवल�2 और�3
d) 1, 2 और�3
QUESTION 12.
पंचायत�(अनुसू�चत��े���के��लए��व�तार) अ�ध�नयम�( PESA), 1996 क���वशेषता��के�संदभ��म���न�न�ल�खत�कथन��पर��वचार�कर�।�

1. अनुसू�चत��े���म��पंचायत��पर�एक�रा�य�कानून��थागत�कानून�, सामा�जक�और�धा�म�क��था��के�अनु�प�होगा।�
2. अनुसू�चत��े���म��लघु�जल��नकाय��क��योजना�और��बंधन�को�उ�चत��तर�पर�पंचायत��को�स�पा�जाना�चा�हए।�
3. यह�सु�न��त�करने�के��लए��क�उ�च��तर�क��पंचायत��, �नचले��तर�या��ाम�सभा�क��पंचायत�क��श��य��और�अ�धकार��को�अ�ध��हत
कर�।�

�न�न�ल�खत�म��से�कौन�सी�अ�ध�नयम�क���वशेषताएं�ह��/ ह��?

a) केवल�1
b) केवल�1 और�2
c) केवल�1 और�3
d) 1, 2 और�3
QUESTION 13.
पंचायत�(अनुसू�चत��े���के��लए��व�तार) अ�ध�नयम�, 1996 के�तहत�आने�वाले��े���म��, �ाम�सभा�क��भू�मका�/ श����या�है�?

1. �ाम�सभा�के�पास�अनुसू�चत��े���म��भू�म�के�अलगाव�को�रोकने�क��श���है।�

IASbaba
Web: http://ilp.iasbaba.com/
Email: ilp@iasbaba.com
Page 37
AIPTS/ILP VETERANS-
Exam Title :
2020 TE...
Email : yadavanurag075@gmail.com
Contact : 8882839768

2. गरीबी�उ�मूलन�और�अ�य�काय��म��के�तहत�लाभा�थ�य��क��पहचान�या�चयन।�
3. अनुसू�चत��े���म���कसी�भी�ख�नज�के��लए�पूव��ण�लाइस�स�या�खनन�प�े �दे ने�के��लए��ाम�सभा�क���सफा�रश�आव�यक�है।�

ऊपर��दए�गए�कथन��म��से�कौन�सा�सही�है�/ ह��?

a) केवल�1 और�2
b) केवल�1 और�3
c) केवल�2 और�3
d) 1, 2 और�3
QUESTION 14.
�न�न�ल�खत�म��से�कौन�73 व��संवैधा�नक�संशोधन�अ�ध�नयम�( 1992) का�अ�नवाय���ावधान�नह��है:

a) �कसी�गाँव�या�गाँव��के�समूह�म���ाम�सभा�का�संगठन।�
b) तीन���तर��पर�पंचायत��म��अनुसू�चत�जा�तय��और�अनुसू�चत�जनजा�तय��के��लए�सीट��(दोन��सद�य��और�अ�य��) का�आर�ण।�
c) गाँव�, म�यवत��और��जला��तर��पर�पंचायत��क��सभी�सीट��पर���य��चुनाव।�
d) इनम��से�कोई�भी�नह�।�
QUESTION 15.
पंचायत��(अनुसू�चत��े���के��लए��व�तार) अ�ध�नयम�, 1996 के�तहत��दान��कए�गए�अनुसू�चत�जनजा�तय��के��लए�आर�ण�के��ावधान�
के�संदभ��म���न�न�ल�खत�कथन��पर��वचार�कर�:

1. अनुसू�चत�जनजा�तय��के��लए�आर�ण�कुल�सीट��क��सं�या�के�एक��तहाई�से�कम�नह��होगा।�
2. सभी��तर��पर�पंचायत��के�अ�य���क��कुल�सीट��म��से�आधी�अनुसू�चत�जनजा�त�के��लए�आर��त�ह�।�

ऊपर��दए�गए�कथन��म��से�कौन�सा�गलत�है�/ ह��?

a) केवल�1
b) केवल�2
c) दोन��1 और�2
d) न�तो�1 और�न�ही�2
QUESTION 16.
�न�न�ल�खत�कथन��पर��वचार�कर�:

1. पया��त�ह�तांतरण�( adequate devolution) का�अभाव।�


2. राजकोषीय�श��य��का�उपयोग�करने�क��अ�न�छा।�
3. नौकरशाही��ारा�अ�य�धक��नयं�ण।�

ऊपर��दए�गए�कथन��म��से�कौन�सा�/ से�पंचायती�राज�सं�थान��के�अ�भावी��दश�न�के�सही�कारण�ह��?

a) केवल�1 और�2
b) केवल�2
c) केवल�2 और�3
d) 1, 2 और�3
QUESTION 17.
74 व��सं�वधान�(संशोधन) अ�ध�नयम�, 1992 के�संदभ��म���न�न�ल�खत�कथन��पर��वचार�कर�:

1. इस�अ�ध�नयम��ारा�भारत�के�सं�वधान�म��भाग�IX A स��म�लत��कया�गया��जसम��नगर�पा�लका��के��लए��ावधान�ह�।�
2. अ�ध�नयम�म����येक�रा�य�हेत�ु दो��कार�क��नगरपा�लका��के�गठन�के��लए�, एक�नगर�प�रषद�और�एक�नगर��नगम�का��ावधान�है।�

IASbaba
Web: http://ilp.iasbaba.com/
Email: ilp@iasbaba.com
Page 38
AIPTS/ILP VETERANS-
Exam Title :
2020 TE...
Email : yadavanurag075@gmail.com
Contact : 8882839768

ऊपर��दए�गए�कथन��म��से�कौन�सा�सही�है�/ ह��?

a) केवल�1
b) केवल�2
c) दोन��1 और�2
d) न�तो�1 और�न�ही�2
QUESTION 18.
क���य��तर�पर�, ‘ शहरी��थानीय�सरकार�’ (urban local government) का��वषय��न�न�ल�खत�मं�ालय�/ मं�ालय��म��से��कससे
संब��है:

1. शहरी��वकास�मं�ालय�
2. र�ा�मं�ालय�
3. गृह�मं�ालय�

ऊपर��दए�गए�कथन��म��से�कौन�सा�सही�है�/ ह��?

a) केवल�1 और�2
b) केवल�1 और�3
c) केवल�2 और�3
d) 1, 2 और�3
QUESTION 19.
�न�न�ल�खत�कथन��पर��वचार�कर�:

1. भारत�म��पहला�नगर��नगम�1687-88 म��बॉ�बे�म���था�पत��कया�गया�था।�
2. लॉड���रपन�को�भारत�म���थानीय-�वशासन�का��पता�कहा�जाता�है।�

सही�कूट�का�चयन�कर�:

a) केवल�1
b) केवल�2
c) दोन��1 और�2
d) न�तो�1 और�न�ही�2
QUESTION 20.
74 व��सं�वधान�(संशोधन) अ�ध�नयम�, 1992 के��ावधान��के�संदभ��म���न�न�ल�खत�कथन��पर��वचार�कर�:

1. रा�य��वधा�यका�एक�नगरपा�लका�के�अ�य��के�चुनाव�का�तरीका��दान�कर�सकती�है।�
2. अ�ध�नयम�म��एससी�, एसट��और�म�हला��के��लए�नगरपा�लका��म��अ�य���के�काया�लय��के�आर�ण�का�तरीका�है।�

ऊपर��दए�गए�कथन��म��से�कौन�सा�सही�है�/ ह��?

a) केवल�1
b) केवल�2
c) दोन��1 और�2
d) न�तो�1 और�न�ही�2
QUESTION 21.

IASbaba
Web: http://ilp.iasbaba.com/
Email: ilp@iasbaba.com
Page 39
AIPTS/ILP VETERANS-
Exam Title :
2020 TE...
Email : yadavanurag075@gmail.com
Contact : 8882839768

�जला�योजना�स�म�त�( District Planning Committee) के�संदभ��म���न�न�ल�खत�कथन��पर��वचार�कर�:

1. इसका�गठन��जले�म��पंचायत��और�नगरपा�लका���ारा�तैयार�क��गई�योजना��को�समे�कत�करने�के��लए��कया�गया�है।�
2. एक�स�म�त�के�सद�य��म��से�4/5 सद�य��को��जले�के�नगरपा�लका��के��नवा��चत�सद�य���ारा�अपने�बीच�से�चुना�जाना�चा�हए।�

ऊपर��दए�गए�कथन��म��से�कौन�सा�सही�है�/ ह��?

a) केवल�1
b) केवल�2
c) दोन��1 और�2
d) न�तो�1 और�न�ही�2
QUESTION 22.
�न�न�ल�खत�कथन��पर��वचार�कर�:

1. मे�ोपॉ�लटन��े��का�अथ��एक��जले�म��10 लाख�या�उससे�अ�धक�क��आबाद��वाला��े��है।�
2. नगर�पा�लका��और�पंचायत��के��नवा��चत�सद�य�महानगर��े��म��एक�महानगरीय�योजना�स�म�त�के�2/3 सद�य��का�चुनाव�करते�ह�।
3. रा�य��वधा�यका�, मे�ोपॉ�लटन�योजना�स�म�त�( Metropolitan Planning Committees -MPC) के�सद�य��के�चुनाव�के
संघटन�और�तरीके�का��ावधान�कर�सकती�है।�

ऊपर��दए�गए�कथन��म��से�कौन�सा�सही�है�/ ह��?

a) केवल�1
b) केवल�1 और�2
c) केवल�3
d) 1, 2 और�3
QUESTION 23.
भारतीय�सं�वधान�क��12 व��अनुसूची�के�तहत��न�न�ल�खत�म��से�कौन�सा��वषय�शा�मल�नह��है:

a) �लम�सुधार�और�उ�यन�
b) मवेशी�तालाब�, पशु��के��लए��ूरता�क��रोकथाम�
c) अ��नशमन�सेवाएं�
d) वय�क�और�गैर-औपचा�रक��श�ा�
QUESTION 24.
�न�न�ल�खत�कथन��पर��वचार�कर�:

1. संसद�के�अ�ध�नयम���ारा�रा�य��म��नगर��नगम��था�पत��कए�जाते�ह�।�
2. एक�मेयर�एक�नगर��नगम�क��प�रषद�का��मुख�होता�है।�
3. नगरपा�लका�आयु��प�रषद�और�उसक���थायी�स�म�तय���ारा��लए�गए��नण�य��के�काया��वयन�के��लए�उ�रदायी�है।�

ऊपर��दए�गए�कथन��म��से�कौन�सा�सही�है�/ ह��?

a) केवल�1
b) केवल�1 और�2
c) केवल�2 और�3
d) 1, 2 और�3
QUESTION 25.

IASbaba
Web: http://ilp.iasbaba.com/
Email: ilp@iasbaba.com
Page 40
AIPTS/ILP VETERANS-
Exam Title :
2020 TE...
Email : yadavanurag075@gmail.com
Contact : 8882839768

नगर�पा�लका�के�संदभ��म���न�न�ल�खत�कथन��पर��वचार�कर�:

1. एक�नगरपा�लका�म��भी�केवल�दो��ा�धकरण�होते�ह��, अथा�त्�प�रषद�और�मु�य�काय�कारी�अ�धकारी।�
2. अ�य��/ �मुख�न�केवल�प�रषद�क��बैठक��क��अ�य�ता�करते�ह��, ब��क�काय�कारी�श��य��का�भी�आनंद�लेते�ह�।�

ऊपर��दए�गए�कथन��म��से�कौन�सा�सही�है�/ ह��?

a) केवल�1
b) केवल�2
c) दोन��1 और�2
d) न�तो�1 और�न�ही�2
QUESTION 26.
�न�न�ल�खत�पर��वचार�कर�:

1. अ�धसू�चत��े��स�म�त�( notified area committee) एक�सां�व�धक��नकाय�है।�


2. अ�धसू�चत��े��स�म�त�को�रा�य�नगरपा�लका�अ�ध�नयम�के�सभी��ावधान��के�अनुसार�काय��करना�होता�है।�

ऊपर��दए�गए�कथन��म��से�कौन�सा�सही�है�/ ह��?

a) केवल�1
b) केवल�2
c) दोन��1 और�2
d) न�तो�1 और�न�ही�2
QUESTION 27.
छावनी�बोड��के�संदभ��म���न�न�ल�खत�कथन��पर��वचार�कर�:

1. यह�संबं�धत�रा�य�सरकार���ारा�अ�ध�नय�मत�कानून�के��ावधान��के�तहत��था�पत��कया�गया�है।�
2. छावनी�बोड��के�काय�कारी�अ�धकारी�क���नयु���भारत�के�रा�प�त��ारा�क��जाती�है।�
3. �टे शन�क��कमान�संभालने�वाला�सै�य�अ�धकारी�बोड��का�पदे न�अ�य��होता�है�तथा�इसक��बैठक��क��अ�य�ता�करता�है।�

ऊपर��दए�गए�कथन��म��से�कौन�सा�सही�है�/ ह��?

a) केवल�1 और�2
b) केवल�1 और�3
c) केवल�2 और�3
d) 1, 2 और�3
QUESTION 28.
�वशेष��योजन�एज�सी�( Special Purpose Agency) के�बारे�म���न�न�ल�खत�कथन��पर��वचार�कर�:

1. यह�एक�काय�-आधा�रत�इकाई�है�जो��न�द���ग�त�व�धय��या��व�श��काय��को�करने�के��लए��था�पत�क��जाती�है।�
2. रा�य��वधानमंडल�के�एक�अ�ध�नयम��ारा�उ�ह��वैधा�नक��नकाय��के��प�म���था�पत��कया�जा�सकता�है।�
3. वे��थानीय�नगर��नकाय��के�अधीन�थ�एज��सय��के��प�म��काय��करते�ह�।�

ऊपर��दए�गए�कथन��म��से�कौन�सा�सही�है�/ ह��?

a) केवल�1
b) केवल�1 और�2
c) केवल�2 और�3

IASbaba
Web: http://ilp.iasbaba.com/
Email: ilp@iasbaba.com
Page 41
AIPTS/ILP VETERANS-
Exam Title :
2020 TE...
Email : yadavanurag075@gmail.com
Contact : 8882839768

d) 1, 2 और�3
QUESTION 29.
�न�न�ल�खत��थानीय�सरकारी�सं�था��म��से�कौन�रा�य��वधानसभा��के�अ�ध�नयम��ारा��था�पत�नह��है/ ह��?

1. छावनी�बोड��( Cantonment Board)


2. टाउन�शप�( Township)
3. टाउन�ए�रया�कमेट��( Town Area Committee)
4. अ�धसू�चत��े��स�म�त�( Notified Area Committee)

�न�न�ल�खत�म��से�सही�कूट�का�चयन�कर�:

a) केवल�1 और�2
b) केवल�1 और�3
c) केवल�1, 2 और�4
d) 1, 2, 3 और�4
QUESTION 30.
क���शा�सत��दे श��के��शासन�के�संदभ��म���न�न�ल�खत�कथन��पर��वचार�कर�:

1. क���शा�सत��दे श�का�एक��शासक�रा�प�त��ारा��नयु���कया�जाता�है�तथा�एक�रा�यपाल�क��तरह�रा�य�का��मुख�होता�है।�
2. रा�प�त��कसी�रा�य�के�रा�यपाल�को�समीपवत��क���शा�सत��दे श�के��शासक�के��प�म��भी��नयु��कर�सकता�है।�

ऊपर��दए�गए�कथन��म��से�कौन�सा�सही�है�/ ह��?

a) केवल�1
b) केवल�2
c) दोन��1 और�2
d) न�तो�1 और�न�ही�2
QUESTION 31.
�न�न�ल�खत�कथन��पर��वचार�कर�:

1. संसद�सभी�क���शा�सत��दे श��के��लए�तीन�सू�चय��(रा�य�सूची�स�हत) के��कसी�भी��वषय�पर�कानून�बना�सकती�है।�


2. भारत�का�सव��च��यायालय�क���शा�सत��दे श�के��लए�एक�उ�च��यायालय��था�पत�कर�सकता�है�या�उसे�समीपवत��रा�य�के�उ�च
�यायालय�के�अ�धकार��े��म��डाल�सकता�है।�

ऊपर��दए�गए�कथन��म��से�कौन�सा�सही�है�/ ह��?

a) केवल�1
b) केवल�2
c) दोन��1 और�2
d) न�तो�1 और�न�ही�2
QUESTION 32.
�द�ली�के��लए��वशेष��ावधान��के�बारे�म���न�न�ल�खत�म��से�कौन�सा�कथन�सही�है�?

a) �वधान�सभा�क��साम�य��60 सद�य��तक��न��त�है�
b) �द�ली�का�रा�य��नवा�चन�आयोग��द�ली�क���वधान�सभा�के��लए�चुनाव�आयो�जत�करता�है।�
c) �द�ली�के�मु�यमं�ी�को��द�ली�के�उपरा�यपाल��ारा��नयु���कया�जाता�है।�
d) �वधानसभा�क��कुल�साम�य��के�दस���तशत�तक�मं��य��क��प�रषद�क��साम�य���न��त�होती�है।�

IASbaba
Web: http://ilp.iasbaba.com/
Email: ilp@iasbaba.com
Page 42
AIPTS/ILP VETERANS-
Exam Title :
2020 TE...
Email : yadavanurag075@gmail.com
Contact : 8882839768

QUESTION 33.
�न�न�ल�खत�कथन��पर��वचार�कर�:

1. गृह�मं�ालय�, क���शा�सत��दे श��के�आव�यक�मामल��के��लए�नोडल�मं�ालय�है।�


2. गृह�मं�ी�क��सलाहकार�स�म�त�( HMAC) के�मंच�क��अ�य�ता�क���य�गृह�मं�ी��ारा�क��जाती�है।�

ऊपर��दए�गए�कथन��म��से�कौन�सा�सही�है�/ ह��?

a) केवल�1
b) केवल�2
c) दोन��1 और�2
d) न�तो�1 और�न�ही�2
QUESTION 34.
सं�वधान�क��छठ��अनुसूची�के�संदभ��म���न�न�ल�खत�कथन��पर��वचार�कर�:

1. यह�चार�उ�र-पूव��रा�य��असम�, मेघालय�, ��पुरा�और��मजोरम�म��जनजातीय��े���के��शासन�और��नयं�ण�से�संबं�धत�है।�


2. इसके�तहत�ऐसे�जनजातीय��े���वाले���येक�रा�य�को�एक�जनजातीय�सलाहकार�प�रषद�क���थापना�करनी�है।�

ऊपर��दए�गए�कथन��म��से�कौन�सा�सही�है�/ ह��?

a) केवल�1
b) केवल�2
c) दोन��1 और�2
d) न�तो�1 और�न�ही�2
QUESTION 35.
पांचव��अनुसूची�म���न�हत��शासन�क���व�भ���वशेषता��के�बारे�म���न�न�ल�खत�कथन��पर��वचार�कर�:

1. रा�प�त�को�एक��े��को�अनुसू�चत��े��घो�षत�करने�का�अ�धकार�है।�
2. जनजा�त�सलाहकार�प�रषद�क��सलाह�के�बाद�रा�प�त�एक�अनुसू�चत��े��क��शां�त�और�अ�छ��सरकार�के��लए��नयम�बना�सकते�ह�।�

ऊपर��दए�गए�कथन��म��से�कौन�सा�सही�है�/ ह��?

a) केवल�1
b) केवल�2
c) दोन��1 और�2
d) न�तो�1 और�न�ही�2
QUESTION 36.
�न�न�ल�खत�जोड़े�पर��वचार�कर�:

रा�य�जनजातीय��े��

1. असम�बोडोल�ड��ादे �शक��े���जला�
2. �मजोरम�चकमा��जला�
3. मेघालय�गारो��ह�स��जला�

ऊपर�द��गई�कौन�सी�जोड़ी�सही�है�/ ह��?

a) केवल�1 और�2

IASbaba
Web: http://ilp.iasbaba.com/
Email: ilp@iasbaba.com
Page 43
AIPTS/ILP VETERANS-
Exam Title :
2020 TE...
Email : yadavanurag075@gmail.com
Contact : 8882839768

b) केवल�1 और�3
c) केवल�2 और�3
d) 1, 2, और�3
QUESTION 37.
भारत�के�चुनाव�आयोग��ारा�मा�यता��ा�त�राजनी�तक�दल��को��दए�गए��वशेषा�धकार��के�संदभ��म���न�न�ल�खत�कथन��पर��वचार�कर�:

1. पाट��के��तीक��का�आवंटन।�
2. �नजी�टे ली�वजन�और�रे�डयो��टे शन��पर�राजनी�तक��सारण�के��लए�समय।�
3. मतदाता�सूची�तक�प�ंच।�

ऊपर��दए�गए�कथन��म��से�कौन�सा�सही�है�/ ह��?

a) केवल�1 और�2
b) केवल�1 और�3
c) केवल�2 और�3
d) 1, 2 और�3
QUESTION 38.
�न�न�ल�खत�कथन��पर��वचार�कर�:

1. चार�रा�य��म��रा�य�पाट��के��प�म��मा�यता��ा�त�एक�राजनी�तक�दल�को�रा�ीय�पाट��के��प�म��मा�यता��ा�त�होती�है।�
2. य�द�कोई�राजनी�तक�पाट��लोकसभा�के�आम�चुनाव�म��6% सीट� �जीतती�है�, तो�उसे�रा�ीय�पाट��के��प�म��मा�यता�द��जाती�है।�

ऊपर��दए�गए�कथन��म��से�कौन�सा�सही�है�/ ह��?

a) केवल�1
b) केवल�2
c) दोन��1 और�2
d) न�तो�1 और�न�ही�2
QUESTION 39.
रा�य�पाट��के��प�म��एक�राजनी�तक�दल�को�मा�यता�दे ने�के�मानदं ड�के�संदभ��म���न�न�ल�खत�कथन��पर��वचार�कर�:

1. य�द�यह�संबं�धत�रा�य�के��वधान�सभा�के�आम�चुनाव�म��2% सीट� �जीतती�है।�


2. य�द�यह�रा�य�म��एक�आम�चुनाव�म��रा�य�से�लोकसभा�के��लए�चुने�गए�कुल�वैध�मत��का�6% �ा�त�करता�है।�

ऊपर��दए�गए�कथन��म��से�कौन�सा�सही�है�/ ह��?

a) केवल�1
b) केवल�2
c) दोन��1 और�2
d) न�तो�1 और�न�ही�2
QUESTION 40.
�न�न�ल�खत�कथन�यु�म��पर��वचार�कर�:

रा�य�रा�य�पाट��का�नाम�

1. महारा��महारा�वाद��गोमांतक�( MAG)
2. असम�ऑल�इं�डया�यूनाइटे ड�डेमो�े�टक��ंट�( AUDF)

IASbaba
Web: http://ilp.iasbaba.com/
Email: ilp@iasbaba.com
Page 44
AIPTS/ILP VETERANS-
Exam Title :
2020 TE...
Email : yadavanurag075@gmail.com
Contact : 8882839768

3. ह�रयाणा�इं�डयन�नेशनल�लोकदल�

ऊपर��दए�गए�कथन��म��से�कौन�सा�सही�है�/ ह��?

a) केवल�1 और�2
b) केवल�1 और�3
c) केवल�2 और�3
d) 1, 2 और�3
QUESTION 41.
�न�न�ल�खत�कथन��पर��वचार�कर��

1. संसद�और�रा�य��वधानसभा��के�चुनाव�के��लए���येक��े�ीय��नवा�चन��े��हेतु�अलग-अलग�सामा�य�मतदाता�सूची�होती�है।�
2. सं�वधान�ने�चुनावी�मता�धकार�के�मामले�म����येक�नाग�रक�को�समानता��दान�क��है।�
3. प�रसीमन�आयोग��ारा�जारी�आदे श�अं�तम�हो�जाते�ह��तथा��कसी�भी��यायालय�म��चुनौती�नह��द��जा�सकती�है।�

ऊपर��दए�गए�कथन��म��से�कौन�सा�सही�है�/ ह��?

a) केवल�1
b) केवल�1 और�3
c) केवल�2 और�3
d) 1, 2 और�3
QUESTION 42.
भारत�के�चुनावी�तं��( election machinery) के�संदभ��म���न�न�ल�खत�कथन��पर��वचार�कर�:

1. भारत�के�रा�प�त�मु�य�चुनाव�आयु��और�चुनाव�आयु���क���नयु���करते�ह�।�
2. मु�य��नवा�चन�अ�धकारी�क���नयु���भारत�के�चुनाव�आयोग��ारा�संबं�धत�रा�य�के�रा�यपाल�के�परामश��से�क��जाती�है।�
3. �जला��नवा�चन�अ�धकारी�क���नयु���रा�य�के�मु�य��नवा�चन�अ�धकारी��ारा�क��जाती�है।�

ऊपर��दए�गए�कथन��म��से�कौन�सा�सही�है�/ ह��?

a) केवल�1
b) केवल�1 और�3
c) केवल�2
d) 1, 2 और�3
QUESTION 43.
�न�न�ल�खत�कथन��पर��वचार�कर�:

1. �रट�न�ग�अ�धकारी�( Returning Officer) संबं�धत�संसद�य�या��वधानसभा��े��म��चुनाव�के�संचालन�के��लए�उ�रदायी�होता�है।�


2. पीठासीन�अ�धकारी�( Presiding Officer) एक�संसद�य�/ �वधानसभा��े��के��लए�मतदाता�सूची�तैयार�करने�के��लए�उ�रदायी
होता�है।�

ऊपर��दए�गए�कथन��म��से�कौन�सा�सही�है�/ ह��?

a) केवल�1
b) केवल�2
c) दोन��1 और�2
d) न�तो�1 और�न�ही�2

IASbaba
Web: http://ilp.iasbaba.com/
Email: ilp@iasbaba.com
Page 45
AIPTS/ILP VETERANS-
Exam Title :
2020 TE...
Email : yadavanurag075@gmail.com
Contact : 8882839768

QUESTION 44.
चुनाव����या�के�बारे�म���न�न�ल�खत�कथन��पर��वचार�कर�:

1. भारतीय�चुनाव�आयोग��ारा�चुनाव�काय��म�क��घोषणा�के�तुरंत�बाद�आदश��आचार�सं�हता�लागू�हो�जाती�है।�
2. संबं�धत�पीठासीन�अ�धकारी�अपने��नवा��चत��नवा�चन��े��के��लए�प�रणाम�घो�षत�करता�है।�

ऊपर��दए�गए�कथन��म��से�कौन�सा�सही�है�/ ह��?

a) केवल�1
b) केवल�2
c) दोन��1 और�2
d) न�तो�1 और�न�ही�2
QUESTION 45.
चुनाव�लड़ने�वाले�उ�मीदवार��क��शपथ�या�पु���के�संदभ��म���न�न�ल�खत�कथन��पर��वचार�कर�:

1. उ�मीदवार�अपनी�उ�मीदवारी�क��जांच�क���त�थ�के�बाद�शपथ�या�पु���कर�सकता�है।�
2. एक�उ�मीदवार�को��नवा�चन�अ�धकारी�और��नवा�चन��े��के��लए�सहायक��नवा�चन�अ�धकारी�के�सम��मु�य��प�से�शपथ�या�पु��
करनी�होती�है।�
3. य�द��कसी�उ�मीदवार�को�जेल�म��या��नरोधा�मक��हरासत�म��रखा�जाता�है�, तो�जेल��के�अधी�क�को�शपथ��दलाने�के��लए�अ�धकृत
�कया�जाता�है।�

ऊपर��दए�गए�कथन��म��से�कौन�सा�सही�है�/ ह��?

a) केवल�1 और�2
b) केवल�1 और�3
c) केवल�2 और�3
d) 1, 2 और�3
QUESTION 46.
�न�न�ल�खत�म��से�कौन�चुनाव�और�मतदान��वहार�म��मी�डया�क��सकारा�मक�भू�मका�नह��है�?

a) सूचना��सार�
b) पेड��यूज�
c) आदश��आचार�सं�हता�और�अ�य�कानून��का��वत�न�
d) मतदाता��श�ा�और�भागीदारी�
QUESTION 47.
�न�न�ल�खत�म��से�कौन�सा�कथन�जन��त�न�ध�व�अ�ध�नयम�, 1951 म���ावधान�ह��/ ह�:

1. इसम��केवल�संसद�और�रा�य��वधानसभा��क��सद�यता�के��लए�यो�यता�के��ावधान�शा�मल�ह��
2. यह�लोक�सभा�तथा�रा�य��क���वधानसभा��और��वधान�प�रषद��म��सीट��के�आवंटन�का��ावधान�करती�है।�
3. इसम�����आचरण�और�चुनावी�अपराध�शा�मल�ह�।�

सही�कूट�का�चयन�कर�:

a) केवल�1
b) केवल�2 और�3
c) केवल�3
d) 1, 2 और�3

IASbaba
Web: http://ilp.iasbaba.com/
Email: ilp@iasbaba.com
Page 46
AIPTS/ILP VETERANS-
Exam Title :
2020 TE...
Email : yadavanurag075@gmail.com
Contact : 8882839768

QUESTION 48.
�दनेश�गो�वामी�स�म�त��कससे�संबं�धत�थी:

a) दलबदल��वरोधी��ावधान�
b) चुनावी�सुधार�
c) मौ�लक�अ�धकार��से��नद� शा�मक��स�ांत��को��ाथ�मकता।�
d) मौ�लक�कत���
QUESTION 49.
‘ ए��जट�पोल�’ के�संबंध�म���न�न�ल�खत�म��से�कौन�सा�कथन�सही�है�?

a) ‘ ए��जट�पोल�’ एक�ऐसा�श�द�है��जसका�इ�तेमाल�मतदाता��के�चुनाव�के�बाद�के�सव��ण�को�सू�चत�करने�के��लए��कया�जाता�है�, �ज
सके�प��म��उ�ह�ने�अपने�मता�धकार�का��योग��कया�है�
b) ‘ ए��जट�पोल�’ और�‘ ओ�प�नयन�पोल�’ एक�और�समान�ह��
c) ‘ ए��जट�पोल�’ वह�उपकरण�है��जसके�मा�यम�से�मतदान�के�प�रणाम��क��सट�क�भ�व�यवाणी�क��जा�सकती�है�
d) ‘ ए��जट�पोल�’ एक��शास�नक�उपकरण�है��जसे�मु�य�चुनाव�आयु��ने���त�पण�( impersonation) को�रोकने�के��लए�हाल�ही
म��बनाया�है�
QUESTION 50.
दल-बदल�के�आधार�पर�अयो�यता�के��खलाफ�अपवाद��के�बारे�म���न�न�ल�खत�कथन��पर��वचार�कर�:

1. य�द�कोई�सद�य��कसी�अ�य�पाट��के�साथ�पाट��के��वलय�के�प�रणाम�व�प�अपनी�पाट��से�बाहर�जाता�है।�
2. �वधायक�दल�के�एक��तहाई�सद�य���ारा��वभाजन�के�मामले�म�।�

उपरो��कथन��म��से�कौन�सा�गलत�है�/ ह��?

a) केवल�1
b) केवल�2
c) दोन��1 और�2
d) न�तो�1 और�न�ही�2
QUESTION 51.
भारत�म��चुनाव�आयोग�के�संबंध�म��, नीचे��दए�गए�कथन��म��से�कौन�सा�/ से�सही�है�/ ह��?

1. यह�एक�अ�खल�भारतीय��नकाय�है�, �जसे�सीधे�भारत�के�सं�वधान��ारा��था�पत��कया�गया�है।�
2. यह�संसद�, रा�य��म��रा�य��वधानसभा��, पंचायत��और�नगरपा�लका��के��लए��वतं��और��न�प��चुनाव�सु�न��त�करता�है।�

सही��वक�प�चुन�:

a) केवल�1
b) केवल�2
c) दोन��1 और�2
d) न�तो�1 और�न�ही�2
QUESTION 52.
भाषाई�अ�पसं�यक��के��लए��वशेष�अ�धकारी�( Special Officer for Linguistic Minorities) के�बारे�म���न�न�ल�खत�म��से�कौन
सा�कथन�सही�है�/ ह��?

1. वह�रा�प�त��ारा��नयु���कया�जाता�है।�

IASbaba
Web: http://ilp.iasbaba.com/
Email: ilp@iasbaba.com
Page 47
AIPTS/ILP VETERANS-
Exam Title :
2020 TE...
Email : yadavanurag075@gmail.com
Contact : 8882839768

2. सं�वधान�भाषाई�अ�पसं�यक��के��लए��वशेष�अ�धकारी�क��यो�यता�, काय�काल�, वेतन�और�भ�े�, सेवा�शत��और�हटाने�क�����या


को��न�द���नह��करता�है।�

उ�चत��वक�प�चुन�:

a) केवल�1
b) केवल�2
c) दोन��1 और�2
d) न�तो�1 और�न�ही�2
QUESTION 53.
मु�य�चुनाव�आयु��( CEC) और�अ�य�चुनाव�आयु���( EC) के�बारे�म���न�न�ल�खत�कथन��पर��वचार�कर�।�सही�कथन�को�पहचान�:

a) वे�समान���थ�त�का�आनंद�लेते�ह��तथा�भारत�के�सव��च��यायालय�के��यायाधीश��के�समान�वेतन�और�भ�े��ा�त�करते�ह�।�
b) CEC और�अ�य�EC के�बीच�राय�के�अंतर�के�मामले�म��, मामला�CEC �ारा�तय��कया�जाता�है।�
c) उ�ह��अपने�काया�लय�से�उसी�तरीके�से�और�उसी�आधार�पर�हटाया�जा�सकता�है�, जैसे�सव��च��यायालय�के��यायाधीश�को।�
d) वे�रा�प�त�के��सादपय�त�अपने�पद�पर�बने�रहते�ह�।�
QUESTION 54.
�न�न�ल�खत�कथन��पर��वचार�कर��तथा�गलत�कथन�क��पहचान�कर�:

a) सं�वधान�ने�चुनाव�आयोग�के�सद�य��क��यो�यता��नधा��रत�नह��क��है।�
b) सं�वधान�ने�चुनाव�आयोग�के�सद�य��के�काय�काल�को��न�द���नह���कया�है।�
c) सं�वधान�ने�सेवा�नवृ��चुनाव�आयु���को�सरकार��ारा��कसी�और��नयु���से�वं�चत�नह���कया�है।�
d) चुनाव�आयोग�ने�अपनी��थापना�के�ठ�क�बाद�से�एक�ब�प�ीय��नकाय�के��प�म��काय���कया�है।�
QUESTION 55.
महा�यायवाद��( Attorney general) के�संदभ��म��, नीचे�से�गलत�कथन�क��पहचान�कर�:

a) वह�भारत�सरकार�क��अनुम�त�से�आपरा�धक�मुकदम��म��अ�भयु���क��र�ा�कर�सकता�है।�
b) वह�उन�मामल��म��एक�सलाह�दे �सकता�है�, जो�भारत�सरकार�के��लए�सलाह�दे ने�या�पेश�करने�के��लए�कहे�जाते�ह�।�
c) उ�ह��भारत�सरकार�क��अनुम�त�के��बना��कसी�कंपनी�या��नगम�म���नदे शक�के��प�म���नयु���को��वीकार�नह��करना�चा�हए।�
d) वह�सरकारी�नौकर��क���ेणी�म��नह��आता�है।�
QUESTION 56.
रा�य�लोक�सेवा�आयोग�के�बारे�म���न�न�ल�खत�कथन��पर��वचार�कर�:

1. सं�वधान�आयोग�क��साम�य��( strength) को��न�द���नह��करता�है�ले�कन�इस�मामले�को�रा�यपाल�के��ववेक�पर�छोड़��दया�है।�


2. आयोग�के�अ�य��और�सद�य�छह�वष��क��अव�ध�के��लए�या�65 वष��क��आयु��ा�त�करने�तक�, जो�भी�पहले�हो�, पद�धारण�करते�ह�।

ऊपर��दए�गए�कथन��म��से�कौन�सा�सही�है�/ ह��?

a) केवल�1
b) केवल�2
c) दोन��1 और�2
d) न�तो�1 और�न�ही�2
QUESTION 57.

IASbaba
Web: http://ilp.iasbaba.com/
Email: ilp@iasbaba.com
Page 48
AIPTS/ILP VETERANS-
Exam Title :
2020 TE...
Email : yadavanurag075@gmail.com
Contact : 8882839768

�न�न�ल�खत�कथन��पर��वचार�कर�:

1. �व��आयोग��ारा�क��गई��सफा�रश��केवल�सलाहकार��कृ�त�क��ह��और�इस�लए�, सरकार�पर�बा�यकारी�नह��ह�।�
2. UPSC �ारा�क��गई��सफा�रश��केवल�सलाहकार��कृ�त�क��ह��और�इस�लए�, सरकार�पर�बा�यकारी�नह��ह�।�

ऊपर��दए�गए�कथन��म��से�कौन�सा�सही�है�/ ह��?

a) केवल�1
b) केवल�2
c) दोन��1 और�2
d) न�तो�1 और�न�ही�2
QUESTION 58.
चुनाव�आयोग�के�संबंध�म���न�न�ल�खत�म��से�कौन�सा�कथन�सही�है�/ ह��?

1. चुनाव�आयु���क��सं�या�रा�प�त��ारा��नधा��रत�क��जाती�है।�
2. चुनाव�आयु���क��सेवा�और�काय�काल�क��शत��भी�रा�प�त��ारा��नधा��रत�क��जाती�ह�।�

सही��वक�प�चुन�:

a) केवल�1
b) केवल�2
c) दोन��1 और�2
d) न�तो�1 और�न�ही�2
QUESTION 59.
UPSC क���वतं�ता�के�संबंध�म��नीचे��दए�गए�कथन��पर��वचार�कर��

1. UPSC के�अ�य��या�सद�य�को�रा�प�त��ारा�पद�से�केवल�सं�वधान�म��व�ण�त�तरीके�से�और�उस�आधार�पर�हटाया�जा�सकता�है।�
2. UPSC के�अ�य��और�सद�य��के�संपूण���य��को�भारत�के�समे�कत�कोष�पर�भा�रत��कया�गया�है।�

ऊपर��दए�गए�कथन��म��से�कौन�सा�सही�है�/ ह��?

a) केवल�1
b) केवल�2
c) दोन��1 और�2
d) न�तो�1 और�न�ही�2
QUESTION 60.
UPSC �ारा��कए�गए�काय��के�संदभ��म��, नीचे��दए�गए�कथन��म��से�कौन�सा�/ से�सही�है�/ ह��?

1. य�द�ऐसा�करने�के��लए�दो�या�दो�से�अ�धक�रा�य��से�अनुरोध��कया�जाता�है�, तो�UPSC �कसी�रा�य�क��सभी�या�कोई�आव�यकता�


को�पूरा�करता�है।�
2. UPSC चुनावी��व�था�से�संबं�धत��ववाद��क��जांच�के��लए�अ�धका�रय��क���नयु���कर�सकता�है।�

उ�चत��वक�प�चुन�:

a) केवल�1
b) केवल�2
c) दोन��1 और�2
d) न�तो�1 और�न�ही�2

IASbaba
Web: http://ilp.iasbaba.com/
Email: ilp@iasbaba.com
Page 49
AIPTS/ILP VETERANS-
Exam Title :
2020 TE...
Email : yadavanurag075@gmail.com
Contact : 8882839768

QUESTION 61.
नीचे��दए�गए�कथन��पर��वचार�कर�:

1. रा�प�त�मु�य�चुनाव�आयु��( CEC) और�अ�य�चुनाव�आयु���दोन��क���नयु���करता�है।�


2. रा�प�त�के�परामश��के�बाद�ही�CEC, �े�ीय�आयु���क���नयु���कर�सकता�है।�

ऊपर��दए�गए�कथन��म��से�कौन�सा�सही�है�/ ह��?

a) केवल�1
b) केवल�2
c) दोन��1 और�2
d) न�तो�1 और�न�ही�2
QUESTION 62.
भारत�का�सं�वधान�भारत�म��राजकोषीय�संघवाद�के�संतुलन�च��के��प�म���न�न�ल�खत�म��से��कस��नकाय�क��प�रक�पना�करता�है�?

a) �नयं�क�और�महालेखा�परी�क�
b) �व��आयोग�
c) नी�त�आयोग�
d) क���य�सतक�ता�आयोग�
QUESTION 63.
रा�ीय�अनुसू�चत�जा�त�आयोग�के�बारे�म���न�न�ल�खत�कथन��पर��वचार�कर�:

1. यह�आव�यक�होने�पर�रा�प�त�को�एक��रपोट� ���तुत�कर�सकता�है।�
2. इसम���वयं�क�����या�को��व�नय�मत�करने�क��श����न�हत�है।�

ऊपर��दए�गए�कथन��म��से�कौन�सा�सही�है�/ ह��?

a) केवल�1
b) केवल�2
c) दोन��1 और�2
d) न�तो�1 और�न�ही�2
QUESTION 64.
�न�न�ल�खत�कथन��पर��वचार�कर��और�पहचान���क�कौन�सा�कथन�स�य�है�/ ह��?

a) य�द�चुनाव�केवल�संसद�के��लए�आयो�जत��कए�जा�रहे�ह��, तो��य�पूरी�तरह�से�क���सरकार��ारा�वहन��कया�जाता�है�, जब�क�केवल


रा�य��वधानमंडल�के��लए�होने�वाले�चुनाव��के��लए�, �य�पूरी�तरह�से�संबं�धत�रा�य��ारा�वहन��कया�जाता�है।�
b) संसद�और�रा�य��वधानमंडल�के��लए�एक�साथ�चुनाव�के�मामले�म��, �य�को�क���और�रा�य�सरकार��के�बीच�समान��प�से�साझा��कया
जाता�है।�
c) (a) और�(b) दोन��
d) कोई�नह��
QUESTION 65.
नीचे��दए�गए�कथन��म��से�कौन�सा�/ से�, चुनाव�आयोग�के�सलाहकारी��े�ा�धकार�और�समी�ा�काय��के�संदभ��म��सही�है�/ ह��?

1. एक�बार�चुनाव�पूरा�होने�और�प�रणाम�घो�षत�होने�के�बाद�, आयोग��वयं��कसी�भी�प�रणाम�क��समी�ा�नह��कर�सकता�है।�
2. सं�वधान�के�तहत�, आयोग�को�संसद�और�रा�य��वधानसभा��के�सद�य��के�चुनाव�के�बाद�अयो�य�ठहराए�जाने�के�मामले�म�
सलाहकारी��े�ा�धकार�है।�

IASbaba
Web: http://ilp.iasbaba.com/
Email: ilp@iasbaba.com
Page 50
AIPTS/ILP VETERANS-
Exam Title :
2020 TE...
Email : yadavanurag075@gmail.com
Contact : 8882839768

उ�चत��वक�प�चुन�:

a) केवल�1
b) केवल�2
c) दोन��1 और�2
d) न�तो�1 और�न�ही�2
QUESTION 66.
नीचे��दए�गए�कथन��पर��वचार�कर�:

1. रा�प�त�पद�, सेवा��और�मामल��को�UPSC के�दायरे�से�बाहर�कर�सकता�है।�


2. रा�प�त��कसी�भी��ा�धकरण�, कारपोरेट��नकाय�या�साव�ज�नक�सं�थान�के�का�म�क��णाली�को�UPSC के�अ�धकार��े��म��रख�सकते
ह�।�

ऊपर��दए�गए�कथन��म��से�कौन�सा�सही�है�/ ह��?

a) केवल�1
b) केवल�2
c) दोन��1 और�2
d) न�तो�1 और�न�ही�2
QUESTION 67.
UPSC के�काया��मक��े�ा�धकार�( functional jurisdiction) के�बाहर��न�न�ल�खत�म��से�कौन�सा�मामला�रखा�जाता�है�?

1. �कसी�भी��पछड़े�वग��के�नाग�रक��के�प��म���नयु��य��या�पद��का�आर�ण�करते�समय�UPSC से�परामश��नह���कया�जाता�है।�
2. सेवा��और�पद��पर��नयु��यां�करने�म��अनुसू�चत�जा�त�और�अनुसू�चत�जनजा�त�के�दाव��को��यान�म��रखते��ए�UPSC से�परामश�
नह���कया�जाता�है।�

उ�चत�उ�र�चुन�:

a) केवल�1
b) केवल�2
c) दोन��1 और�2
d) न�तो�1 और�न�ही�2
QUESTION 68.
�न�न�ल�खत�म��से�कौन��कसी�कर�या�शु�क�(अनु�छे द�279 के�तहत) क��शु���ा��त�क��जाँच�और��मा�णत�करता�है�?

a) कैग�( CAG)
b) �व��मं�ी�
c) अ�य��, �व��आयोग�
d) अ�य��, नी�त�आयोग�
QUESTION 69.
भाषाई�अ�पसं�यक��के��लए�आयु��के�बारे�म���न�न�ल�खत�कथन��पर��वचार�कर�:

1. उनक���नयु���भाषाई�अ�पसं�यक��के��लए��वशेष�अ�धकारी��ारा�क��जाती�है।�
2. क���य��तर�पर�, आयु��अ�पसं�यक�मामल��के�मं�ालय�के�अंतग�त�आता�है।�

ऊपर��दए�गए�कथन��म��से�कौन�सा�सही�है�/ ह��?

IASbaba
Web: http://ilp.iasbaba.com/
Email: ilp@iasbaba.com
Page 51
AIPTS/ILP VETERANS-
Exam Title :
2020 TE...
Email : yadavanurag075@gmail.com
Contact : 8882839768

a) केवल�1
b) केवल�2
c) दोन��1 और�2
d) न�तो�1 और�न�ही�2
QUESTION 70.
भारत�म��सरकार�क��लोकतां��क��णाली�के��पलर�( bulwark) के��प�म���न�न�ल�खत�म��से�कौन�सा�/ से�माना�जाता�है�?

1. �नयं�क�और�महालेखा�परी�क�
2. संघ�लोक�सेवा�आयोग�
3. चुनाव�आयोग�
4. सु�ीम�कोट� �

उ�चत�उ�र�का�चयन�कर�:

a) केवल�1
b) केवल�4
c) 1 और�2
d) 1, 2, 3 और�4
QUESTION 71.
�नयं�क�और�महालेखा�परी�क�( CAG) के�बारे�म���न�न�ल�खत�कथन��पर��वचार�कर�।�

1. वह�भारत�के�रा�प�त��ारा�अपने�हाथ�और�मुहर�( hand and seal) के�तहत�एक�वारंट��ारा��नयु���कया�जाता�है।�


2. उनका�वेतन�और�अ�य�सेवा�शत��रा�प�त��ारा��नधा��रत�क��जाती�ह�।�
3. वह�रा�प�त�को�उस�खाते�के��प�के�संबंध�म��सलाह�दे ता�है��जसम��क���और�रा�य��के�खाते�रखे�जाएंगे।�

ऊपर��दए�गए�कथन��म��से�कौन�सा�सही�है�/ ह��?

a) 1 और�2
b) 1 और�3
c) केवल�1
d) 1, 2 और�3
QUESTION 72.
�न�न�ल�खत�म��से�कौन�चुनाव�आयोग�क��श��य��और�काय��का��ह�सा�है�?

1. यह�पूरे�दे श�म���नवा�चन��े���के��े�ीय�भाग��( territorial areas) को��नधा��रत�करता�है।�


2. यह�राजनी�तक�दल��को�चुनाव��च�ह�आवं�टत�करता�है�तथा�उ�ह��चुनाव��च�ह�के�ऐसे�आवंटन�से�संबं�धत��ववाद��को��नपटाने�के��लए
एक�अदालत�के��प�म��काय��करता�है।�
3. यह�चुनाव��के�समय�म��रे�डयो�और�ट�वी�पर�राजनी�तक�दल��क��नी�तय��के��चार�के��लए�एक�रो�टर�( roster) तैयार�कर�सकता�है।�

सही��वक�प�चुन�:

a) केवल�1
b) केवल�2
c) 1 और�2
d) 1, 2 और�3
QUESTION 73.

IASbaba
Web: http://ilp.iasbaba.com/
Email: ilp@iasbaba.com
Page 52
AIPTS/ILP VETERANS-
Exam Title :
2020 TE...
Email : yadavanurag075@gmail.com
Contact : 8882839768

�न�न�ल�खत�कथन��पर��वचार�कर�:

1. CAG भारत�के�समे�कत��न�ध�, ��येक�रा�य�के�समे�कत��न�ध�और���येक��वधानसभा��े��वाले�क���शा�सत��दे श�क��समे�कत��न�ध


से�सभी��य�से�संबं�धत�खात��का�ऑ�डट�करता�है।�
2. CAG भारत�के�आक��मकता�कोष�और�भारत�के�साव�ज�नक�खाते�के�साथ-साथ���येक�रा�य�के�आक��मक��न�ध�और���येक�रा�य
के�साव�ज�नक�खाते�से�सभी��य�का�ऑ�डट�करता�है।�

ऊपर��दए�गए�कथन��म��से�कौन�सा�सही�है�/ ह��?

a) केवल�1
b) केवल�2
c) दोन��1 और�2
d) न�तो�1 और�न�ही�2
QUESTION 74.
CAG के�बारे�म���न�न�ल�खत�कथन��पर��वचार�कर��और�गलत�कथन�क��पहचान�कर�:

a) CAG संसद�का�एक�एज�ट�है।�
b) CAG केवल�संसद�के���त�उ�रदायी�है।�
c) CAG क��ओर�से�कानूनी�, �नयामक�और�औ�च�य�लेखा�परी�ा�अ�नवाय��है।�
d) भारत�का�सं�वधान�CAG को��नयं�क�और�महालेखा�परी�क�के��प�म��बताता�है।�
QUESTION 75.
�न�न�ल�खत�म��से��कसे�रा�प�त��ारा�सव��च��यायालय�के��यायाधीश�के�समान�उसी�तरीके�और�आधार�पर�हटाया�जा�सकता�है�?

1. �नयं�क�और�महालेखा�परी�क�
2. मु�य�चुनाव�आयु��
3. उ�च��यायालय�के��यायाधीश�

उ�चत��वक�प�चुन�:

a) केवल�1
b) केवल�2
c) 1 और�2
d) 1, 2 और�3
QUESTION 76.
�न�न�ल�खत�म��से�कौन�नी�त�आयोग�काय��णाली�के��लए�माग�दश�क��स�ांत��का��ह�सा�ह��?

1. अ��योदय�( Antyodaya)
2. समावेश�( Inclusion)
3. सततता�( Sustainability)
4. जनसां��यक�य�लाभांश�

सही��वक�प�चुन�:

a) 1 और�2
b) 1, 3 और�4
c) 2 और�3
d) 1, 2, 3 और�4

IASbaba
Web: http://ilp.iasbaba.com/
Email: ilp@iasbaba.com
Page 53
AIPTS/ILP VETERANS-
Exam Title :
2020 TE...
Email : yadavanurag075@gmail.com
Contact : 8882839768

QUESTION 77.
�न�न�ल�खत�म��से��कसे�दे श�का�सव��च�कानून�अ�धकारी�माना�जाता�है�?

a) भारत�के�मु�य��यायाधीश�
b) महा�यायवाद��
c) महा�धव�ा�
d) रा�प�त�
QUESTION 78.
महा�धव�ा�( Advocate General) के�संदभ��म���न�न�ल�खत�कथन��पर��वचार�कर��और�गलत�कथन�क��पहचान�कर�:

a) वह�रा�प�त��ारा��नयु���कया�जाता�है�और�रा�प�त�के��सादपय�त�पद�धारण�करता�है।�
b) महा�धव�ा�के�काया�लय�का�काय�काल�सं�वधान��ारा��नधा��रत�नह��है।�
c) सं�वधान��ारा�महा�धव�ा�का�पा�र��मक�तय�नह��है।�
d) कोई�नह��
QUESTION 79.
�न�न�ल�खत�म��से�कौन�रा�प�त�के��सादपय�त�( pleasure) अपना�पद�धारण�नह��करता�है�?

1. �नयं�क�और�महालेखा�परी�क�
2. मु�य�चुनाव�आयु��
3. मं��य��( Ministers)
4. महा�यायवाद��
5. एक�उ�च��यायालय�के��यायाधीश�

उ�चत��वक�प�चुन�:

a) केवल�1 और�2
b) केवल�1, 2 और�5
c) 1, 2, 3 और�5
d) 1, 2, 3, 4 और�5
QUESTION 80.
भारत�के�सॉ�ल�सटर�जनरल�के�संबंध�म���न�न�ल�खत�कथन��पर��वचार�कर�:

1. सॉ�ल�सटर�जनरल�, अटॉन��जनरल�को�अपनी�आ�धका�रक��ज�मेदा�रय��क��पू�त��म��सहायता�करता�है।�
2. सं�वधान�म��भारत�के�सॉ�ल�सटर�जनरल�के�बारे�म��कह��नह���लखा�गया�है।�
3. कै�बनेट�क���नयु���स�म�त�( ACC) भारत�के�सॉ�ल�सटर�जनरल�क���नयु���करती�है।�

ऊपर��दए�गए�कथन��म��से�कौन�सा�सही�है�/ ह��?

a) केवल�1
b) केवल�1 और�2
c) केवल�2 और�3
d) 1, 2 और�3
QUESTION 81.
नीचे��दए�गए�कथन��म��से�कौन�सा�UPSC के�बारे�म��स�य�नह��है�?

1. UPSC सेवा��के�वग�करण�, वेतन�और�सेवा�शत��, कैडर��बंधन�, ��श�ण�, और�इसी�तरह�के�अ�य�से�संबं��नह��है।�

IASbaba
Web: http://ilp.iasbaba.com/
Email: ilp@iasbaba.com
Page 54
AIPTS/ILP VETERANS-
Exam Title :
2020 TE...
Email : yadavanurag075@gmail.com
Contact : 8882839768

2. UPSC भारत�म��एक�क���य�का�म�क�एज�सी�है�तथा�इसे�अ�सर�भारत�म��' मे�रट��स�टम�का�रखवाला�' (watch-dog of merit


system) माना�जाता�है।�

उ�चत��वक�प�चुन�:

a) केवल�1
b) केवल�2
c) दोन��1 और�2
d) न�तो�1 और�न�ही�2
QUESTION 82.
महा�यायवाद��( AG) के�बारे�म���न�न�ल�खत�म��से�कौन�सा�कथन�सही�है�/ ह��?

1. उसे�संसद�के�दोन��सदन��क��काय�वाही�म��बोलने�और�भाग�लेने�का�अ�धकार�है।�
2. उसे�संसद�क��संयु��बैठक�म��बोलने�और�भाग�लेने�का�अ�धकार�है।�
3. वह�उन�सभी��वशेषा�धकार��और���तर�ा��का�आनंद�लेता�है�जो�संसद�के�सद�य�के��लए�उपल�ध�ह�।�

उ�चत��वक�प�चुन�:

a) केवल�1
b) केवल�1 और�2
c) केवल�2 और�3
d) 1, 2 और�3
QUESTION 83.
�न�न�ल�खत�कत����पर��वचार�कर�:

1. ऐसे�कानूनी�मामल��पर�रा�य�क��सरकार�को�सलाह�दे ना�जो�रा�यपाल��ारा�उसे�संद�भ�त��कया�जाता�है।�
2. एक�कानूनी�च�र��के�ऐसे�अ�य�कत����को��नभाने�के��लए�जो�रा�यपाल��ारा�उसे�स�पे�जाते�ह�।�
3. अपने�आ�धका�रक�कत����के��दश�न�म��, वह�रा�य�के�कानून�के��कसी�भी��यायालय�के�सम��उप��थत�होने�का�हकदार�है।�

उपरो��कत����से�संबं�धत�ह��-

a) महा�यायवाद��
b) महा�धव�ा�
c) सॉ�ल�सटर�जनरल�
d) ��यात��याय�वद�
QUESTION 84.
नी�त�आयोग�के�संदभ��म��, नीचे��दए�गए�कथन��म��से�कौन-सा�/ से�सही�है�/ ह��?

1. योजना�आयोग�क��तरह�, यह�भी�न�तो�एक�संवैधा�नक��नकाय�है�और�न�ही�एक�वैधा�नक��नकाय�है।�
2. यह�भारत�सरकार�क���मुख�' �थ�क�ट� क�' है।�

सही�उ�र�चुन�:

a) केवल�1
b) केवल�2
c) दोन��1 और�2
d) न�तो�1 और�न�ही�2

IASbaba
Web: http://ilp.iasbaba.com/
Email: ilp@iasbaba.com
Page 55
AIPTS/ILP VETERANS-
Exam Title :
2020 TE...
Email : yadavanurag075@gmail.com
Contact : 8882839768

QUESTION 85.
नी�त�आयोग�क��शासन�प�रषद�( Governing Council) म��शा�मल�ह��-

1. सभी�रा�य��के�मु�यमं�ी�
2. �वधानसभा��वाले�क���शा�सत��दे श��के�मु�यमं�ी�
3. सभी�रा�य��और�क���शा�सत��दे श��के�रा�यपाल�

उपयु��कूट�चुन�:

a) केवल�1
b) केवल�1 और�2
c) केवल�2 और�3
d) 1, 2 और�3
QUESTION 86.
�न�न�ल�खत�कथन��पर��वचार�कर�:

1. कैग�रा�प�त�को�तीन�ऑ�डट��रपोट� �दे ता�है- �व�नयोग�खात��पर�ऑ�डट��रपोट� �, �व��खात��पर�ऑ�डट��रपोट� �और�साव�ज�नक�उप�म�


पर�ऑ�डट��रपोट� ।�
2. भारत�म��, काय�पा�लका�सरकारी�खजाने�से�केवल�कैग�क��मंजूरी�के�साथ�पैसे��नकाल�सकती�है।�

ऊपर��दए�गए�कथन��म��से�कौन�सा�सही�है�/ ह��?

a) केवल�1
b) केवल�2
c) दोन��1 और�2
d) न�तो�1 और�न�ही�2
QUESTION 87.
नी�त�आयोग�के��भावी��शासन�के��लए�सात��तंभ�म��से�नीचे��दए�गए�कौन�से�ह��?

1. सभी�पहलु��म��म�हला��को�सश��बनाना।�
2. युवा��के��लए�अवसर�क��समानता।�
3. �ौ�ो�गक��के�उपयोग�के�मा�यम�से�पारद�श�ता।�
4. आतंकवाद�का�मुकाबला।�

उ�चत��वक�प�चुन�:

a) 1 और�2
b) 2 और�4
c) 1, 2 और�3
d) 1, 2, 3 और�4
QUESTION 88.
रा�ीय��म�अथ�शा���अनुसंधान�एवं��वकास�सं�थान�( NILERD) के�बारे�म���न�न�ल�खत�कथन��पर��वचार�कर�:

1. यह�नी�त�आयोग�का�एक�अधीन�थ�काया�लय�है।�
2. यह�मु�य��प�से�नी�त�आयोग�क��अनुदान�सहायता��ारा��व��पो�षत�है।�
3. इं�ट��ूट�ऑफ�ए�लाइड�मैनपावर��रसच��( IAMR), �जसे�1962 म��सोसायट��पंजीकरण�अ�ध�नयम�, 1860 के�तहत��था�पत
�कया�गया�था�, का�नाम�बदलकर�NILERD कर��दया�गया�है।�

IASbaba
Web: http://ilp.iasbaba.com/
Email: ilp@iasbaba.com
Page 56
AIPTS/ILP VETERANS-
Exam Title :
2020 TE...
Email : yadavanurag075@gmail.com
Contact : 8882839768

ऊपर��दए�गए�कथन��म��से�कौन�सा�सही�है�/ ह��?

a) केवल�1
b) केवल�1 और�2
c) केवल�2 और�3
d) 1, 2 और�3
QUESTION 89.
रा�ीय�मानवा�धकार�आयोग�के�बारे�म���न�न�ल�खत�म��से�कौन�सा�कथन�स�य�है�/ नह��है�?

1. यह�एक�वैधा�नक��नकाय�है��जसे�2006 म��संसद��ारा�अ�ध�नय�मत�कानून�के�तहत��था�पत��कया�गया�था।�
2. यह�एक�ब�-सद�यीय��नकाय�है��जसम��एक�अ�य��और�चार�सद�य�होते�ह�।�
3. यह��व-सं�ान�( suo motu) के�आधार�पर�मानवा�धकार��के��कसी�भी�उ�लंघन�क��जांच�कर�सकता�है।�

उपयु���वक�प�चुन�:

a) केवल�1
b) केवल�1 और�2
c) केवल�2 और�3
d) 1, 2 और�3
QUESTION 90.
�न�न�ल�खत�कथन��पर��वचार�कर�:

1. रा�ीय�मानवा�धकार�आयोग�के�अ�य��क���नयु���रा�प�त��ारा�क��जाती�है।�
2. रा�ीय�मानवा�धकार�आयोग�के�सद�य��को��धानमं�ी�क��अ�य�ता�और�लोकसभा�अ�य��, रा�यसभा�के�उपा�य��, संसद�के�दोन�
सदन��म���वप��के�नेता�और�क���य�गृह�मं�ी�वाली�छह�सद�यीय�स�म�त��ारा��नयु���कया�जाता�है।�

ऊपर��दए�गए�कथन��म��से�कौन�सा�सही�है�/ ह��?

a) केवल�1
b) केवल�2
c) दोन��1 और�2
d) न�तो�1 और�न�ही�2
QUESTION 91.
�न�न�ल�खत�म��से�कौन�पाँच�वष��क��अव�ध�के��लए�या�जब�तक�वे�65 वष��क��आयु��ा�त�नह��कर�लेते�ह��, पद�धारण�करते�ह��?

a) NHRC के�अ�य��और�सद�य�
b) SHRC के�अ�य��और�सद�य�
c) CAT के�अ�य��और�सद�य�
d) इनम��से�कोई�भी�नह��
QUESTION 92.
सं�वधान�ने�सरकार�को��न�न�ल�खत�म��से��कस�को�सेवा�समा��त�के�बाद�आगे�क���नयु���के��लए�रोका�नह��है�?

1. सेवा�नवृ��चुनाव�आयु��
2. CAG
3. NHRC और�SHRC के�सद�य�

सही�उ�र�चुन�:

IASbaba
Web: http://ilp.iasbaba.com/
Email: ilp@iasbaba.com
Page 57
AIPTS/ILP VETERANS-
Exam Title :
2020 TE...
Email : yadavanurag075@gmail.com
Contact : 8882839768

a) केवल�1
b) 1 और�2
c) 1, 2 और�3
d) कोई�नह��
QUESTION 93.
NHRC और�SHRC के�बारे�म���न�न�ल�खत�कथन��पर��वचार�कर�:

1. NHRC के�अ�य��या�सद�य�के�वेतन�, भ�े�और�अ�य�शत��क���सरकार��ारा��नधा��रत�क��जाती�ह�।�


2. SHRC के�अ�य��या�सद�य�के�वेतन�, भ�े�और�अ�य�शत��का��नधा�रण�रा�य�सरकार��ारा��कया�जाता�है।�

ऊपर��दए�गए�कथन��म��से�कौन�सा�सही�है�/ ह��?

a) केवल�1
b) केवल�2
c) दोन��1 और�2
d) न�तो�1 और�न�ही�2
QUESTION 94.
रा�ीय�मानवा�धकार�आयोग�के�संदभ��म��, गलत�कथन�क��पहचान�कर�:

a) इसम��अपनी����या�को��व�नय�मत�करने�क��श����न�हत�है।�
b) इसम���स�वल��यायालय�क��सभी�श��याँ�होती�ह��तथा�इसक��काय�वाही�म��एक��या�यक�च�र��होता�है।�
c) यह�क���और�रा�य�सरकार��या��कसी�अ�य�अधीन�थ��ा�धकारी�से�सूचना�या��रपोट� �के��लए�मांग�कर�सकता�है।�
d) मानवा�धकार��के�उ�लंघन�क���शकायत��क��जांच�के��लए�जांच�कम�चा�रय��का�अपना�संघटक�( nucleus) नह��है।�
QUESTION 95.
NHRC या�SHRC के�बारे�म���न�न�ल�खत�कथन��पर��वचार�कर�:

1. इसम��मानवा�धकार��के�उ�लंघन�करने�वाल��को�दं �डत�करने�क��, न�ही�पी�ड़त�को�मौ��क�राहत�स�हत�कोई�राहत�दे ने�के��लए�कोई


श���नह��है।�
2. आयोग�के�काय���कृ�त�म��मु�य��प�से��सफा�रशी�ह�।�

ऊपर��दए�गए�कथन��म��से�कौन�सा�गलत�है�/ ह��?

a) केवल�1
b) केवल�2
c) दोन��1 और�2
d) न�तो�1 और�न�ही�2
QUESTION 96.
�न�न�ल�खत�म��से�कौन�रा�प�त�को�अपनी�वा�ष�क�या��वशेष��रपोट� ���तुत�नह��करता�है�?

a) कैग�( CAG)
b) भाषाई�अ�पसं�यक��के��लए�आयु��
c) रा�ीय�मानवा�धकार�आयोग�
d) कोई�नह��
QUESTION 97.
लोकायु��और�उपलोकायु���कसके��ारा��नयु��होते�ह��-

IASbaba
Web: http://ilp.iasbaba.com/
Email: ilp@iasbaba.com
Page 58
AIPTS/ILP VETERANS-
Exam Title :
2020 TE...
Email : yadavanurag075@gmail.com
Contact : 8882839768

a) मु�यमं�ी�
b) रा�यपाल�
c) उ�च��यायालय�के��यायाधीश�
d) उ�च��यायालय�के�मु�य��यायाधीश�
QUESTION 98.
क���य�सूचना�आयोग�( CIC) के�बारे�म���न�न�ल�खत�कथन��पर��वचार�कर�:

1. वह�क���सरकार�, रा�य�सरकार�और�क���शा�सत��दे श��के�अधीन�काया�लय��, �व�ीय�सं�थान��, साव�ज�नक�उप�म��आ�द�से�संबं�धत


�शकायत��और�अपील��क��सुनवाई�कर�सकता�है।�
2. वह�5 वष��क��अव�ध�तक�या�65 वष��क��आयु��ा�त�करने�तक�, जो�भी�पहले�हो�, पद�धारण�कर�सकता�है।�

ऊपर��दए�गए�कथन��म��से�कौन�सा�सही�है�/ ह��?

a) केवल�1
b) केवल�2
c) दोन��1 और�2
d) न�तो�1 और�न�ही�2
QUESTION 99.
UPSC के��े�ा�धकार�को��कसके��ारा�बढ़ाया�जा�सकता�है�-

a) गृह�मं�ालय�
b) का�म�क�मं�ालय�
c) रा�प�त�
d) संसद�
QUESTION 100.
मानवा�धकार��यायालय��( Human Rights Courts) के�संबंध�म���न�न�ल�खत�कथन��पर��वचार�कर�:

1. मानवा�धकार�अ�ध�नयम�क��सुर�ा�के��लए���येक��जले�म��मानवा�धकार��यायालय�क���थापना�क��गयी�है।�
2. मानवा�धकार��यायालय�रा�य�सरकार��ारा�केवल�उस�रा�य�के�उ�च��यायालय�के�मु�य��यायाधीश�क��सहम�त�से��था�पत�क��जा
सकते�ह�।�

ऊपर��दए�गए�कथन��म��से�कौन�सा�सही�है�/ ह��?

a) केवल�1
b) केवल�2
c) दोन��1 और�2
d) न�तो�1 और�न�ही�2

IASbaba
Web: http://ilp.iasbaba.com/
Email: ilp@iasbaba.com
Page 59
AIPTS/ILP VETERANS-
Exam Title :
2020 TE...
Email : yadavanurag075@gmail.com
Contact : 8882839768

Review in Hindi
QUESTION 1.
73 व��संवैधा�नक�संशोधन�अ�ध�नयम�, 1992 के�तहत���तुत��कए�गए�पंचायती�राज�म���मुखता�से�संचा�लत�होता�है:

a) म�यवत��और��जला��तर।�
b) स�म�त�और��जला��तर।�
c) स�म�त�और�म�यवत���तर।�
d) �ाम�, म�यवत��और��जला��तर�
Correct Answer: D
Your Answer:
Explanation

Solution (d)

Basic Information:

• 73 वाँ�संवैधा�नक�संशोधन�अ�ध�नयम�, 1992 ��येक�रा�य�म��पंचायती�राज�क�����तरीय��व�था��दान�करता�है�, अथा�त्��ाम�म�


पंचायत��, म�यवत��(�लॉक) और��जला��तर�पर।�यह�अ�ध�नयम�पूरे�दे श�म��पंचायती�राज�क��संरचना�म��एक�पता�लाता�है।�
• हालाँ�क�, 20 लाख�से�कम�आबाद��वाले�रा�य�म��म�यवत���तर�पर�पंचायत��नह��हो�सकती�ह�।�
• यह�एक�अ�नवाय���ावधान�है।�
• �ाम�, म�यवत��और��जला��तर��पर�पंचायत��क��सभी�सीट��पर���य��चुनाव�होते�ह�।�तथा�म�यवत��और��जला��तर��पर�पंचायत��के
अ�य��के�पद�के��लए�अ��य��चुनाव�होते�ह�।�
• बलवंत�राय�मेहता�ने��ाम��तर�पर�एक����तरीय�पंचायती�राज��व�था- �ाम�पंचायत�, �लॉक��तर�पर�पंचायत�स�म�त�और��जला��तर
पर��जला�प�रषद�क���थापना�क���सफा�रश�क��थी।�
• थुंगन�और�गाड�गल�स�म�तय��ने�भी�पंचायती�राज�क�����तरीय��णाली�क���सफा�रश��ाम�, �लॉक�और��जला��तर�पर�पंचायत��के
साथ�क��थी।�

Elimination:

य�द�आप�जानते�ह��(और�आपको�यह�जानना�चा�हए) �क�पंचायती�राज��णाली���-�तरीय�है�, तो�केवल��वक�प�( d) म����-�तर��के�सभी


उपयु��नाम�ह��, इस�लए�हमारा�उ�र��वक�प�( d) होगा।�

QUESTION 2.
�न�न�ल�खत�म��से�कौन�सा�कथन�सही�है�/ ह��?

1. पंचायत��के�चुनाव�लड़ने�के��लए��कसी�भी�����के��लए��नधा��रत��यूनतम�आयु�25 वष��है।�
2. समय�से�पहले��वघटन�के�बाद�एक�पंचायत�का�पुनग�ठन�केवल�शेष�अव�ध�के��लए�जारी�रहता�है।�

नीचे��दए�गए�कूट�का�उपयोग�करके�सही�उ�र�चुन�:

a) केवल�1
b) केवल�2
c) दोन��1 और�2
d) न�तो�1 और�न�ही�2
Correct Answer: B
Your Answer:
Explanation

Solution (b)

IASbaba
Web: http://ilp.iasbaba.com/ Score:
Email: ilp@iasbaba.com 0.00 / 200
Page 102
AIPTS/ILP VETERANS-
Exam Title :
2020 TE...
Email : yadavanurag075@gmail.com
Contact : 8882839768

Basic Information:

• 73 व��संवैधा�नक�संशोधन�अ�ध�नयम�, 1992 के�अनुसार�, �ाम�पंचायत�के�सद�य�या�सरपंच�के�उ�मीदवार�के��प�म��चुनाव�लड़ने


के��लए�, उ�मीदवार�क��आयु�21 वष��से�कम�नह��होनी�चा�हए।�
• अ�ध�नयम���येक��तर�पर�पंचायत�को�पांच�साल�के��लए�काय�काल��दान�करता�है।�हालां�क�, यह�अपने�काय�काल�के�पूरा�होने�से
पहले�भी�भंग�हो�सकती�है।�
• इसके�अलावा�, पंचायत�का�गठन�करने�के��लए�नए�चुनाव�पूरे��कए�जाएंगे�

(a) पांच�साल�क��अव�ध�समा�त�होने�से�पहले�; या�

(b) �वघटन�के�मामले�म��, इसके��वघटन�क��तारीख�से�छह�महीने�क��अव�ध�समा�त�होने�से�पहले।�

• ले�कन�, जहां�शेष�अव�ध�(�जसके��लए�भंग��ई�पंचायत�जारी�रहती�है) छह�महीने�से�कम�है�, तो�ऐसी�अव�ध�के��लए�नई�पंचायत�के


गठन�के��लए�कोई�चुनाव�आयो�जत�करना�आव�यक�नह��होगा।�
• इसके�अलावा�, समय�से�पहले��वघटन�के�बाद�एक�पंचायत�का�पुनग�ठन�पांच�साल�क��पूरी�अव�ध�के��लए�नह��होता�है�, ब��क�केवल
शेष�अव�ध�के��लए�काया�लय�म��बने�रहता�है।�

कथन��व�ेषण:

कथन�1 कथन�2

अस�य� स�य�

समय�से�पहले��वघटन�के�बाद�एक�पंचायत�का�पुनग�ठन�(यानी
अ�नवाय���ावधान��के�तहत�पंचायत��के�चुनाव�लड़ने�के��लए
पांच�साल�क��पूण��अव�ध�क��समा��त�से�पहले) केवल�शेष
�यूनतम�आयु�21 वष��होनी�चा�हए।�
अव�ध�के��लए�ही�जारी�रहेगा।�

QUESTION 3.
�न�न�ल�खत�म��से�कौन�से�रा�य�पंचायती�राज��ारा�कवर�नह���कये�गये�ह��?

a) ��पुरा�और��स��कम।�
b) म�णपुर�और�असम।�
c) मेघालय�और�नागाल�ड।�
d) उपरो��सभी�
Correct Answer: C
Your Answer:
Explanation

Solution (c)

Basic Information:

IASbaba
Web: http://ilp.iasbaba.com/ Score:
Email: ilp@iasbaba.com 0.00 / 200
Page 103
AIPTS/ILP VETERANS-
Exam Title :
2020 TE...
Email : yadavanurag075@gmail.com
Contact : 8882839768

73 वां�संवैधा�नक�संशोधन�अ�ध�नयम�, 1992 ज�मू�और�क�मीर�, नागाल�ड�, मेघालय�और��मजोरम�और�कुछ�अ�य��े���म��लागू�नह��होता


है।�

• इन��े���म��शा�मल�ह�:
◦ रा�य��म��अनुसू�चत��े��और�आ�दवासी��े��
◦ म�णपुर�का�पहाड़ी��े���जसके��लए�एक��जला�प�रषद�मौजूद�है�; तथा�
◦ प��म�बंगाल�का�दा�ज��ल�ग��जला��जसके��लए�दा�ज��ल�ग�गोरखा��हल�काउं�सल�मौजूद�है।�
• मेघालय�म��केवल��वाय���जला�प�रषद�है�और�नागाल�ड�म��केवल��ाम�प�रषद�है�जो�अ�ध�नयम�के�अनुसार�पंचायती�राज�सं�था��का
�ह�सा�नह��है।�
• हालाँ�क�, संसद�इस��ह�से�के��ावधान��को�अनुसू�चत��े���और�जनजातीय��े���म��ऐसे�अपवाद��और�संशोधन��के�अधीन��व�तार�कर
सकती�है�, जैसा��क�यह��न�द���कर�सकती�है।�
• ��पुरा�म���ाम�पंचायत��, पंचायत�स�म�तयाँ�, �जला�पंचायत��और��वाय���जला�प�रषद�ह�।�
• �स��कम�म���ाम�पंचायत��और��जला�पंचायत��ह���जनका�म�यवत���तर�नह��है।�
• म�णपुर�म���ाम�पंचायत�, �जला�पंचायत�और�एक��वाय���जला�प�रषद�ह�।�
• असम�म��गांव�पंचायत��, आंच�लक�पंचायत��, �जला�प�रषद�और�एक��वाय���जला�प�रषद�ह�।�

QUESTION 4.
73 वां�संवैधा�नक�संशोधन�अ�ध�नयम�, 1992, �जसका�उ�े �य�दे श�म��पंचायती�राज�सं�थान��को�बढ़ावा�दे ना�है�, �न�न�ल�खत�म��से��कसके
�लए��ावधान�करता�है�?

1. �जला�योजना�स�म�तय��का�गठन।�
2. पंचायत��म��चुनाव�कराने�के��लए�रा�य�चुनाव�आयोग।�
3. रा�य��व��आयोग�का�गठन।�

उपरो��कथन��म��से�कौन�सा�सही�है�/ ह��?

a) केवल�1
b) केवल�1 और�2
c) केवल�2 और�3
d) 1, 2 और�3
Correct Answer: C
Your Answer:
Explanation

Solution (c)

Basic Information:

73 व��संवैधा�नक�संशोधन�अ�ध�नयम�, 1992 के��ावधान�रा�य��नवा�चन�आयोग��और�रा�य��व��आयोग�के�गठन�के��लए��ावधान��दान


करते�ह�:

• पंचायत��के�चुनाव�कराने�के��लए�एक�रा�य��नवा�चन�आयोग।�
• पंचायत��क���व�ीय���थ�त�क��समी�ा�करने�के��लए���येक�पांच�साल�के�बाद�एक�रा�य��व��आयोग।�

यह�भी��दान�करता�है:

• �कसी�गाँव�या�गाँव��के�समूह�म���ाम�सभा�का�संगठन।�
• �ाम�, म�यवत��और��जला��तर��पर�पंचायत��क���थापना।�

कथन��व�ेषण:

IASbaba
Web: http://ilp.iasbaba.com/ Score:
Email: ilp@iasbaba.com 0.00 / 200
Page 104
AIPTS/ILP VETERANS-
Exam Title :
2020 TE...
Email : yadavanurag075@gmail.com
Contact : 8882839768

कथन�1 कथन�2 कथन�3

अस�य� स�य� स�य�

�जला�योजना�स�म�त�73 व� पंचायत��क���व�ीय���थ�त�क��समी�ा�करने
पंचायत��के�चुनाव�कराने�के��लए�रा�य
संशोधन�म��नह��, ब��क�74 व� के��लए���येक�पांच�साल�के�बाद�रा�य��व�
�नवा�चन�आयोग�का�गठन।�
संशोधन�के�तहत�आती�है।� आयोग�का�गठन।�

अ�त�र��जानकारी:

�जला�योजना�स�म�त:

• 1992 के�74 व��संवैधा�नक�संशोधन�अ�ध�नयम�म��कहा�गया�है��क�, ��येक�रा�य��जला��तर�पर�, �जले�म��पंचायत��और


नगरपा�लका���ारा�तैयार�क��गई�योजना��को�समे�कत�करने�के��लए�, और�सम���प�से��जले�के��लए�एक�मसौदा��वकास�योजना
तैयार�करने�के��लए�एक��जला�योजना�स�म�त�का�गठन�करेगा।�
• यह�अ�ध�नयम�इस�बात�क��पु���करता�है��क��जला�योजना�स�म�त�के�सद�य��म��से�4/5 सद�य��जला�पंचायत�और�नगर�पा�लका�
के��नवा��चत�सद�य���ारा�आपस�म��चुने�जाय�गे।�स�म�त�म��इन�सद�य��का���त�न�ध�व��जले�म���ामीण�और�शहरी�आबाद��के�अनुपात
के�अनुपात�म��होना�चा�हए।�

QUESTION 5.
�न�न�ल�खत�म��से�कौन�पंचायती�राज�के��े��म��73 व��संवैधा�नक�संशोधन��ारा���ता�वत�नह��था�?

a) सभी��नवा��चत��ामीण��थानीय��नकाय��म��, जो�कुल�सीट��क��एक��तहाई�से�कम�नह��होगी�, सभी��तर��पर�म�हला�उ�मीदवार��के��लए


आर��त�होगी।�
b) रा�य�पंचायती�राज�सं�था��को�संसाधन�आवं�टत�करने�के��लए�अपने��व��आयोग��का�गठन�कर�गे।�
c) य�द�दो�से�अ�धक�ब�चे�ह��तो�पंचायती�राज�पदा�धका�रय��को�अपने�काया�लय�म��बने�रहने�के��लए�अयो�य�घो�षत�कर��दया�जाएगा।�
d) पंचायती�राज��नकाय��को�रा�य�सरकार��ारा�अ�धगृहीत�या�भंग�कर��दए�जाने�पर�छह�महीने�के�अंदर�चुनाव�ह�गे।�
Correct Answer: C
Your Answer:
Explanation

Solution (c)

Basic Information:

• 1992 के�73 व��संवैधा�नक�संशोधन�अ�ध�नयम�म��दो�से�अ�धक�जी�वत�ब�च��वाले�पंचायती�राज�पदा�धका�रय��को�अयो�य�ठहराने


का�कोई��ावधान���ता�वत�नह���कया�गया�था।�
• के. क�णाकरन�क��अ�य�ता�म��1992 म��जनसं�या�पर�एक�स�म�त�क���थापना�के�साथ�दो�ब�चे�के�मानदं ड��का�इ�तहास�शु��होता
है�, �जसम��संसद�म��कानून�क���सफा�रश�क��गई�थी�ता�क�भ�व�य�म��पंचायत��से��नवा��चत��कसी�भी�पद�को�धारण�करने�वाले�दो�से
अ�धक�ब�च��वाले����य��को�संसद��ारा�रोक�लगायी�जा�सके।�
• रा�ीय�जनसं�या�नी�त�2000 भी�दो-ब�चे�के�मानक�के�आवेदन�क��प�रक�पना�नह��करती�है�, ले�कन�अ�धकांश�रा�य��ने�एनपीट��2
000 से�पहले�भी�दो-ब�चे�के�मानक�को�अपनाया�है।�
• �जन�भारतीय�रा�य��ने�दो-ब�चे�के�मानदं ड�को�अपनाया�है�, उनम��राज�थान�, म�य��दे श�, �हमाचल��दे श�, ह�रयाणा�, आं���दे श�,
छ�ीसगढ़�और�उड़ीसा�शा�मल�ह�।�राज�थान�म��पहली�बार�पंचायत��और�नगर�पा�लका��म��दो-ब�चे�के�आदश��को�लागू��कया�गया�था।
• असम�मं��मंडल�ने�फैसला��कया�है��क�2021 से�दो�से�अ�धक�ब�च��वाले�लोग�सरकारी�नौकरी�के��लए�अयो�य�ह�गे।�

IASbaba
Web: http://ilp.iasbaba.com/ Score:
Email: ilp@iasbaba.com 0.00 / 200
Page 105
AIPTS/ILP VETERANS-
Exam Title :
2020 TE...
Email : yadavanurag075@gmail.com
Contact : 8882839768

73 व��संवैधा�नक�संशोधन�अ�ध�नयम�म���न�न�ल�खत��ावधान��के��लए�भी���ताव��दया�गया�है:

• म�हला��के��लए�सीट��क��कुल�सं�या�का�आर�ण�एक��तहाई�से�कम�नह��होना�चा�हए�(एससी�और�एसट��से�संबं�धत�म�हला��के
�लए�आर��त�सीट��क��सं�या�स�हत)।�
• इसके�अलावा�, ��येक��तर�पर�पंचायत��म��अ�य���के��लए�भी�कुल�सं�या�के�एक��तहाई�से�कम�म�हला��के��लए�आर��त�नह��होनी
चा�हए।�
• हर�पांच�साल�के�बाद��कसी�रा�य�का�रा�यपाल�पंचायत��क���व�ीय���थ�त�क��समी�ा�के��लए�एक��व��आयोग�का�गठन�करता�है।�
• अ�ध�नयम�हर��तर�पर�पंचायत�को�पांच�साल�के�काय�काल�के��लए��दान�करता�है।�हालां�क�, यह�अपने�काय�काल�के�पूरा�होने�से�पहले
ही�भंग�हो�सकती�है।�इसके�अलावा�, पंचायत�ग�ठत�करने�के��लए�नए�चुनाव�पांच�साल�क��अव�ध�पूरी�होने�से�पहले�( a) पूरे��कए
जाएंगे�; या�( b) �वघटन�के�मामले�म��, इसके��वघटन�क��तारीख�से�छह�महीने�क��अव�ध�समा�त�होने�से�पहले।�

QUESTION 6.
�न�न�ल�खत�म��से�कौन�सी�स�म�त�पंचायती�राज�क���थापना�से�संबं�धत�है�?

1. थुंगन�स�म�त�
2. अशोक�मेहता�स�म�त�
3. गाड�गल�स�म�त�
4. बलवंत�राय�मेहता�स�म�त�

नीचे��दए�गए�कूट�का�उपयोग�करके�सही�उ�र�चुन�:

a) केवल�1 और�2
b) केवल�2 और�3
c) केवल�1, 2, और�3
d) 1, 2, 3 और�4
Correct Answer: D
Your Answer:
Explanation

Solution (d)

Basic Information:

सामुदा�यक��वकास�काय��म�( 1952) और�रा�ीय��व�तार�सेवा�( 1953) जो�पंचायती�राज��व�था�के�पूव�वत��थे।�जनवरी�1957 म��, भा


रत�सरकार�ने�सामुदा�यक��वकास�काय��म�( 1952) और�रा�ीय��व�तार�सेवा�( 1953) के�कामकाज�क��जाँच�करने�और�उनके�बेहतर�काम
करने�के�उपाय��के�बारे�म��सुझाव�दे ने�के��लए�एक�स�म�त��नयु��क��थी।�इस�स�म�त�के�अ�य��बलवंत�राय�जी�मेहता�थे।�

बलवंत�राय�मेहता�स�म�त�( 1957) �ारा�क��गई�कुछ��मुख��सफा�रश�:

• तीन��तरीय�पंचायती�राज��णाली�क���थापना�- �ाम��तर�पर��ाम�पंचायत�, �लॉक��तर�पर�पंचायत�स�म�त�और��जला��तर�पर��जला


प�रषद।�
• �ाम�पंचायत�का�गठन�सीधे�चुने��ए���त�न�धय��के�साथ�होना�चा�हए�, जब�क�पंचायत�स�म�त�और��जला�प�रषद�का�गठन�अ��य�
�प�से�चुने�गए�सद�य��के�साथ��कया�जाना�चा�हए।�

अशोक�मेहता�स�म�त�( 1977) �ारा�क��गई�कुछ��मुख��सफा�रश�:

• पंचायती�राज�क����-�तरीय��णाली�को�दो-�तरीय��णाली�, यानी��जला��तर�पर��जला�प�रषद��ारा���त�था�पत��कया�जाना�चा�हए�,
और�इसके�नीचे�, मंडल�पंचायत��जसम��15,000 से�20,000 क��कुल�आबाद��वाले�गांव��का�समूह�शा�मल�थे।�
• रा�य��तर�के�नीचे�लोक��य�पय�वे�ण�के�तहत��वक���करण�के��लए�एक��जला��थम��ब���होना�चा�हए।�

IASbaba
Web: http://ilp.iasbaba.com/ Score:
Email: ilp@iasbaba.com 0.00 / 200
Page 106
AIPTS/ILP VETERANS-
Exam Title :
2020 TE...
Email : yadavanurag075@gmail.com
Contact : 8882839768

थुंगन�स�म�त�( 1988) �ारा�क��गई�कुछ��मुख��सफा�रश�:

• पंचायती�राज��नकाय��को�संवैधा�नक��प�से�मा�यता�द��जानी�चा�हए।�
• �ाम�, �लॉक�और��जला��तर�पर�पंचायत��के�साथ�पंचायती�राज�क�����तरीय��णाली।�
• पंचायती�राज��नकाय��का�काय�काल�पांच�वष��का�होना�चा�हए।�

गाड�गल�स�म�त�( 1988) �ारा�क��गई�कुछ��मुख��सफा�रश�:

• पंचायती�राज�सं�था��को�एक�संवैधा�नक�दजा���दया�जाना�चा�हए।�
• �ाम�, �लॉक�और��जला��तर�पर�पंचायत��के�साथ�पंचायती�राज�क�����तरीय��णाली।�
• एससी�, एसट��और�म�हला��के��लए�आर�ण।�

पंचायती�राज�पर�अ�य��मुख�स�म�तयाँ:

• जी�वी�के�राव�स�म�त�( 1985), एल�एम��स�घवी�स�म�त�( 1986) ।

Elimination:

य�द�आप�जानते�ह��(और�आपको�यह�जानना�चा�हए) �क�बलवंत�राय�स�म�त�(कथन�4) पंचायती�राज�पर�सबसे��मुख�स�म�तय��म��से�एक�है�,


केवल��वक�प�( d) म��कथन�4 है।�तो�, �वक�प�( d) हमारा�उ�र�होगा।�

कथन��व�ेषण:

कथन�1 कथन�2 कथन�3 कथन�4

स�य� स�य� स�य� स�य�

1988 म��“ सबसे�अ�छा 1957 म��ग�ठत�बलवंत�राय


1977 म��ग�ठत�अशोक�मेहता
पंचायती�राज��व�था�क� पंचायती�राज�सं�थान�कैसे मेहता�स�म�त�ने�‘ लोकतां��क
स�म�त�ने�दे श�म��घटती�पंचायती
मजबूती�के��लए�1988 म� �भावी�बनया�जा�सकता�है�” �वक���करण�’ क��योजना�क�
राज��व�था�को�पुनज��वत�करने
थुंगन�स�म�त�का�गठन��कया के����पर��वचार�करने�के��लए �थापना�क���सफा�रश�क��, �जसे
और�मजबूत�करने�क���सफा�रश�
गया�था।� गाड�गल�स�म�त�का�गठन अंततः�पंचायती�राज�के��प�म�
क��थी।�
�कया�गया�था।� जाना�जाता�है।�

QUESTION 7.
�न�न�ल�खत�म��से�कौन�सा�कथन�गलत�है�?

a) 73 व��संवैधा�नक�संशोधन�अ�ध�नयम�, 1992 को�पी. वी. नर�स�हा�राव�के��धानमं���व�काल�म��अ�ध�नय�मत��कया�गया�था।�


b) पंचायत��का�गठन�रा�य�सरकार�क��इ�छा�पर��नभ�र�करता�है।�
c) पंचायती�राज�सं�थान�सं�वधान�के��यायो�चत��ह�से�के�दायरे�म��ह�।�
d) इनम��से�कोई�भी�नह�।�
Correct Answer: B
Your Answer:
Explanation

IASbaba
Web: http://ilp.iasbaba.com/ Score:
Email: ilp@iasbaba.com 0.00 / 200
Page 107
AIPTS/ILP VETERANS-
Exam Title :
2020 TE...
Email : yadavanurag075@gmail.com
Contact : 8882839768

Solution (b)

Basic Information:

• पी.वी. नर�स�हा�राव�के��धानमं���व�काल�म��कां�ेस�सरकार�ने��ववादा�पद�पहलु��को�हटाने�के���ताव��म��भारी�बदलाव��कया�तथा
�सतंबर�, 1991 म��लोकसभा�म��एक�संवैधा�नक�संशोधन��वधेयक�पेश��कया।�यह��वधेयक�अंततः�73 व��संवैधा�नक�संशोधन
अ�ध�नयम�, 1992 के��प�म��उभरा�और�24 अ�ैल�, 1993 को��भावी��आ।�
• इस�अ�ध�नयम�ने�भारत�के�सं�वधान�म��एक�नया�भाग- IX और�एक�नई��यारहव��अनुसूची�जोड़ी।�
• अ�ध�नयम�पंचायती�राज�सं�था��को�एक�संवैधा�नक�दजा��दे ता�है।�रा�य�सरकार��अ�ध�नयम�के��ावधान��के�अनुसार�नई�पंचायती�राज
�णाली�को�अपनाने�के��लए�संवैधा�नक�दा�य�व�के�अंतग�त�ह�।�इस�लए�, न�तो�पंचायत��का�गठन�और�न�ही��नय�मत�अंतराल�पर
चुनाव��का�आयोजन�रा�य�सरकार�क��इ�छा�पर��नभ�र�करता�है।�
• रा�य��के��ववेक�पर�अ�ध�नयम�के��वै��छक��ावधान��को�शा�मल��कया�जाना�था।�

Elimination:

य�द�आप�जानते�ह��(और�आपको�अव�य�जानना�चा�हए) �क�73 व��संवैधा�नक�संशोधन�अ�ध�नयम�, 1992 को�मह�वपूण��मामल��म��रा�य�


के��ववेक�को�हटाने�और�पंचायती�राज��व�था�म��एक�पता�लाने�के��लए�बनाया�गया�था।�इस�लए�पंचायत��के�गठन�का�मह�वपूण��काय��रा�य
सरकार��क��इ�छा�पर�नह��छोड़ा�गया�था।�इस�लए�, हमारा�उ�र��वक�प�( b) है।�

QUESTION 8.
�न�न�ल�खत�कथन��पर��वचार�कर�:

1. �ाम�, म�यवत��और��जला��तर�पर�पंचायत��के�सभी�सद�य�सीधे�लोग���ारा�चुने�जाएंगे।�
2. �ाम�, म�यवत��और��ज़ला��तर�पर�एक�पंचायत�के�अ�य��का�चुनाव�इस�तरह�से��कया�जाएगा�जैसा�रा�य��वधानमंडल��नधा��रत
करता�है।�

ऊपर��दए�गए�कथन��म��से�कौन�सा�सही�है�/ ह��?

a) केवल�1
b) केवल�2
c) दोन��1 और�2
d) न�तो�1 और�न�ही�2
Correct Answer: A
Your Answer:
Explanation

Solution (a)

Basic Information:

• �ाम�, म�यवत��और��जला��तर�पर�पंचायत��के�सभी�सद�य�सीधे�लोग���ारा�चुने�जाएंगे।�
• रा�य��वधा�यका�, पंचायत�के�अ�य���के�आर�ण�के��लए�गाँव�या��कसी�अ�य��तर�पर�अनुसू�चत�जा�त�और�अनुसू�चत�जनजा�त�के
�लए��ावधान��दान�करेगी।�
• ��येक��तर�पर�पंचायत��म��अ�य���क��कुल�सं�या�के�एक��तहाई�से�कम�म�हला��के��लए�आर��त�नह��होगी।�
• यह�अ�ध�नयम��कसी�रा�य�क���वधा�यका�को��पछड़े�वग��के�प��म���कसी�भी��तर�पर�पंचायत�म���कसी�भी�पंचायत�या�चेयरपस�न�के
काया�लय��म��सीट��के�आर�ण�का��ावधान�करने�का�अ�धकार�दे ता�है।�

कथन��व�ेषण:

IASbaba
Web: http://ilp.iasbaba.com/ Score:
Email: ilp@iasbaba.com 0.00 / 200
Page 108
AIPTS/ILP VETERANS-
Exam Title :
2020 TE...
Email : yadavanurag075@gmail.com
Contact : 8882839768

कथन�1 कथन�2

स�य� अस�य�

�ाम��तर�पर�एक�पंचायत�के�अ�य��को�इस�तरह�से�चुना�जाएगा
जैसा��क�रा�य��वधानमंडल��नधा��रत�करता�है।�म�यवत��और
�ाम�, म�यवत��और��जला��तर�पर�पंचायत��क��सभी�सीट��पर
�जला��तर�पर�पंचायत��के�अ�य��का��नवा�चन�अ��य���प�से
��य��चुनाव�अ�नवाय���ावधान�ह�।�
- चुने�गए�सद�य��म��से�और�अ�ध�नयम�के�अनुसार��कया
जाएगा।�

QUESTION 9.
73 व��संवैधा�नक�संशोधन�अ�ध�नयम�, 1992 के�अनुसार�सीट��के�आर�ण�के�संदभ��म���न�न�ल�खत�कथन��पर��वचार�कर�:

1. ��येक�पंचायत�म��अनुसू�चत�जा�त�और�अनुसू�चत�जनजा�त�के��लए�सीट��का�आर�ण�पंचायत��े��म��उनक��जनसँ�या�के�अनुपात�म�
है।�
2. अ�ध�नयम�म��म�हला��के��लए�सीट��के�आर�ण�का��ावधान�है�, जो�कुल�सं�या�के�आधे�से�कम�नह��होना�चा�हए।�

ऊपर��दए�गए�कथन��म��से�कौन�सा�सही�है�/ ह��?

a) केवल�1
b) केवल�2
c) दोन��1 और�2
d) न�तो�1 और�न�ही�2
Correct Answer: A
Your Answer:
Explanation

Solution (a)

Basic Information:

• अ�ध�नयम�म��पंचायत��े��म��कुल�आबाद��के�अनुपात�म����येक�पंचायत�(यानी�, सभी�तीन��तर��पर) म��अनुसू�चत�जा�त�और


अनुसू�चत�जनजा�त�के��लए�सीट��के�आर�ण�का��ावधान�है।�
• रा�य��वधा�यका�एससी�और�एसट��के��लए��ाम�पंचायत�म��चेयरपस�न�के�काया�लय��या��कसी�अ�य��तर�के��लए�आर�ण�का��ावधान
करती�है।�
• ��येक��तर�पर�पंचायत��म��अ�य���क��कुल�सं�या�के�एक��तहाई�से�कम�म�हला��के��लए�आर��त�नह��होगी।�
• यह�अ�ध�नयम��कसी�रा�य�क���वधा�यका�को��पछड़े�वग��के�प��म���कसी�भी��तर�पर�पंचायत�म���कसी�भी�पंचायत�या�चेयरपस�न�के
काया�लय��म��सीट��के�आर�ण�का��ावधान�करने�का�अ�धकार�दे ता�है।�

कथन��व�ेषण:

कथन�1 कथन�2

IASbaba
Web: http://ilp.iasbaba.com/ Score:
Email: ilp@iasbaba.com 0.00 / 200
Page 109
AIPTS/ILP VETERANS-
Exam Title :
2020 TE...
Email : yadavanurag075@gmail.com
Contact : 8882839768

स�य� अस�य�

अ�ध�नयम�म��म�हला��के��लए�सीट��क��कुल�सं�या�के�एक
एससी�और�एसट��के��लए�आर�ण�पंचायत��े��म��कुल �तहाई�से�कम�नह��(एससी�और�एसट��से�संबं�धत�म�हला��के
जनसं�या�के�अनुपात�पर�आधा�रत�ह�।� �लए�आर��त�सीट��क��सं�या�स�हत) के��लए�आर�ण�का
�ावधान�है।�

QUESTION 10.
रा�य�चुनाव�आयोग�के�संदभ��म���न�न�ल�खत�कथन��पर��वचार�कर�:

1. रा�य��नवा�चन�आयु��को�भारत�के�मु�य�चुनाव�आयु��के�परामश��से�रा�यपाल��ारा��नयु���कया�जाता�है।�
2. रा�य��नवा�चन�आयु��के�काया�लय�क��सेवा�और�काय�काल�क��शत��रा�य��वधा�यका��ारा��नधा��रत�क��जाती�ह�।�

उपरो��कथन��म��से�कौन�सा�सही�है�/ ह��?

a) केवल�1
b) केवल�2
c) दोन��1 और�2
d) न�तो�1 और�न�ही�2
Correct Answer: D
Your Answer:
Explanation

Solution (d)

Basic Information:

• रा�य��नवा�चन�आयोग�मतदाता�सूची�तैयार�करने�, पंचायत��के�सभी�चुनाव��के�संचालन�, �नद� शन�और��नयं�ण�के��लए�उ�रदायी�है।�


• इसम��रा�यपाल��ारा��नयु���कए�जाने�वाले�रा�य�चुनाव�आयु��होते�ह�।�
• उसे�रा�य�के�उ�च��यायालय�के��यायाधीश�को�हटाने�के��लए��नधा��रत�तरीके�से�और�आधार�को�छोड़कर�काया�लय�से�नह��हटाया
जाएगा।�
• रा�य��वधा�यका�पंचायत��के�चुनाव�से�संबं�धत�सभी�मामल��के�संबंध�म���ावधान�कर�सकती�है।�

कथन��व�ेषण:

कथन�1 कथन�2

अस�य� अस�य�

IASbaba
Web: http://ilp.iasbaba.com/ Score:
Email: ilp@iasbaba.com 0.00 / 200
Page 110
AIPTS/ILP VETERANS-
Exam Title :
2020 TE...
Email : yadavanurag075@gmail.com
Contact : 8882839768

रा�य�चुनाव�आयु��को�रा�यपाल��ारा��नयु���कया�जाता�है
रा�य��नवा�चन�आयु��के�काया�लय�क��सेवा�और�काय�काल�क�
तथा�भारत�के�मु�य�चुनाव�आयु��के�साथ�परामश��का�कोई
शत��रा�यपाल��ारा��नधा��रत�क��जाती�ह�।�
�ावधान�नह��है।�

QUESTION 11.
पंचायती�राज�के��न�न�ल�खत��ावधान��पर��वचार�कर�:

1. �कसी�भी��तर�पर�पंचायत��म���पछड़े�वग��के��लए�सीट��(दोन��सद�य��और�अ�य��) का�आर�ण��दान�करना।�
2. आ�थ�क��वकास�और�सामा�जक��याय�क��योजना�तैयार�करने�के��लए�पंचायत��को�श��य��और�उ�रदा�य�व��का�ह�तांतरण�
3. पंचायत��को��व�ीय�श��यां��दान�करना�, अथा�त्�उ�ह��उ�चत�कर�, शु�क�और�टोल�, फ�स�को�आरो�पत�और�एक��त�करने�के��लए
अ�धकृत�करना।�

उपरो���ावधान��म��से�कौन�सा�/ से��वै��छक��ावधान�( voluntary provisions) ह��/ ह��?

a) केवल�1
b) केवल�1 और�2
c) केवल�2 और�3
d) 1, 2 और�3
Correct Answer: D
Your Answer:
Explanation

Solution (d)

Basic Information:

• 73 व��संवैधा�नक�संशोधन�अ�ध�नयम�, 1992 के��ावधान��को�दो��े�णय�- अ�नवाय��और��वै��छक�म��वग�कृत��कया�जा�सकता�है।�


• अ�ध�नयम�के�अ�नवाय��(अ�नवाय��या�आव�यक) �ावधान��को�नई�पंचायती�राज��व�था�बनाने�वाले�रा�य�कानून��म��शा�मल��कया
जाना�है।�
• �वै��छक��ावधान��को�रा�य��के��ववेक�पर�शा�मल��कया�जा�सकता�है।�इस��कार�अ�ध�नयम�के��वै��छक��ावधान�रा�य��के�अ�धकार
को�भौगो�लक�, राजनी�तक-�शास�नक�और�अ�य�जैसे��थानीय�कारक��को��यान�म��रखते��ए�नई�पंचायती�राज��णाली�को�अपनाते
�ए�सु�न��त�करते�ह�।�

�दए�गए�कथन��के�अलावा�अ�य��वै��छक��ावधान:

• संसद�के�(दोन��सदन�) और�रा�य��वधा�यका�(दोन��सदन�) के�सद�य��को�पंचायत��म��उनके��नवा�चन��े���के�अंतग�त�आने�वाले


�व�भ���तर��पर���त�न�ध�व�दे ना।�
• पंचायत��को�श��यां�और�अ�धकार��दान�करना�, उ�ह���व-शासन�क��सं�था��के��प�म��काय��करने�म��स�म�बनाना�(सं�ेप�म��, उ�ह�
�वाय���नकाय�बनाना)।�
• आ�थ�क��वकास�और�सामा�जक��याय�क��योजना�तैयार�करने�के��लए�तथा�सं�वधान�क���यारहव��अनुसूची�म��सूचीब��29 काय��म��से
कुछ�या�सभी�को�पूरा�करने�के��लए�पंचायत��को�श��य��और��ज�मेदा�रय��का�ह�तांतरण�;

QUESTION 12.
पंचायत�(अनुसू�चत��े���के��लए��व�तार) अ�ध�नयम�( PESA), 1996 क���वशेषता��के�संदभ��म���न�न�ल�खत�कथन��पर��वचार�कर�।�

1. अनुसू�चत��े���म��पंचायत��पर�एक�रा�य�कानून��थागत�कानून�, सामा�जक�और�धा�म�क��था��के�अनु�प�होगा।�
2. अनुसू�चत��े���म��लघु�जल��नकाय��क��योजना�और��बंधन�को�उ�चत��तर�पर�पंचायत��को�स�पा�जाना�चा�हए।�
3. यह�सु�न��त�करने�के��लए��क�उ�च��तर�क��पंचायत��, �नचले��तर�या��ाम�सभा�क��पंचायत�क��श��य��और�अ�धकार��को�अ�ध��हत
कर�।�

IASbaba
Web: http://ilp.iasbaba.com/ Score:
Email: ilp@iasbaba.com 0.00 / 200
Page 111
AIPTS/ILP VETERANS-
Exam Title :
2020 TE...
Email : yadavanurag075@gmail.com
Contact : 8882839768

�न�न�ल�खत�म��से�कौन�सी�अ�ध�नयम�क���वशेषताएं�ह��/ ह��?

a) केवल�1
b) केवल�1 और�2
c) केवल�1 और�3
d) 1, 2 और�3
Correct Answer: B
Your Answer:
Explanation

Solution (b)

Basic Information:

पंचायत��से�संबं�धत�सं�वधान�के�भाग�IX के��ावधान�पांचव��अनुसूची�के��े���पर�लागू�नह��ह�।�संसद�ने�" पंचायत��(अनुसू�चत��े���के��लए


�व�तार) अ�ध�नयम�", 1996 को�लागू��कया�है�, �जसे�लोक��य��प�से�पेसा�अ�ध�नयम�या��व�तार�अ�ध�नयम�के��प�म��जाना�जाता�है�ता�क
सं�वधान�के�भाग�IX के��ावधान��को�अनुसू�चत��े���म��पंचायत��से�संबं�धत��कया�जा�सके।�

अ�ध�नयम�क��कुछ��वशेषताएं:

• अनुसू�चत��े���म��पंचायत��पर�एक�रा�य�कानून��थागत�कानून�, सामा�जक�और�धा�म�क��था��और�सामुदा�यक�संसाधन��के
पारंप�रक��बंधन��था��के�अनु�प�होगा।�
• एक�गाँव�म��आमतौर�पर�एक��नवास��थान�या�ब��तय��का�एक�समूह�या�एक�समुदाय�या�एक�समुदाय�शा�मल�होता�है�तथा�परंपरा�
और�री�त-�रवाज��के�अनुसार�अपने�मामल��का��बंधन�करता�है।�
• ��येक�गाँव�म��एक��ाम�सभा�होगी��जसम���ाम��तर�पर�पंचायत�के��लए�मतदाता�सूची�म��नाम�शा�मल�ह�।�
• ��येक��ाम�सभा�लोग��क��परंपरा��और�री�त-�रवाज��, उनक��सां�कृ�तक�पहचान�, सामुदा�यक�संसाधन��और��ववाद�समाधान�के
�थागत�मोड�को�सुर��त�रखने�और�संर��त�करने�के��लए�स�म�होगी।�
• �ाम��तर�पर���येक�पंचायत�को��ाम�सभा�से�उपरो��योजना��, काय��म��और�प�रयोजना��के��लए�धन�के�उपयोग�का��माण�प�
�ा�त�करना�आव�यक�होगा।�
• रा�य��वधानमंडल��म��यह�सु�न��त�करने�के��लए�सुर�ा�उपाय�ह�गे��क�उ�च��तर�क��पंचायत��, �कसी�भी��नचले��तर�या��ाम�सभा
पंचायत�क��श��य��और�अ�धकार��को�अ�ध��हत�न�कर�।�
• रा�य�सरकार�ऐसे�अनुसू�चत�जनजा�तय��को�ना�मत�कर�सकती�है��जनका�पंचायत�म��म�यवत���तर�पर�या��जला��तर�पर�पंचायत�म�
कोई���त�न�ध�व�नह��है।�ले�कन�ऐसा�नामांकन�उस�पंचायत�म��चुने�जाने�वाले�कुल�सद�य��के�दसव���ह�से�से�अ�धक�नह��होगा।�

कथन��व�ेषण:

कथन�1 कथन�2 कथन�3

स�य� स�य� अस�य�

अ�ध�नयम�यह�सु�न��त�करने�के��लए
पंचायत���ारा�लघु�जल��नकाय��क��योजना सुर�ा�उपाय�करता�है��क�उ�च��तर�क�
रा�य�का�कानून�आ�दवासी�समुदाय��क�
और��बंधन�भागीदारी��शासन�के�साथ��ाम पंचायत��, �नचले��तर�या��ाम�सभा�क�
पारंप�रक��था��के�अनु�प�होना�चा�हए�
�शासन�को�स�म�बनाता�है।� �कसी�पंचायत�क��श��य��और
अ�धकार��को�अ�ध��हत�न�कर�।�

IASbaba
Web: http://ilp.iasbaba.com/ Score:
Email: ilp@iasbaba.com 0.00 / 200
Page 112
AIPTS/ILP VETERANS-
Exam Title :
2020 TE...
Email : yadavanurag075@gmail.com
Contact : 8882839768

QUESTION 13.
पंचायत�(अनुसू�चत��े���के��लए��व�तार) अ�ध�नयम�, 1996 के�तहत�आने�वाले��े���म��, �ाम�सभा�क��भू�मका�/ श����या�है�?

1. �ाम�सभा�के�पास�अनुसू�चत��े���म��भू�म�के�अलगाव�को�रोकने�क��श���है।�
2. गरीबी�उ�मूलन�और�अ�य�काय��म��के�तहत�लाभा�थ�य��क��पहचान�या�चयन।�
3. अनुसू�चत��े���म���कसी�भी�ख�नज�के��लए�पूव��ण�लाइस�स�या�खनन�प�े �दे ने�के��लए��ाम�सभा�क���सफा�रश�आव�यक�है।�

ऊपर��दए�गए�कथन��म��से�कौन�सा�सही�है�/ ह��?

a) केवल�1 और�2
b) केवल�1 और�3
c) केवल�2 और�3
d) 1, 2 और�3
Correct Answer: A
Your Answer:
Explanation

Solution (a)

Basic Information:

पंचायत��(अनुसू�चत��े���के��लए��व�तार) अ�ध�नयम�, 1996 �न�न�ल�खत��ावधान���ारा�अनुसू�चत��े���पर�सभी��ाम�सभा


को�अ�धकार�दे ता�है:

• यह���येक�गाँव�म��एक��ाम�सभा�आयो�जत�करने�के��लए�बा�य�करता�है�, �जसम���ाम��तर�पर�पंचायत�के��लए�मतदाता�सूची�म�
शा�मल�सभी�����स��म�लत�ह�गे।�
• अ�ध�नयम�म��स�म��ाम�सभा�को�लोग��क��परंपरा��और�री�त-�रवाज��, उनक��सां�कृ�तक�पहचान�, सामुदा�यक�संसाधन��और
�ववाद�समाधान�के��थागत�मोड�क��सुर�ा�और�संर�ण�का���ताव�है।�
• ��येक��ाम�सभा�को�सामा�जक�और�आ�थ�क��वकास�के��लए�योजना��, काय��म��और�प�रयोजना��को�मंजूरी�दे नी�होती�है�, इससे
पहले��क�इस�तरह�क��योजना��, काय��म��और�प�रयोजना��को�पंचायत��ारा��ामीण��तर�पर�लागू��कया�जाए।�
• ��येक��ाम�सभा�गरीबी�उ�मूलन�और�अ�य�काय��म��के�तहत�लाभा�थ�य��के��प�म�����य��क��पहचान�या�चयन�के��लए�उ�रदायी
है।�
• �ाम��तर�पर���येक�पंचायत�को��ाम�सभा�से�योजना��, काय��म��और�प�रयोजना��के��लए�उस�पंचायत��ारा�धन�के�उपयोग�का
�माण�प���ा�त�करना�आव�यक�है।�
• �वकास�प�रयोजना��के��लए�अनुसू�चत��े���म��भू�म�के�अ�ध�हण�से�पहले�और�अनुसू�चत��े���म��ऐसी�प�रयोजना��से��भा�वत
���य��को��फर�से�बसाने�या�पुनवा�स�करने�से�पहले�उ�चत��तर�पर��ाम�सभा�या�पंचायत��से�परामश���कया�जाएगा।�
• अनुसू�चत��े���म��लघु�जल��नकाय��क��योजना�और��बंधन�पंचायत��को�उ�चत��तर�पर�स�पा�गया�है।�
• अनुसू�चत��े���म��लघु�ख�नज��के��लए�पूव��ण�लाइस�स�या�खनन�प�े �दे ने�से�पहले��ाम�सभा�या�पंचायत��क��उपयु���तर�पर
�सफा�रश��अ�नवाय��ह�।�

Elimination:

य�द�आप�' कथन�3' को�दे खते�ह��, तो�इसम��' �कसी�भी�ख�नज�' का�उ�लेख��कया�गया�है�, जो����के�संदभ��म��चरम�और�ता�क�क�नह��है।�तो


, कथन�3 वाले�सभी��वक�प��को�समा�त��कया�जा�सकता�है।�इस�लए�, हम��हमारे�उ�र�के��प�म���वक�प�( a) के�साथ�छोड़��दया�गया�है।�

कथन��व�ेषण:

कथन�1 कथन�2 कथन�3

IASbaba
Web: http://ilp.iasbaba.com/ Score:
Email: ilp@iasbaba.com 0.00 / 200
Page 113
AIPTS/ILP VETERANS-
Exam Title :
2020 TE...
Email : yadavanurag075@gmail.com
Contact : 8882839768

स�य� स�य� अस�य�

अनुसू�चत��े���म��लघु�ख�नज
��येक��ाम�सभा�गरीबी�उ�मूलन�और�अ�य
अनुसू�चत��े���म��भू�म�का�अ�ध�हण के��लए�पूव��ण�लाइस�स�या
काय��म��के�तहत�लाभा�थ�य��के��प�म�
करने�से�पहले�उ�चत��तर�पर��ाम�सभा�या खनन�प�े �दे ने�के��लए��ाम
���य��क��पहचान�या�चयन�के��लए
पंचायत��से�परामश���कया�जाएगा� सभा�क���सफा�रश�आव�यक
�ज�मेदार�होगी।�
है।�

QUESTION 14.
�न�न�ल�खत�म��से�कौन�73 व��संवैधा�नक�संशोधन�अ�ध�नयम�( 1992) का�अ�नवाय���ावधान�नह��है:

a) �कसी�गाँव�या�गाँव��के�समूह�म���ाम�सभा�का�संगठन।�
b) तीन���तर��पर�पंचायत��म��अनुसू�चत�जा�तय��और�अनुसू�चत�जनजा�तय��के��लए�सीट��(दोन��सद�य��और�अ�य��) का�आर�ण।�
c) गाँव�, म�यवत��और��जला��तर��पर�पंचायत��क��सभी�सीट��पर���य��चुनाव।�
d) इनम��से�कोई�भी�नह�।�
Correct Answer: D
Your Answer:
Explanation

Solution (d)

Basic Information:

• 73 व��संवैधा�नक�संशोधन�अ�ध�नयम�, 1992 के��ावधान��को�दो��े�णय�- अ�नवाय��और��वै��छक�म��वग�कृत��कया�जा�सकता�है।�


• अ�ध�नयम�के�अ�नवाय��(अ�नवाय��या�आव�यक) �ावधान��को�नई�पंचायती�राज��व�था�बनाने�वाले�रा�य�कानून��म��शा�मल��कया
जाना�है।�

73 व��संवैधा�नक�संशोधन�अ�ध�नयम�( 1992) के�अ�य�अ�नवाय���ावधान:

• �ाम�, म�यवत��और��जला��तर��पर�पंचायत��क���थापना।�
• म�यवत��और��जला��तर��पर�पंचायत��के�अ�य��के�पद�के��लए�अ��य��चुनाव।�
• पंचायत��के�चुनाव�लड़ने�के��लए��यूनतम�आयु�21 वष��है।�
• तीन���तर��पर�पंचायत��म��म�हला��के��लए�एक-�तहाई�सीट��(दोन��सद�य��और�अ�य��) का�आर�ण।�
• सभी��तर��पर�पंचायत��के��लए�पांच�साल�का�काय�काल�तय�करना�तथा��कसी�भी�पंचायत�के�भंग�होने�क����थ�त�म��छह�महीने�के
भीतर�नए�चुनाव�कराना।�
• पंचायत��के�चुनाव�कराने�के��लए�एक�रा�य�चुनाव�आयोग�क���थापना।�पंचायत��क���व�ीय���थ�त�क��समी�ा�के��लए�हर�पांच�साल
के�बाद�एक�रा�य��व��आयोग�का�गठन�करना।�

QUESTION 15.
पंचायत��(अनुसू�चत��े���के��लए��व�तार) अ�ध�नयम�, 1996 के�तहत��दान��कए�गए�अनुसू�चत�जनजा�तय��के��लए�आर�ण�के��ावधान�
के�संदभ��म���न�न�ल�खत�कथन��पर��वचार�कर�:

1. अनुसू�चत�जनजा�तय��के��लए�आर�ण�कुल�सीट��क��सं�या�के�एक��तहाई�से�कम�नह��होगा।�
2. सभी��तर��पर�पंचायत��के�अ�य���क��कुल�सीट��म��से�आधी�अनुसू�चत�जनजा�त�के��लए�आर��त�ह�।�

ऊपर��दए�गए�कथन��म��से�कौन�सा�गलत�है�/ ह��?

IASbaba
Web: http://ilp.iasbaba.com/ Score:
Email: ilp@iasbaba.com 0.00 / 200
Page 114
AIPTS/ILP VETERANS-
Exam Title :
2020 TE...
Email : yadavanurag075@gmail.com
Contact : 8882839768

a) केवल�1
b) केवल�2
c) दोन��1 और�2
d) न�तो�1 और�न�ही�2
Correct Answer: C
Your Answer:
Explanation

Solution (c)

Basic Information:

पंचायत��(अनुसू�चत��े���के��लए��व�तार) अ�ध�नयम�, 1996 के�तहत�अनुसू�चत�जनजा�तय��के��लए�आर�ण�के��ावधान:

• ��येक�पंचायत�म��अनुसू�चत��े���म��सीट��का�आर�ण�उन�समुदाय��क��जनसं�या�के�अनुपात�म��होगा��जनके��लए�आर�ण�सं�वधान
के�भाग�IX के�तहत��दया�जाना�है।�हालाँ�क�, अनुसू�चत�जनजा�तय��के��लए�आर�ण�कुल�सीट��क��सं�या�के�आधे�से�कम�नह��होगी।
• सभी��तर��पर�पंचायत��के�अ�य���क��सभी�सीट� �अनुसू�चत�जनजा�त�के��लए�आर��त�ह�गी।�
• रा�य�सरकार�ऐसे�अनुसू�चत�जनजा�तय��को�ना�मत�कर�सकती�है��जनका�पंचायत�म��म�यवत���तर�पर�या��जला��तर�पर�पंचायत�म�
कोई���त�न�ध�व�नह��है।�ले�कन�ऐसा�नामांकन�उस�पंचायत�म��चुने�जाने�वाले�कुल�सद�य��के�दसव���ह�से�से�अ�धक�नह��होगा।�

कथन��व�ेषण:

कथन�1 कथन�2

अस�य� अस�य�

अनुसू�चत�जनजा�तय��के��लए�आर�ण�कुल�सीट��क��सं�या�के सभी��तर��पर�पंचायत��के�अ�य���क��सभी�सीट� �अनुसू�चत


आधे�से�कम�नह��होगा।� जनजा�त�के��लए�आर��त�ह�गी।�

QUESTION 16.
�न�न�ल�खत�कथन��पर��वचार�कर�:

1. पया��त�ह�तांतरण�( adequate devolution) का�अभाव।�


2. राजकोषीय�श��य��का�उपयोग�करने�क��अ�न�छा।�
3. नौकरशाही��ारा�अ�य�धक��नयं�ण।�

ऊपर��दए�गए�कथन��म��से�कौन�सा�/ से�पंचायती�राज�सं�थान��के�अ�भावी��दश�न�के�सही�कारण�ह��?

a) केवल�1 और�2
b) केवल�2
c) केवल�2 और�3
d) 1, 2 और�3
Correct Answer: D
Your Answer:
Explanation

IASbaba
Web: http://ilp.iasbaba.com/ Score:
Email: ilp@iasbaba.com 0.00 / 200
Page 115
AIPTS/ILP VETERANS-
Exam Title :
2020 TE...
Email : yadavanurag075@gmail.com
Contact : 8882839768

Solution (d)

Basic Information:

73 व��संशोधन�अ�ध�नयम�(1992) के�मा�यम�से�संवैधा�नक�दजा��और�संर�ण��दान�करने�के�बाद�भी�, पंचायती�राज�सं�थान��( PRI) का


�दश�न�संतोषजनक�नह��रहा�है�और�अपे��त��तर�तक�नह��है।�इस�उप-इ�तम��दश�न�के��व�भ��कारण�इस��कार�ह�:

• पया��त�ह�तांतरण�का�अभाव�
• नौकरशाही��ारा�अ�य�धक��नयं�ण�
• �न�धय��क��बंधी��ई��कृ�त�
• सरकारी��व��पोषण�पर��नभ�रता�
• राजकोषीय�श��य��का�उपयोग�करने�क��अ�न�छा�
• �ाम�सभा�क����थ�त�
• समानांतर��नकाय��का��नमा�ण�
• कमज़ोर�अधोसंरचना�

कथन��व�ेषण:

कथन�1 कथन�2 कथन�3

स�य� स�य� स�य�

कई�रा�य��ने�पंचायती�राज�सं�था��को पंचायत�के��मुख��का�तक��है��क�अपने��वयं �ाम�पंचायत��को�अधीनता�क����थ�त�म�


पया��त�श��यां�और�अ�धकार�दे ने�के��लए के��नवा�चन��े��पर�कर�लगाना�मु��कल�है�, रखा�गया�है�तथा�उ�ह��ज�टल�नौकरशाही
पया��त�कदम�नह��उठाए�ह�।� खासकर�जब�आप�समुदाय�म��रहते�ह�।� बाधा��से�गुजरना�पड़ता�है।�

QUESTION 17.
74 व��सं�वधान�(संशोधन) अ�ध�नयम�, 1992 के�संदभ��म���न�न�ल�खत�कथन��पर��वचार�कर�:

1. इस�अ�ध�नयम��ारा�भारत�के�सं�वधान�म��भाग�IX A स��म�लत��कया�गया��जसम��नगर�पा�लका��के��लए��ावधान�ह�।�
2. अ�ध�नयम�म����येक�रा�य�हेत�ु दो��कार�क��नगरपा�लका��के�गठन�के��लए�, एक�नगर�प�रषद�और�एक�नगर��नगम�का��ावधान�है।�

ऊपर��दए�गए�कथन��म��से�कौन�सा�सही�है�/ ह��?

a) केवल�1
b) केवल�2
c) दोन��1 और�2
d) न�तो�1 और�न�ही�2
Correct Answer: A
Your Answer:
Explanation

Solution (a)

IASbaba
Web: http://ilp.iasbaba.com/ Score:
Email: ilp@iasbaba.com 0.00 / 200
Page 116
AIPTS/ILP VETERANS-
Exam Title :
2020 TE...
Email : yadavanurag075@gmail.com
Contact : 8882839768

Basic Information:

• 74 व��सं�वधान�(संशोधन) अ�ध�नयम�, 1992 ने�भारत�के�सं�वधान�म��एक�नया�भाग�IX-A जोड़ा�, �जसका�शीष�क�‘ द


�यु�न�सपै�लट�ज�’ है�तथा�इसम��अनु�छे द�243-P से�243-ZG तक�के��ावधान�शा�मल�ह�।�
• अ�ध�नयम�ने�सं�वधान�म��एक�नई�बारहव��अनुसूची�भी�जोड़ी�है।�इस�अनुसूची�म��नगरपा�लका��के�18 काया��मक��वषय�शा�मल�ह�।
यह�अनु�छे द�243-W से�संबं�धत�है।�
• रा�य�सरकार��अ�ध�नयम�के��ावधान��के�अनुसार�नगरपा�लका��क��नई��णाली�को�अपनाने�के��लए�संवैधा�नक�दा�य�व�के�अंतग�त
ह�।�
• अ�ध�नयम�म����येक�रा�य�म���न�न�ल�खत�तीन��कार�क��नगरपा�लका��के�गठन�का��ावधान�है।�

1. एक�सं�मणकालीन��े��के��लए�एक�नगर�पंचायत�(�जसे�कुछ�भी�नाम��दया�जाता�है) , अथा�त्�, एक��ामीण��े��से�शहरी��े��म��सं�मण


का��े�।�

2. एक�छोटे �शहरी��े��के��लए�एक�नगरपा�लका�प�रषद।�

3. एक�बड़े�शहरी��े��के��लए�एक�नगर��नगम�

कथन��व�ेषण:

कथन�1 कथन�2

स�य� अस�य�

74 व��सं�वधान�(संशोधन) अ�ध�नयम�, 1992 ने�भारत�के अ�ध�नयम�म��अनु�छे द�243Q के�तहत�3 �कार�क��नगर


सं�वधान�म��एक�नया�भाग�IX-A जोड़ा�और�नगरपा�लका��को पा�लका��क��प�रक�पना�क��गई�है।�एक�नगर�पंचायत�, एक
संवैधा�नक�दजा���दया।� नगरपा�लका�प�रषद�और�एक�नगर��नगम।�

QUESTION 18.
क���य��तर�पर�, ‘ शहरी��थानीय�सरकार�’ (urban local government) का��वषय��न�न�ल�खत�मं�ालय�/ मं�ालय��म��से��कससे
संब��है:

1. शहरी��वकास�मं�ालय�
2. र�ा�मं�ालय�
3. गृह�मं�ालय�

ऊपर��दए�गए�कथन��म��से�कौन�सा�सही�है�/ ह��?

a) केवल�1 और�2
b) केवल�1 और�3
c) केवल�2 और�3
d) 1, 2 और�3
Correct Answer: D
Your Answer:
Explanation

IASbaba
Web: http://ilp.iasbaba.com/ Score:
Email: ilp@iasbaba.com 0.00 / 200
Page 117
AIPTS/ILP VETERANS-
Exam Title :
2020 TE...
Email : yadavanurag075@gmail.com
Contact : 8882839768

Solution (d)

Basic Information:

शहरी�सरकार�क���णाली�का�गठन�74 व��संवैधा�नक�संशोधन�अ�ध�नयम�, 1992 के�मा�यम�से��कया�गया�था।�क���य��तर�पर�, ‘ शहरी


�थानीय�सरकार�’ के��वषय�को��न�न�ल�खत�तीन�मं�ालय���ारा��नपटाया�जाता�है:

1. शहरी��वकास�मं�ालय�, 1985 म��एक�अलग�मं�ालय�बनाया�गया�


2. छावनी�बोड��के�मामले�म��र�ा�मं�ालय।�
3. क���शा�सत��दे श��के�मामले�म��गृह�मं�ालय�

QUESTION 19.
�न�न�ल�खत�कथन��पर��वचार�कर�:

1. भारत�म��पहला�नगर��नगम�1687-88 म��बॉ�बे�म���था�पत��कया�गया�था।�
2. लॉड���रपन�को�भारत�म���थानीय-�वशासन�का��पता�कहा�जाता�है।�

सही�कूट�का�चयन�कर�:

a) केवल�1
b) केवल�2
c) दोन��1 और�2
d) न�तो�1 और�न�ही�2
Correct Answer: B
Your Answer:
Explanation

Solution (b)

Basic Information:

शहरी��थानीय�सरकार�क��सं�थाएँ����टश�शासन�के�काल�म��आधु�नक�भारत�म��उ�प�����और��वक�सत���।�इस�संदभ��म���मुख�घटनाएं�इस
�कार�ह�:

• �व�ीय��वक���करण�पर�लॉड��मेयो�के�1870 के�संक�प�ने��थानीय��व-शासन�सं�थान��के��वकास�क��क�पना�क�।�
• �वक���करण�पर�रॉयल�कमीशन�1907 म���नयु���कया�गया�था�तथा�इसने�1909 म��अपनी��रपोट� ���तुत�क�।�इसके�अ�य�
हॉबहाउस�थे।�
• 1919 के�भारत�सरकार�अ�ध�नयम��ारा��ांत��म��शु��क��गई���शासन�योजना�( dyarchical scheme ) के�तहत�, �थानीय
�वशासन�एक�उ�रदायी�भारतीय�मं�ी�के��भार�के�तहत�एक�ह�तांत�रत��वषय�बन�गया।�
• 1924 म��, छावनी�अ�ध�नयम�क���य��वधा�यका��ारा�पा�रत��कया�गया�था।�
• 1935 के�भारत�सरकार�अ�ध�नयम��ारा�शु��क��गई��ांतीय��वाय�ता�योजना�के�तहत�, �थानीय��वशासन�को�एक��ांतीय��वषय
घो�षत��कया�गया�था।�

कथन��व�ेषण:

कथन�1 कथन�2

IASbaba
Web: http://ilp.iasbaba.com/ Score:
Email: ilp@iasbaba.com 0.00 / 200
Page 118
AIPTS/ILP VETERANS-
Exam Title :
2020 TE...
Email : yadavanurag075@gmail.com
Contact : 8882839768

अस�य� स�य�

लॉड���रपन�के�1882 के�संक�प�को��थानीय��वशासन�के�' मै�ना


भारत�म��पहला�नगर��नगम�1686-87 म��म�ास�म��और�172
काटा��' के��प�म����त��त��कया�गया�है�, इस�लए�उ�ह��भारत�म�
6 म��बंबई�, कलक�ा�म���था�पत��कया�गया�था।�
�थानीय-�वशासन�का��पता�कहा�जाता�है।�

QUESTION 20.
74 व��सं�वधान�(संशोधन) अ�ध�नयम�, 1992 के��ावधान��के�संदभ��म���न�न�ल�खत�कथन��पर��वचार�कर�:

1. रा�य��वधा�यका�एक�नगरपा�लका�के�अ�य��के�चुनाव�का�तरीका��दान�कर�सकती�है।�
2. अ�ध�नयम�म��एससी�, एसट��और�म�हला��के��लए�नगरपा�लका��म��अ�य���के�काया�लय��के�आर�ण�का�तरीका�है।�

ऊपर��दए�गए�कथन��म��से�कौन�सा�सही�है�/ ह��?

a) केवल�1
b) केवल�2
c) दोन��1 और�2
d) न�तो�1 और�न�ही�2
Correct Answer: A
Your Answer:
Explanation

Solution (a)

Basic Information:

74 व��सं�वधान�(संशोधन) अ�ध�नयम�, 1992 के�कुछ��ावधान�जो�रा�य��वधा�यका��ारा��नधा��रत��कए�जा�सकते�ह�:

• रा�य��वधा�यका�एक�नगरपा�लका�के�अ�य��के�चुनाव�का�तरीका��दान�कर�सकती�है।�
• रा�य��वधा�यका�एक�वाड��स�म�त�क��संरचना�और��े�ीय��थान�के�संबंध�म���ावधान�कर�सकती�है�तथा��कस�तरह�से�एक�वाड��स�म�त
म��सीट� �भरी�जाएंगी।�यह�वाड��स�म�तय��के�अलावा�स�म�तय��के�गठन�के��लए�भी�कोई��ावधान�कर�सकती�है।�
• रा�य��वधा�यका�अनुसू�चत�जा�त�, अनुसू�चत�जनजा�त�और�म�हला��के��लए�नगरपा�लका��म��अ�य���के�काया�लय��के�आर�ण
के��लए�तरीके��दान�कर�सकती�है।�
• रा�य��वधा�यका��पछड़े�वग��के�प��म���कसी�भी�नगर�पा�लका�या�नगरपा�लका��के�अ�य���के�काया�लय��म��सीट��के�आर�ण�का
कोई��ावधान�कर�सकती�है।�

कथन��व�ेषण:

कथन�1 कथन�2

स�य� अस�य�

IASbaba
Web: http://ilp.iasbaba.com/ Score:
Email: ilp@iasbaba.com 0.00 / 200
Page 119
AIPTS/ILP VETERANS-
Exam Title :
2020 TE...
Email : yadavanurag075@gmail.com
Contact : 8882839768

रा�य��वधा�यका�अनुसू�चत�जा�त�, अनुसू�चत�जनजा�त�और
रा�य��वधा�यका�एक�नगरपा�लका�के�अ�य��के�चुनाव�के�तरीके
म�हला��के��लए�नगरपा�लका��म��अ�य���के�काया�लय��के
को��नधा��रत�कर�सकती�है।�
�लए�आर�ण�के�तरीके��दान�कर�सकती�है।�

QUESTION 21.
�जला�योजना�स�म�त�( District Planning Committee) के�संदभ��म���न�न�ल�खत�कथन��पर��वचार�कर�:

1. इसका�गठन��जले�म��पंचायत��और�नगरपा�लका���ारा�तैयार�क��गई�योजना��को�समे�कत�करने�के��लए��कया�गया�है।�
2. एक�स�म�त�के�सद�य��म��से�4/5 सद�य��को��जले�के�नगरपा�लका��के��नवा��चत�सद�य���ारा�अपने�बीच�से�चुना�जाना�चा�हए।�

ऊपर��दए�गए�कथन��म��से�कौन�सा�सही�है�/ ह��?

a) केवल�1
b) केवल�2
c) दोन��1 और�2
d) न�तो�1 और�न�ही�2
Correct Answer: A
Your Answer:
Explanation

Solution (a)

Basic Information:

• �जले�म��पंचायत��और�नगरपा�लका���ारा�तैयार�क��गई�योजना��को�समे�कत�करने�और�सम���प�से��जले�के��लए�एक�मसौदा
�वकास�योजना�तैयार�करने�के��लए�हर�रा�य�को��जला��तर�पर�एक��जला�योजना�स�म�त�का�गठन�करना�है।�
• यह�अ�ध�नयम�इस�बात�क��पु���करता�है��क��जला�योजना�स�म�त�के�सद�य��म��से�4/5 सद�य��जला�पंचायत�और�नगर�पा�लका�
के��नवा��चत�सद�य���ारा�आपस�म��चुने�जाते�ह�।�स�म�त�म��इन�सद�य��का���त�न�ध�व��जले�म���ामीण�और�शहरी�आबाद��के�अनुपात
के�अनुपात�म��होना�चा�हए।�
• ऐसी�स�म�त�का�अ�य��रा�य�सरकार�के�पास��वकास�योजना�को�आगे�बढ़ाएगा।�

कथन��व�ेषण:

कथन�1 कथन�2

स�य� अस�य�

स�म�त�पंचायत��और�नगरपा�लका���ारा�तैयार�योजना��को �जले�म��एक��जला�योजना�स�म�त�के�सद�य��के�4/5 सद�य


समे�कत�करती�है�और�सम���प�से��जले�के��लए�एक��वकास �जला�पंचायत�और�नगर�पा�लका��के��नवा��चत�सद�य���ारा
योजना�का�मसौदा�तैयार�करती�है� चुना�जाना�चा�हए।�

QUESTION 22.

IASbaba
Web: http://ilp.iasbaba.com/ Score:
Email: ilp@iasbaba.com 0.00 / 200
Page 120
AIPTS/ILP VETERANS-
Exam Title :
2020 TE...
Email : yadavanurag075@gmail.com
Contact : 8882839768

�न�न�ल�खत�कथन��पर��वचार�कर�:

1. मे�ोपॉ�लटन��े��का�अथ��एक��जले�म��10 लाख�या�उससे�अ�धक�क��आबाद��वाला��े��है।�
2. नगर�पा�लका��और�पंचायत��के��नवा��चत�सद�य�महानगर��े��म��एक�महानगरीय�योजना�स�म�त�के�2/3 सद�य��का�चुनाव�करते�ह�।
3. रा�य��वधा�यका�, मे�ोपॉ�लटन�योजना�स�म�त�( Metropolitan Planning Committees -MPC) के�सद�य��के�चुनाव�के
संघटन�और�तरीके�का��ावधान�कर�सकती�है।�

ऊपर��दए�गए�कथन��म��से�कौन�सा�सही�है�/ ह��?

a) केवल�1
b) केवल�1 और�2
c) केवल�3
d) 1, 2 और�3
Correct Answer: C
Your Answer:
Explanation

Solution (c)

Basic Information:

• 74 व��सं�वधान�(संशोधन) अ�ध�नयम�, 1992 के�अनुसार�, एक�मसौदा��वकास�योजना�तैयार�करने�के��लए���येक�महानगरीय��े�


( metropolitan area ) के��लए�एक�महानगरीय�योजना�स�म�त�का��ावधान�है।�
• महानगरीय��े��का�अथ��एक�ऐसे��े��म��10 लाख�या�उससे�अ�धक�क��आबाद��वाला��े��है�, �जसम��एक�या�एक�से�अ�धक��जले�ह�
और�दो�या�अ�धक�नगर�पा�लका��या�पंचायत��या�अ�य�स���हत��े���से��मलकर�बना��आ�है।�

रा�य��वधा�यका�महानगरीय�योजना�स�म�त�(एमपीसी) के�संबंध�म���ावधान�कर�सकती�है:

◦ ऐसी�स�म�तय��क��संरचना�;
◦ ऐसी�स�म�तय��के�सद�य��के�चुनाव�का�तरीका�;
◦ क���सरकार�, रा�य�सरकार�और�अ�य�संगठन��का�ऐसी�स�म�तय��म����त�न�ध�व�;
◦ महानगरीय��े��के��लए�योजना�और�सम�वय�के�संबंध�म��ऐसी�स�म�तय��के�काय��; तथा�
◦ ऐसी�स�म�तय��के�अ�य���के�चुनाव�का�तरीका।�

• यह�अ�ध�नयम�कहता�है��क�एक�महानगरीय�योजना�स�म�त�के�दो-�तहाई�सद�य��का�चुनाव�नगर�पा�लका��के��नवा��चत�सद�य��और
पंचायत��के�अ�य����ारा��कया�जाना�चा�हए।�
• स�म�त�म��इन�सद�य��का���त�न�ध�व�उस�महानगरीय��े��म��नगरपा�लका��और�पंचायत��क��जनसं�या�के�अनुपात�के�अनुपात�म�
होना�चा�हए।�
• ऐसी�स�म�तय��के�अ�य��रा�य�सरकार�के�पास��वकास�योजना�को�आगे�बढ़ाएंगे।�

कथन��व�ेषण:

कथन�1 कथन�2 कथन�3

अस�य� अस�य� स�य�

IASbaba
Web: http://ilp.iasbaba.com/ Score:
Email: ilp@iasbaba.com 0.00 / 200
Page 121
AIPTS/ILP VETERANS-
Exam Title :
2020 TE...
Email : yadavanurag075@gmail.com
Contact : 8882839768

एक��े���जसम��10 लाख�या�उससे
अ�धक�क��आबाद��है�, एक�या�एक�से नगर�पा�लका��के��नवा��चत�सद�य�और रा�य��वधा�यका�एमपीसी�क��संरचना
अ�धक��जल��म��और�दो�या�अ�धक�नगर पंचायत��के�अ�य��दो�-�तहाई�MPC सद�य और�सद�य��के�चुनाव�के�तरीके�का
पा�लका��या�पंचायत��या�अ�य�सं�ामक का�चुनाव�करते�ह�।� �नधा�रण�कर�सकती�है।�
�े���से��मलकर�बना�है�

QUESTION 23.
भारतीय�सं�वधान�क��12 व��अनुसूची�के�तहत��न�न�ल�खत�म��से�कौन�सा��वषय�शा�मल�नह��है:

a) �लम�सुधार�और�उ�यन�
b) मवेशी�तालाब�, पशु��के��लए��ूरता�क��रोकथाम�
c) अ��नशमन�सेवाएं�
d) वय�क�और�गैर-औपचा�रक��श�ा�
Correct Answer: D
Your Answer: Unanswered
Explanation

Solution (d)

Basic Information:

74 व��सं�वधान�(संशोधन) अ�ध�नयम�, 1992 ने�सं�वधान�म��एक�नई�बारहव��अनुसूची�जोड़ी।�इस�अनुसूची�म��नगरपा�लका��के�18 काया�


�मक��वषय�शा�मल�ह�।�यह�अनु�छे द�243-W से�संबं�धत�है�

12 व��अनुसूची�के�तहत�कुछ�काया��मक��वषय:

• नगर��नयोजन�स�हत�शहरी��नयोजन�
• भू�म�के�उपयोग�और�भवन��के��नमा�ण�का��व�नयमन�
• आ�थ�क�और�सामा�जक��वकास�के��लए�योजना�बनाना�
• शहरी�वा�नक��, पया�वरण�क��सुर�ा�और�पा�र��थ�तक�पहलु��का�संवध�न�
• क��तान�और�क��तान�के�मैदान�, �मशान�और��मशान�के�मैदान�और��व�ुत्��मशान�;
• बूचड़खान��और�टे न�रय��का��नयमन।�
• ज�म�और�मृ�यु�के�पंजीकरण�स�हत�मह�वपूण��आँकड़े�;
• साव�ज�नक�सु�वधाएं��जनम�����ट�लाइ�ट�ग�, पा�क�ग��थल�, बस��टॉप�और�साव�ज�नक�सु�वधाएं�शा�मल�ह��;

वय�क�और�गैर-औपचा�रक��श�ा�11 व��अनुसूची�का�एक��वषय�है।�

QUESTION 24.
�न�न�ल�खत�कथन��पर��वचार�कर�:

1. संसद�के�अ�ध�नयम���ारा�रा�य��म��नगर��नगम��था�पत��कए�जाते�ह�।�
2. एक�मेयर�एक�नगर��नगम�क��प�रषद�का��मुख�होता�है।�
3. नगरपा�लका�आयु��प�रषद�और�उसक���थायी�स�म�तय���ारा��लए�गए��नण�य��के�काया��वयन�के��लए�उ�रदायी�है।�

ऊपर��दए�गए�कथन��म��से�कौन�सा�सही�है�/ ह��?

a) केवल�1
b) केवल�1 और�2
c) केवल�2 और�3

IASbaba
Web: http://ilp.iasbaba.com/ Score:
Email: ilp@iasbaba.com 0.00 / 200
Page 122
AIPTS/ILP VETERANS-
Exam Title :
2020 TE...
Email : yadavanurag075@gmail.com
Contact : 8882839768

d) 1, 2 और�3
Correct Answer: C
Your Answer: Unanswered
Explanation

Solution (c)

Basic Information:

• �द�ली�, मुंबई�, कोलकाता�, हैदराबाद�, ब�गलोर�और�अ�य�जैसे�बड़े�शहर��के��शासन�के��लए�नगर��नगम�बनाए�जाते�ह�।�वे�रा�य��म�


संबं�धत�रा�य��वधानसभा��के�अ�ध�नयम���ारा�, और�भारत�के�संसद�के�अ�ध�नयम���ारा�क���शा�सत��दे श��म���था�पत��कए�जाते
ह�।�
• रा�य�के�सभी�नगर��नगम��के��लए�एक�सामा�य�काय��हो�सकता�है�या���येक�नगर��नगम�के��लए�एक�अलग�अ�ध�नयम�हो�सकता�है।�
• एक�नगर��नगम�म��तीन��ा�धकरण�होते�ह��, अथा�त्�प�रषद�, �थायी�स�म�तयाँ�और�आयु�।�
• प�रषद��नगम�क���वचारशील�और��वधायी�शाखा�है।�इसम��सीधे�तौर�पर�लोग���ारा�चुने�गए�पाष�द�और�कुछ�नामां�कत�����शा�मल
होते�ह�।�अनुसू�चत�जा�त�, अनुसू�चत�जनजा�त�और�म�हला��के��लए�सीट��के�आर�ण�स�हत�प�रषद�क��संरचना�74 व��संवैधा�नक
संशोधन�अ�ध�नयम��ारा�शा�सत�है।�
• प�रषद�का�नेतृ�व�मेयर�करता�है।�उ�ह��उप�महापौर��ारा�सहायता��दान�क��जाती�है।�वह�मूल��प�से�एक�मह�वपूण������और��नगम
के�एक�औपचा�रक��मुख�ह�।�उनका�मु�य�काय��प�रषद�क��बैठक��क��अ�य�ता�करना�है।�
• �थायी�स�म�तयाँ�प�रषद�के�काम�को�सु�वधाजनक�बनाने�के��लए�बनाई�गई�ह��, जो�आकार�म��ब�त�बड़ी�है।�वे�साव�ज�नक�काय��, �शक्
षा�, �वा��य�, कराधान�, �व��आ�द�से��नपटते�ह�।�
• नगरपा�लका�आयु��रा�य�सरकार��ारा��नयु���कया�जाता�है�और�आम�तौर�पर�IAS अ�धकारी�होता�है।�

कथन��व�ेषण:

कथन�1 कथन�2 कथन�3

अस�य� स�य� स�य�

नगर��नगम�आयु���नगम�का�मु�य
संबं�धत�रा�य��वधानसभा��के काय�कारी�अ�धकारी�है�तथा�प�रषद�और
नगर��नगम�क��अ�य�ता�मेयर�क�
अ�ध�नयम���ारा�रा�य��म��नगर��नगम उसक���थायी�स�म�तय���ारा��लए�गए
अ�य�ता�म��होती�है।�
�था�पत��कए�जाते�ह�।� �नण�य��के�काया��वयन�के��लए�उ�रदायी
है।�

QUESTION 25.
नगर�पा�लका�के�संदभ��म���न�न�ल�खत�कथन��पर��वचार�कर�:

1. एक�नगरपा�लका�म��भी�केवल�दो��ा�धकरण�होते�ह��, अथा�त्�प�रषद�और�मु�य�काय�कारी�अ�धकारी।�
2. अ�य��/ �मुख�न�केवल�प�रषद�क��बैठक��क��अ�य�ता�करते�ह��, ब��क�काय�कारी�श��य��का�भी�आनंद�लेते�ह�।�

ऊपर��दए�गए�कथन��म��से�कौन�सा�सही�है�/ ह��?

a) केवल�1
b) केवल�2
c) दोन��1 और�2

IASbaba
Web: http://ilp.iasbaba.com/ Score:
Email: ilp@iasbaba.com 0.00 / 200
Page 123
AIPTS/ILP VETERANS-
Exam Title :
2020 TE...
Email : yadavanurag075@gmail.com
Contact : 8882839768

d) न�तो�1 और�न�ही�2
Correct Answer: B
Your Answer:
Explanation

Solution (b)

Basic Information:

• नगरपा�लका�क�ब��और�छोटे �शहर��के��शासन�के��लए��था�पत�क��जाती�ह�।��नगम��क��तरह�, वे�भी�रा�य��म��संबं�धत�रा�य


�वधानसभा��के�अ�ध�नयम��और�भारत�के�संसद�के�अ�ध�नयम���ारा�क���शा�सत��दे श�म���था�पत��कए�जाते�ह�।�
• उ�ह��कई�अ�य�नाम��से�भी�जाना�जाता�है�जैसे�नगरपा�लका�प�रषद�, नगरपा�लका�स�म�त�, नगरपा�लका�बोड��, नगर�पा�लका�, शहरी
नगर�पा�लका�और�अ�य।�
• नगर�पा�लका�प�रषद�क��अ�य�ता�अ�य��/ चेयरमैन�करता�है।�उ�ह��एक�उपा�य���ारा�सहायता��दान�क��जाती�है।�
• मु�य�काय�कारी�अ�धकारी�/ मु�य�नगरपा�लका�अ�धकारी�नगर�पा�लका�के��दन-��त�दन�के�सामा�य��शासन�के��लए�उ�रदायी�होता
है।�उनक���नयु���रा�य�सरकार��ारा�क��जाती�है।�

कथन��व�ेषण:

कथन�1 कथन�2

अस�य� स�य�

नगर��नगम�क��तरह�, एक�नगर�पा�लका�म��भी�तीन प�रषद�क��अ�य�ता�एक�अ�य��/ चेयरमैन�करता�है।�प�रषद


�ा�धकरण�होते�ह��, अथा�त्�प�रषद�, �थायी�स�म�तयाँ�और क��बैठक��क��अ�य�ता�करने�के�अलावा�, उसे�काय�कारी
मु�य�काय�कारी�अ�धकारी।� श��याँ�भी��ा�त�होती�ह�।�

QUESTION 26.
�न�न�ल�खत�पर��वचार�कर�:

1. अ�धसू�चत��े��स�म�त�( notified area committee) एक�सां�व�धक��नकाय�है।�


2. अ�धसू�चत��े��स�म�त�को�रा�य�नगरपा�लका�अ�ध�नयम�के�सभी��ावधान��के�अनुसार�काय��करना�होता�है।�

ऊपर��दए�गए�कथन��म��से�कौन�सा�सही�है�/ ह��?

a) केवल�1
b) केवल�2
c) दोन��1 और�2
d) न�तो�1 और�न�ही�2
Correct Answer: D
Your Answer:
Explanation

Solution (d)

IASbaba
Web: http://ilp.iasbaba.com/ Score:
Email: ilp@iasbaba.com 0.00 / 200
Page 124
AIPTS/ILP VETERANS-
Exam Title :
2020 TE...
Email : yadavanurag075@gmail.com
Contact : 8882839768

Basic Information:

• दो��कार�के��े���के��शासन�के��लए�एक�अ�धसू�चत��े��स�म�त�बनाई�गई�है�- औ�ोगीकरण�के�कारण�तेजी�से��वक�सत�हो�रहा�नगर
, और�एक�नगर�जो�अभी�तक�नगरपा�लका�के�गठन�के��लए�आव�यक�सभी�शत��को�पूरा�नह��करता�है�, ले�कन�रा�य�सरकार��ारा
�जसे�अ�यथा�मह�वपूण��माना�जाता�है।�
• चूं�क�यह�सरकारी�गजट�म��एक�अ�धसूचना��ारा��था�पत�होता�है�, इस�लए�इसे�अ�धसू�चत��े��स�म�त�कहा�जाता�है।�
• इसे�रा�य�नगरपा�लका�अ�ध�नयम�के�आव�यक��ावधान��क��तुलना�म���कसी�अ�य�अ�ध�नयम�के�तहत�श��य��का��योग�करने�के��लए
स�पा�जा�सकता�है�
• इसक��श��यां�लगभग�एक�नगरपा�लका�के�समान�होती�ह�।�ले�कन�नगरपा�लका�के��वपरीत�, यह�एक�पूरी�तरह�से�ना�मत��नकाय�है�,
अथा�त�, अ�य��स�हत�एक�अ�धसू�चत��े��स�म�त�के�सभी�सद�य�रा�य�सरकार��ारा�ना�मत�होते�ह�।�इस��कार�, यह�न�तो�एक
�नवा��चत��नकाय�है�और�न�ही�एक�वैधा�नक��नकाय�है।�

कथन��व�ेषण:

कथन�1 कथन�2

अस�य� अस�य�

यह�रा�य�नगरपा�लका�अ�ध�नयम�के�ढांचे�के�भीतर�काय��करता�है
टाउन�ए�रया�कमेट��सरकारी�गजट�म��एक�अ�धसूचना��ारा
, केवल�अ�ध�नयम�के�वह��ावधान�लागू�होते�ह��जो�सरकारी�गजट
�था�पत�क��गई�है�, इस�लए�यह�एक�वैधा�नक��नकाय�नह��है।�
म��अ�धसू�चत��कए�जाते�ह���जसके��ारा�इसे�बनाया�जाता�है।�

QUESTION 27.
छावनी�बोड��के�संदभ��म���न�न�ल�खत�कथन��पर��वचार�कर�:

1. यह�संबं�धत�रा�य�सरकार���ारा�अ�ध�नय�मत�कानून�के��ावधान��के�तहत��था�पत��कया�गया�है।�
2. छावनी�बोड��के�काय�कारी�अ�धकारी�क���नयु���भारत�के�रा�प�त��ारा�क��जाती�है।�
3. �टे शन�क��कमान�संभालने�वाला�सै�य�अ�धकारी�बोड��का�पदे न�अ�य��होता�है�तथा�इसक��बैठक��क��अ�य�ता�करता�है।�

ऊपर��दए�गए�कथन��म��से�कौन�सा�सही�है�/ ह��?

a) केवल�1 और�2
b) केवल�1 और�3
c) केवल�2 और�3
d) 1, 2 और�3
Correct Answer: C
Your Answer:
Explanation

Solution (c)

Basic Information:

• छावनी��े��म��नाग�रक�आबाद��के��लए�नगरपा�लका��शासन�हेतु�एक�छावनी�बोड���था�पत��कया�गया�है।�

IASbaba
Web: http://ilp.iasbaba.com/ Score:
Email: ilp@iasbaba.com 0.00 / 200
Page 125
AIPTS/ILP VETERANS-
Exam Title :
2020 TE...
Email : yadavanurag075@gmail.com
Contact : 8882839768

• यह�2006 के�छावनी�अ�ध�नयम�- क���सरकार��ारा�अ�ध�नय�मत�कानून�के��ावधान��के�तहत��था�पत��कया�गया�है�।�इस�अ�ध�नयम


ने�1924 के�छावनी�अ�ध�नयम�को��नर�त�कर��दया�है।�
• यह�क���सरकार�के�र�ा�मं�ालय�के��शास�नक��नयं�ण�म��काय��करता�है।�
• एक�छावनी�बोड��म��आं�शक��प�से��नवा��चत�और�आं�शक��प�से�ना�मत�सद�य�होते�ह�।��नवा��चत�सद�य�पाँच�साल�के�काय�काल�के
�लए�पद�पर�रहते�ह��जब�क�मनोनीत�सद�य�(यानी�, पदे न�सद�य) तब�तक�जारी�रहते�ह��जब�तक�वे�उस��टे शन�म��काया�लय�का
संचालन�करते�ह�।�
• बोड��के�उपा�य��को��नवा��चत�सद�य���ारा�पांच�साल�क��अव�ध�के��लए�अपने�बीच�से�चुना�जाता�है।�
• एक�छावनी�बोड���ारा��कए�गए�काय��एक�नगर�पा�लका�के�समान�ह��
• ये�सां�व�धक�काय��और��ववेकाधीन�काय��म��वैधा�नक��प�से�वग�कृत�ह�।�आय�के��ोत��म��कर�राज�व�और�गैर-कर�राज�व�दोन�
शा�मल�ह�।�
• छावनी�बोड��का�काय�कारी�अ�धकारी�बोड��और�उसक��स�म�तय��के�सभी���ताव��और��नण�य��को�लागू�करता�है।�

Elimination:

य�द�आप�जानते�ह���क�छावनी�बोड��क���य��प�से��शा�सत�सं�थाएँ�ह��, न��क�रा�य�सरकार��, तो�आप�कथन�1 वाले�सभी��वक�प��को�समा�त


कर�सकते�ह�।�इस�लए�, हम��केवल�‘ �वक�प�( c)' के�साथ�हमारे�उ�र�के��प�म��छोड़��दया�जाता�है।�

कथन��व�ेषण:

कथन�1 कथन�2 कथन�3

अस�य� स�य� स�य�

यह�2006 के�छावनी छावनी�बोड��के�काय�कारी�अ�धकारी�क�


अ�ध�नयम�- क���सरकार��ारा �नयु���भारत�के�रा�प�त��ारा�क��जाती�है। �टे शन�क��कमान�संभालने�वाला�सै�य
अ�ध�नय�मत�कानून�के वह�इस�उ�े �य�के��लए��था�पत�क���य�कैडर अ�धकारी�छावनी�बोड��का��मुख�होता
�ावधान��के�तहत��था�पत से�संबं�धत�है�, जो�संभवतः��डफ�स�ए�टे ट है।�
�कया�गया�है।� स�व�सेज�से�होता�है।�

QUESTION 28.
�वशेष��योजन�एज�सी�( Special Purpose Agency) के�बारे�म���न�न�ल�खत�कथन��पर��वचार�कर�:

1. यह�एक�काय�-आधा�रत�इकाई�है�जो��न�द���ग�त�व�धय��या��व�श��काय��को�करने�के��लए��था�पत�क��जाती�है।�
2. रा�य��वधानमंडल�के�एक�अ�ध�नयम��ारा�उ�ह��वैधा�नक��नकाय��के��प�म���था�पत��कया�जा�सकता�है।�
3. वे��थानीय�नगर��नकाय��के�अधीन�थ�एज��सय��के��प�म��काय��करते�ह�।�

ऊपर��दए�गए�कथन��म��से�कौन�सा�सही�है�/ ह��?

a) केवल�1
b) केवल�1 और�2
c) केवल�2 और�3
d) 1, 2 और�3
Correct Answer: B
Your Answer:
Explanation

IASbaba
Web: http://ilp.iasbaba.com/ Score:
Email: ilp@iasbaba.com 0.00 / 200
Page 126
AIPTS/ILP VETERANS-
Exam Title :
2020 TE...
Email : yadavanurag075@gmail.com
Contact : 8882839768

Solution (b)

Basic Information:

• �वशेष��योजन�एज��सयां�रा�य�सरकार��ारा��न�द���ग�त�व�धय��या��व�श��काय��को�करने�के��लए��था�पत�क��जाती�ह��जो�' वैध��प
से�' नगर��नगम��या�नगर�पा�लका��या�अ�य��थानीय�शहरी�सरकार��के�डोमेन�से�संबं�धत�ह�।�
• उ�ह��‘ एकल�उ�े �य�’, ‘ एक-उ�े �य�’ या�‘ �वशेष�उ�े �य�’ एज��सय��या�’ काया��मक��थानीय��नकाय��’ के��प�म��जाना�जाता�है।�
• �वशेष��योजन�एज��सय��के�कुछ�उदाहरण:

हाउ�स�ग�बोड��, ��षण��नयं�ण�बोड��, �बजली�आपू�त��बोड��, �सट���ांसपोट� �बोड��, शहरी��वकास��ा�धकरण�आ�द।�

कथन��व�ेषण:

कथन�1 कथन�2 कथन�3

स�य� स�य� अस�य�

वे��वाय���नकाय��के��प�म�
वे�रा�य��वधा�यका�के�एक�अ�ध�नयम�या
�वशेष��योजन�एज��सयां�काय�-आधा�रत काय��करते�ह��तथा��थानीय
एक�काय�कारी�संक�प��ारा��वभाग��के��प
होती�ह��और�न��क��े�-आधा�रत�जैसे शहरी�सरकार��को��वतं���प
म��वैधा�नक��नकाय��के��प�म���था�पत�होती
नगर�पा�लका�, छावनी�बोड��इ�या�द।� से�आवं�टत��कए�गए�काय��से
ह�।�
�नपटते�ह�।�

QUESTION 29.
�न�न�ल�खत��थानीय�सरकारी�सं�था��म��से�कौन�रा�य��वधानसभा��के�अ�ध�नयम��ारा��था�पत�नह��है/ ह��?

1. छावनी�बोड��( Cantonment Board)


2. टाउन�शप�( Township)
3. टाउन�ए�रया�कमेट��( Town Area Committee)
4. अ�धसू�चत��े��स�म�त�( Notified Area Committee)

�न�न�ल�खत�म��से�सही�कूट�का�चयन�कर�:

a) केवल�1 और�2
b) केवल�1 और�3
c) केवल�1, 2 और�4
d) 1, 2, 3 और�4
Correct Answer: C
Your Answer:
Explanation

Solution (c)

IASbaba
Web: http://ilp.iasbaba.com/ Score:
Email: ilp@iasbaba.com 0.00 / 200
Page 127
AIPTS/ILP VETERANS-
Exam Title :
2020 TE...
Email : yadavanurag075@gmail.com
Contact : 8882839768

Basic Information:

• भारत�म��‘ शहरी��थानीय�सरकार�’ श�द�अपने�चुने��ए���त�न�धय��के�मा�यम�से�लोग���ारा�एक�शहरी��े��के�शासन�को�दशा�ता�है।


एक�शहरी��थानीय�सरकार�का�अ�धकार��े��एक��व�श��शहरी��े��तक�सी�मत�होता�है�, �जसे�रा�य�सरकार��ारा�इस�उ�े �य�के��लए
सीमां�कत��कया�जाता�है।�
• ' �थानीय�सरकार�' सं�वधान�क��7 व��अनुसूची�के�तहत�रा�य�सूची�म��उ��ल�खत�एक��वषय�है।�
• भारत�म��आठ��कार�क��शहरी��थानीय�सरकार��ह�- नगर��नगम�, नगर�पा�लका�, अ�धसू�चत��े��स�म�त�, नगर��े��स�म�त�, छावनी
बोड��, टाउन�शप�, पोट� ���ट�और��वशेष��योजन�एज�सी।�ले�कन�सभी�रा�य��वधानसभा��के�अ�ध�नयम��ारा��था�पत�नह��ह�।�
• रा�य�म��नगर��नगम�, नगर�पा�लका�, टाउन�ए�रया�कमेट��, �वशेष��योजन�एज�सी�आ�द�रा�य��वधानमंडल�के�अ�ध�नयम��ारा��था�पत
�कए�जाते�ह�।�

नगर��े��स�म�त�( Town Area Committee ):

एक�छोटे �क�बे�के��शासन�के��लए�एक�नगर��े��स�म�त�का�गठन��कया�जाता�है।�यह�एक�अध�-नगरपा�लका��ा�धकरण�है�और�इसे�नाग�रक
काय��क��एक�सी�मत�सं�या�स�पा�गया�है�जैसे�जल��नकासी�, सड़क��, सड़क��काश��व�था�और�संर�ण।�इसक��संरचना�, काय��और�अ�य
मामले�अ�ध�नयम��ारा�शा�सत�होते�ह�।�यह�पूण���प�से��नवा��चत�या�पूण���प�से�रा�य�सरकार��ारा�ना�मत�या�आं�शक��प�से��नवा��चत�और
आं�शक��प�से�नामां�कत�हो�सकता�है�

टाउन�शप�( Township ):

यह�बड़े�साव�ज�नक�उ�म���ारा�अपने�कम�चा�रय��और���मक��को�नाग�रक�सु�वधाएं��दान�करने�के��लए��था�पत��कया�जाता�है�जो��लांट�के
पास�बनी�हाउ�स�ग�कॉलो�नय��म��रहते�ह�।�उ�म�टाउन�शप�के��शासन�क��दे खभाल�के��लए�एक�टाउन��शासक��नयु��करता�है।�शहरी�सरकार
के�टाउन�शप�फॉम��म��कोई��नवा��चत�सद�य�नह��होता�है।�वा�तव�म��, यह�उ�म��क��नौकरशाही�संरचना�का�एक��व�तार�है।�

कथन��व�ेषण:

कथन�1 कथन�2 कथन�3 कथन�4

अस�य� अस�य� स�य� अस�य�

यह�बड़े�साव�ज�नक�उ�म���ारा
यह�2006 के�छावनी
अपने�कम�चा�रय��और���मक�
अ�ध�नयम�- क���सरकार��ारा यह�रा�य��वधानमंडल�के�एक यह�सरकारी�गजट�म��एक
को�नाग�रक�सु�वधाएं��दान
अ�ध�नय�मत�कानून�के अलग�अ�ध�नयम��ारा�बनाया अ�धसूचना��ारा��था�पत��कया
करने�के��लए��था�पत��कया
�ावधान��के�तहत��था�पत गया�है।� गया�है�
जाता�है�जो��लांट�के�पास�बनी
�कया�गया�है।�
हाउ�स�ग�कॉलो�नय��म��रहते�ह�।�

QUESTION 30.
क���शा�सत��दे श��के��शासन�के�संदभ��म���न�न�ल�खत�कथन��पर��वचार�कर�:

1. क���शा�सत��दे श�का�एक��शासक�रा�प�त��ारा��नयु���कया�जाता�है�तथा�एक�रा�यपाल�क��तरह�रा�य�का��मुख�होता�है।�
2. रा�प�त��कसी�रा�य�के�रा�यपाल�को�समीपवत��क���शा�सत��दे श�के��शासक�के��प�म��भी��नयु��कर�सकता�है।�

ऊपर��दए�गए�कथन��म��से�कौन�सा�सही�है�/ ह��?

a) केवल�1

IASbaba
Web: http://ilp.iasbaba.com/ Score:
Email: ilp@iasbaba.com 0.00 / 200
Page 128
AIPTS/ILP VETERANS-
Exam Title :
2020 TE...
Email : yadavanurag075@gmail.com
Contact : 8882839768

b) केवल�2
c) दोन��1 और�2
d) न�तो�1 और�न�ही�2
Correct Answer: B
Your Answer:
Explanation

Solution (b)

Basic Information:

• सं�वधान�के�भाग�VIII म��क���शा�सत��दे श��के�साथ�अनु�छे द�239 से�241 के�अनुसार�भले�ही�सभी�क���शा�सत��दे श�एक��ेणी�के


ह��, ले�कन�उनक���शास�नक��व�था�म��एक�पता�नह��है।�
• ��येक�क���शा�सत��दे श�को�रा�प�त��ारा��नयु���शासक��ारा��शा�सत��कया�जाता�है।�
• रा�प�त�एक��शासक�के�पदनाम�को��न�द���कर�सकता�है�; यह�उपरा�यपाल�या�मु�य�आयु��या��शासक�हो�सकता�है।�वत�मान�म��,
यह��द�ली�, पुडुचेरी�, अंडमान�और��नकोबार���प�समूह�और�ल�ाख�के�मामले�म��उपरा�यपाल�है।�चंडीगढ़�, दादरा�और�नगर�हवेली�,
दमन�और�द�व�और�ल���प�के�मामले�म���शासक�है।�
• क���शा�सत��दे श�पुडुचेरी�( 1963 म�) और��द�ली�( 1992 म�) एक��वधान�सभा�और�एक�मु�यमं�ी�क��अ�य�ता�वाले�मं��य��क�
एक�प�रषद��दान�क��गयी�ह�।�

कथन��व�ेषण:

कथन�1 कथन�2

अस�य� स�य�

क���शा�सत��दे श�का�एक��शासक�रा�प�त�का�एज�ट�होता�है इस�मामले�म��, रा�यपाल�अपने�मं��प�रषद�से��वतं���प�से�काय�


तथा�रा�यपाल�क��तरह�रा�य�का��मुख�नह��होता�है।� करता�है।�

QUESTION 31.
�न�न�ल�खत�कथन��पर��वचार�कर�:

1. संसद�सभी�क���शा�सत��दे श��के��लए�तीन�सू�चय��(रा�य�सूची�स�हत) के��कसी�भी��वषय�पर�कानून�बना�सकती�है।�


2. भारत�का�सव��च��यायालय�क���शा�सत��दे श�के��लए�एक�उ�च��यायालय��था�पत�कर�सकता�है�या�उसे�समीपवत��रा�य�के�उ�च
�यायालय�के�अ�धकार��े��म��डाल�सकता�है।�

ऊपर��दए�गए�कथन��म��से�कौन�सा�सही�है�/ ह��?

a) केवल�1
b) केवल�2
c) दोन��1 और�2
d) न�तो�1 और�न�ही�2
Correct Answer: A
Your Answer:
Explanation

IASbaba
Web: http://ilp.iasbaba.com/ Score:
Email: ilp@iasbaba.com 0.00 / 200
Page 129
AIPTS/ILP VETERANS-
Exam Title :
2020 TE...
Email : yadavanurag075@gmail.com
Contact : 8882839768

Solution (a)

Basic Information:

• क��शा�सत��दे श��के��लए�संसद�तीन�सू�चय��(रा�य�सूची�स�हत) के��कसी�भी��वषय�पर�कानून�बना�सकती�है।�संसद�क��यह�श��


पुडुचेरी�और��द�ली�तक�भी��व�तृत�है�, �जनक��अपनी��वधानसभाएं�ह�।�
• ले�कन�, पु�चेरी�क���वधान�सभा�रा�य�सूची�और�समवत��सूची�के��कसी�भी��वषय�पर�कानून�बना�सकती�है।�इसी��कार�, �द�ली�क�
�वधान�सभा�रा�य�सूची�(साव�ज�नक��व�था�, पु�लस�और�भू�म�को�छोड़कर) और�समवत��सूची�के��कसी�भी��वषय�पर�कानून�बना
सकती�है।�
• रा�प�त�अंडमान�और��नकोबार���प�समूह�, ल���प�, दादरा�और�नगर�हवेली�और�दमन�और�द�व�क��शां�त�, �ग�त�और�अ�छ�
सरकार�के��लए��नयम�बना�सकते�ह�।�पुडुचेरी�के�मामले�म��भी�, रा�प�त��व�नयम�बनाकर�कानून�बना�सकते�ह��, ले�कन�केवल�तभी�जब
�वधानसभा��नलं�बत�या�भंग�कर�द��जाए।�रा�प�त��ारा�बनाए�गए��व�नयमन�म��संसद�के�अ�ध�नयम�के�समान�श���और��भाव�होता
है�और�इन�क���शा�सत��दे श��के�संबंध�म��संसद�के��कसी�भी�काय��को��नर�त�या�संशो�धत�भी�कर�सकता�है।�
• �द�ली�एकमा��क���शा�सत��दे श�है��जसका�अपना�उ�च��यायालय�है�( 1966 से)।�बॉ�बे�उ�च��यायालय�को�दो�क���शा�सत��दे श�-
दादरा�और�नगर�हवेली�और�दमन�और�द�व�पर�अ�धकार��े���मल�गया�है।�अंडमान�और��नकोबार���प�समूह�, चंडीगढ़�, ल���प�और
पु�चेरी�को��मशः�कलक�ा�, पंजाब�और�ह�रयाणा�, केरल�और�म�ास�उ�च��यायालय��के�अधीन�रखा�गया�है।�

कथन��व�ेषण:

कथन�1 कथन�2

स�य� अस�य�

रा�य�सूची�स�हत�सभी�तीन�सू�चय��के��वषय��पर�क���शा�सत
संसद�म��संघ�शा�सत��दे श��के��लए�एक�उ�च��यायालय��था�पत
�दे श��के��लए�संसद�क���वधायी�श���संघ�शा�सत��दे श��के
करने�या�उ�ह��समीपवत��रा�य�के�उ�च��यायालय�के�अ�धकार
�लए�एक��थानीय��वधा�यका��था�पत�करने�के�बाद�भी
�े��म��लाने�क��श��याँ�ह�।�
अ�भा�वत�रहती�है।�

QUESTION 32.
�द�ली�के��लए��वशेष��ावधान��के�बारे�म���न�न�ल�खत�म��से�कौन�सा�कथन�सही�है�?

a) �वधान�सभा�क��साम�य��60 सद�य��तक��न��त�है�
b) �द�ली�का�रा�य��नवा�चन�आयोग��द�ली�क���वधान�सभा�के��लए�चुनाव�आयो�जत�करता�है।�
c) �द�ली�के�मु�यमं�ी�को��द�ली�के�उपरा�यपाल��ारा��नयु���कया�जाता�है।�
d) �वधानसभा�क��कुल�साम�य��के�दस���तशत�तक�मं��य��क��प�रषद�क��साम�य���न��त�होती�है।�
Correct Answer: D
Your Answer:
Explanation

Solution (d)

Basic Information:

• 1991 के�69 व��संवैधा�नक�संशोधन�अ�ध�नयम�ने��द�ली�के�क���शा�सत��दे श�को�एक��वशेष�दजा���दान��कया�, और�इसे��द�ली�के


रा�ीय�राजधानी��े��को��फर�से��डज़ाइन��कया�तथा��द�ली�के��शासक�को�ले��टन�ट�गवन�र�(उप-रा�यपाल) के��प�म��ना�मत��कया।�

IASbaba
Web: http://ilp.iasbaba.com/ Score:
Email: ilp@iasbaba.com 0.00 / 200
Page 130
AIPTS/ILP VETERANS-
Exam Title :
2020 TE...
Email : yadavanurag075@gmail.com
Contact : 8882839768

• रा�य��वधान�सभा�रा�य�सूची�और�समवत��सूची�के�सभी�मामल��पर�कानून�बना�सकती�है�, �सवाय�रा�य�सूची�के�तीन�मामल��को
छोड़कर�, अथा�त्�साव�ज�नक��व�था�, पु�लस�और�भू�म।�ले�कन�, संसद�के�कानून��वधानसभा��ारा�बनाए�गए�कानून�पर��भावी�होते
ह�।�
• मु�यमं�ी�क���नयु���रा�प�त��ारा�क��जाती�है�(उप-रा�यपाल��ारा�नह�)।�अ�य�मं��य��को�मु�यमं�ी�क��सलाह�पर�रा�प�त��ारा
�नयु���कया�जाता�है।�रा�प�त�के��सादपय�त�मं�ी�काया�लय�म��बने�रहते�ह�।�
• उप�-रा�यपाल�और�उनके�मं��य��के�बीच�राय�के�अंतर�के�मामले�म��उप�-रा�यपाल��नण�य�के��लए�रा�प�त�को�मामले�को�संद�भ�त
करता�है�और�तदनुसार�काय��करता�है।�

कथन��व�ेषण:

�वक�प�a �वक�प�b �वक�प�c �वक�प�d

अस�य� अस�य� अस�य� स�य�

�द�ली�के�मं��य��क��प�रषद�क�
चुनाव�का�संचालन�भारत�के मु�यमं�ी�क���नयु���रा�प�त
�वधान�सभा�क��साम�य��70 स सं�या�सात�है�- एक�मु�यमं�ी
चुनाव�आयोग��ारा��कया�जाता �ारा�क��जाती�है�(उप-रा�यपाल
द�य��पर��नधा��रत�है।� और�छह�अ�य�मं�ी�जो��क�70
है।� �ारा�नह�)।�
का�10% है।�

QUESTION 33.
�न�न�ल�खत�कथन��पर��वचार�कर�:

1. गृह�मं�ालय�, क���शा�सत��दे श��के�आव�यक�मामल��के��लए�नोडल�मं�ालय�है।�


2. गृह�मं�ी�क��सलाहकार�स�म�त�( HMAC) के�मंच�क��अ�य�ता�क���य�गृह�मं�ी��ारा�क��जाती�है।�

ऊपर��दए�गए�कथन��म��से�कौन�सा�सही�है�/ ह��?

a) केवल�1
b) केवल�2
c) दोन��1 और�2
d) न�तो�1 और�न�ही�2
Correct Answer: C
Your Answer:
Explanation

Solution (c)

Basic Information:

• भारत�सरकार�(�वसाय�का�आवंटन) �नयम�1961 के�तहत�, गृह�मं�ालय�कानून�, �व��और�बजट�, सेवा��और�उप-रा�यपाल�


और��शासक��क���नयु���से�संबं�धत�क���शा�सत��दे श��के�सभी�मामल��के��लए�नोडल�मं�ालय�है।�
• �बना��वधा�यका�(अंडमान�और��नकोबार���पसमूह�, चंडीगढ़�, दमन�और�द�व�, दादरा�और�नगर�हवेली�और�ल���प) के�सभी�पाँच
क���शा�सत��दे श��म��गृह�मं�ी�क��सलाहकार�स�म�त�( HMAC / �शासक�क��सलाहकार�स�म�त�( AAC) का�फोरम�है।�
• AAC संबं�धत�UTs के��शासक�क��अ�य�ता�म��है।�संसद�सद�य�और��थानीय��नकाय��से��नवा��चत�सद�य�उदाहरण��व�प�संबं�धत
पंचायत��के��जला�पंचायत�और�नगर�प�रषद��सर��अ�य�इन�स�म�तय��के�सद�य�होते�ह�।�स�म�त�संघ�शा�सत��दे श��के�सामा�जक�और
आ�थ�क��वकास�से�संबं�धत�सामा�य�मु���पर�चचा��करती�है।�

IASbaba
Web: http://ilp.iasbaba.com/ Score:
Email: ilp@iasbaba.com 0.00 / 200
Page 131
AIPTS/ILP VETERANS-
Exam Title :
2020 TE...
Email : yadavanurag075@gmail.com
Contact : 8882839768

QUESTION 34.
सं�वधान�क��छठ��अनुसूची�के�संदभ��म���न�न�ल�खत�कथन��पर��वचार�कर�:

1. यह�चार�उ�र-पूव��रा�य��असम�, मेघालय�, ��पुरा�और��मजोरम�म��जनजातीय��े���के��शासन�और��नयं�ण�से�संबं�धत�है।�


2. इसके�तहत�ऐसे�जनजातीय��े���वाले���येक�रा�य�को�एक�जनजातीय�सलाहकार�प�रषद�क���थापना�करनी�है।�

ऊपर��दए�गए�कथन��म��से�कौन�सा�सही�है�/ ह��?

a) केवल�1
b) केवल�2
c) दोन��1 और�2
d) न�तो�1 और�न�ही�2
Correct Answer: A
Your Answer:
Explanation

Solution (a)

Basic Information:

• सं�वधान�के�भाग�X म��अनु�छे द�244 ‘ अनुसू�चत��े���’ और�आ�दवासी��े���’ के��प�म���न�द���कुछ��े���के��लए��शासन�क��एक


�वशेष��णाली�क��प�रक�पना�करता�है।�
• सं�वधान�क��पांचव��अनुसूची�असम�, मेघालय�, ��पुरा�और��मजोरम�को�छोड़कर��कसी�भी�रा�य�म��अनुसू�चत��े���और�अनुसू�चत
जनजा�तय��के��शासन�और��नयं�ण�से�संबं�धत�है।�
• �सरी�ओर�, सं�वधान�क��छठ��अनुसूची�दे श�के�चार�पूव��र�रा�य��असम�, मेघालय�, ��पुरा�और��मजोरम�म��जनजातीय��े���के
�शासन�से�संबं�धत�है।�

जनजा�त�सलाहकार�प�रषद:

• पांचव��अनुसूची�के�अनुसार�, अनुसू�चत��े���वाले���येक�रा�य�को�अनुसू�चत�जनजा�तय��के�क�याण�और�उ��त�के��लए�सलाह�दे ने
के��लए�एक�जनजातीय�सलाहकार�प�रषद�क���थापना�करनी�है।�
• इसम��20 सद�य�शा�मल�ह��, �जनम��से�तीन-चौथाई�रा�य��वधान�सभा�म��अनुसू�चत�जनजा�तय��के���त�न�ध�होते�ह�।�

�वाय���जले�और��वाय���े�:

• छठ��अनुसूची�के�अनुसार�, चार�रा�य��असम�, मेघालय�, ��पुरा�और��मजोरम�म��जनजातीय��े���को��वाय���जल��के��प�म��ग�ठत


�कया�गया�है।�
• य�द�एक��वाय���जले�म��अलग-अलग�जनजा�तयां�ह��, तो�रा�यपाल��जले�को�कई��वाय���े���म���वभा�जत�कर�सकता�है।�
• ��येक��वाय���जले�म��एक��जला�प�रषद�होती�है��जसम��30 सद�य�होते�ह��, �जनम��से�चार�रा�यपाल��ारा�ना�मत��कए�जाते�ह��और
शेष�26 वय�क�मता�धकार�के�आधार�पर�चुने�जाते�ह�।�

कथन��व�ेषण:

कथन�1 कथन�2

स�य� अस�य�

IASbaba
Web: http://ilp.iasbaba.com/ Score:
Email: ilp@iasbaba.com 0.00 / 200
Page 132
AIPTS/ILP VETERANS-
Exam Title :
2020 TE...
Email : yadavanurag075@gmail.com
Contact : 8882839768

छठ��अनुसूची�के�तहत�सं�वधान�म��चार�उ�र-पूव��रा�य��असम जनजा�त�सलाहकार�प�रषद�पांचव��अनुसूची�क��एक��वशेषता�है।
, मेघालय�, ��पुरा�और��मजोरम�म��जनजातीय��े���के��शासन चार�रा�य��असम�, मेघालय�, ��पुरा�और��मजोरम�म��जनजातीय
के��लए��वशेष��ावधान�शा�मल�ह�।� �े���को��वाय���जल��के��प�म��ग�ठत��कया�गया�है।�

QUESTION 35.
पांचव��अनुसूची�म���न�हत��शासन�क���व�भ���वशेषता��के�बारे�म���न�न�ल�खत�कथन��पर��वचार�कर�:

1. रा�प�त�को�एक��े��को�अनुसू�चत��े��घो�षत�करने�का�अ�धकार�है।�
2. जनजा�त�सलाहकार�प�रषद�क��सलाह�के�बाद�रा�प�त�एक�अनुसू�चत��े��क��शां�त�और�अ�छ��सरकार�के��लए��नयम�बना�सकते�ह�।�

ऊपर��दए�गए�कथन��म��से�कौन�सा�सही�है�/ ह��?

a) केवल�1
b) केवल�2
c) दोन��1 और�2
d) न�तो�1 और�न�ही�2
Correct Answer: A
Your Answer:
Explanation

Solution (a)

Basic Information:

• सं�वधान�क��पांचव��अनुसूची�असम�, मेघालय�, ��पुरा�और��मजोरम�को�छोड़कर��कसी�भी�रा�य�म��अनुसू�चत��े���और�अनुसू�चत


जनजा�तय��के��शासन�और��नयं�ण�से�संबं�धत�है।�
• अनुसू�चत��े���को�दे श�के�अ�य��े���से�अलग�तरीके�से��शा�सत��कया�जाता�है��य��क�वे�‘ आ�दवा�सय��’ �ारा�बसे��ए�ह��जो
सामा�जक�और�आ�थ�क��प�से��पछड़े��ए�ह��, तथा�उनक����थ�त�म��सुधार�के��लए��वशेष��यास��कए�जाने�क��आव�यकता�है।
इस�लए�, एक�रा�य�म��काम�करने�वाले�पूरे�सामा�य��शास�नक�तं��को�अनुसू�चत��े���तक��व�ता�रत�नह���कया�जाता�है�और�क��
सरकार�क��इन��े���के��लए�कुछ�बड़ी��ज�मेदा�रयां�ह�।�

पांचव��अनुसूची�म���न�हत��शासन�क���वशेषताएं:

• एक�रा�य�क��काय�कारी�श���उसम��अनुसू�चत��े���तक�फैली��ई�है।�ले�कन�ऐसे��े���के�संबंध�म��रा�यपाल�क��एक��वशेष��ज�मेदारी
है।�उसे�ऐसे��े���के��शासन�के�बारे�म��रा�प�त�को���तवष��या�जब�भी�रा�प�त�को�आव�यकता�होती�है�, एक��रपोट� ���तुत�करनी�होती
है।�
• रा�यपाल�को�यह��नद� श�दे ने�का�अ�धकार�है��क�संसद�या�रा�य��वधा�यका�का�कोई��वशेष�अ�ध�नयम�अनुसू�चत��े��पर�लागू�नह��होता
है�या��न�द���संशोधन��और�अपवाद��के�साथ�लागू�होता�है।�एक��व�नयमन�संसद�या�रा�य��वधानमंडल�के��कसी�भी�अ�ध�नयम�को
�नर�त�या�संशो�धत�कर�सकता�है�, जो�एक�अनुसू�चत��े��पर�लागू�होता�है।�ले�कन�, ऐसे�सभी��नयम��म��रा�प�त�क��सहम�त�क�
आव�यकता�होती�है।�

कथन��व�ेषण:

कथन�1 कथन�2

IASbaba
Web: http://ilp.iasbaba.com/ Score:
Email: ilp@iasbaba.com 0.00 / 200
Page 133
AIPTS/ILP VETERANS-
Exam Title :
2020 TE...
Email : yadavanurag075@gmail.com
Contact : 8882839768

स�य� अस�य�

जनजा�तय��सलाहकार�प�रषद�से�परामश��के�बाद�रा�यपाल�एक
रा�प�त�को�न�केवल�एक��े��को�अनुसू�चत��े��घो�षत�करने
अनुसू�चत��े��क��शां�त�और�अ�छ��सरकार�के��लए��नयम�बना
का�अ�धकार�है�, ब��क�वह�अपने��े��को�बढ़ा�या�घटा�भी
सकते�ह�।�ऐसे��नयम�अनुसू�चत�जनजा�तय��के�सद�य��के�बीच
सकता�है�, अपनी�सीमा�रेखा�को�बदल�सकता�है�, ऐसे�पदनाम
या�भू�म�के�ह�तांतरण�को���तबं�धत�या�रोक�सकते�ह��, अनुसू�च
को�बदल�सकता�है�या�संबं�धत�रा�य�के�रा�यपाल�के�परामश��से
त�जनजा�तय��के�सद�य��क��भू�म�के�आवंटन�को��व�नय�मत�कर
�कसी��े��पर�इस�तरह�के�पुन�न�द�शन�के��लए�नए�आदे श�बना
सकते�ह��और�अनुसू�चत�जनजा�तय��के�संबंध�म��मौ��क�ऋण�के
सकता�है�
�वसाय�को��व�नय�मत�कर�सकते�ह�।�

QUESTION 36.
�न�न�ल�खत�जोड़े�पर��वचार�कर�:

रा�य�जनजातीय��े��

1. असम�बोडोल�ड��ादे �शक��े���जला�
2. �मजोरम�चकमा��जला�
3. मेघालय�गारो��ह�स��जला�

ऊपर�द��गई�कौन�सी�जोड़ी�सही�है�/ ह��?

a) केवल�1 और�2
b) केवल�1 और�3
c) केवल�2 और�3
d) 1, 2, और�3
Correct Answer: D
Your Answer:
Explanation

Solution (d)

Basic Information:

IASbaba
Web: http://ilp.iasbaba.com/ Score:
Email: ilp@iasbaba.com 0.00 / 200
Page 134
AIPTS/ILP VETERANS-
Exam Title :
2020 TE...
Email : yadavanurag075@gmail.com
Contact : 8882839768

QUESTION 37.
भारत�के�चुनाव�आयोग��ारा�मा�यता��ा�त�राजनी�तक�दल��को��दए�गए��वशेषा�धकार��के�संदभ��म���न�न�ल�खत�कथन��पर��वचार�कर�:

1. पाट��के��तीक��का�आवंटन।�
2. �नजी�टे ली�वजन�और�रे�डयो��टे शन��पर�राजनी�तक��सारण�के��लए�समय।�
3. मतदाता�सूची�तक�प�ंच।�

ऊपर��दए�गए�कथन��म��से�कौन�सा�सही�है�/ ह��?

a) केवल�1 और�2
b) केवल�1 और�3
c) केवल�2 और�3
d) 1, 2 और�3
Correct Answer: B
Your Answer:
Explanation

Solution (b)

Basic Information:

पा�ट�य��को�आयोग��ारा�द��गई�मा�यता�कुछ�अ�य��वशेषा�धकार��के��लए�उनके�अ�धकार�को��नधा��रत�करती�है�जैसे:

• नामांकन�दा�खल�करने�के��लए�मा�यता��ा�त�दल��को�केवल�एक���तावक�क��आव�यकता�होती�है।�
• उ�ह��चुनाव�के�समय�चालीस�" �टार��चारक�" रखने�क��अनुम�त�है�और�पंजीकृत-गैर�मा�यता��ा�त�दल��को�बीस�" �टार��चारक�" र
खने�क��अनुम�त�है।�
• उ�ह��रा�य�के��वा�म�व�वाले�टे ली�वजन�और�रे�डयो��टे शन��पर�राजनी�तक��सारण�और�मतदाता�सूची�तक�प�ंचने�का�समय��मलता�है।
• ��येक�मा�यता��ा�त�रा�ीय�पाट��को�पूरे�दे श�म��इसके�उपयोग�के��लए��वशेष��प�से�आर��त�एक��तीक�आवं�टत��कया�जाता�है।�
• ��येक�मा�यता�ा�त�रा�य�पाट��को�रा�य�या�रा�य��म��इसके�उपयोग�के��लए��वशेष��प�से�आर��त�एक��तीक�आवं�टत��कया�जाता
है।�

IASbaba
Web: http://ilp.iasbaba.com/ Score:
Email: ilp@iasbaba.com 0.00 / 200
Page 135
AIPTS/ILP VETERANS-
Exam Title :
2020 TE...
Email : yadavanurag075@gmail.com
Contact : 8882839768

QUESTION 38.
�न�न�ल�खत�कथन��पर��वचार�कर�:

1. चार�रा�य��म��रा�य�पाट��के��प�म��मा�यता��ा�त�एक�राजनी�तक�दल�को�रा�ीय�पाट��के��प�म��मा�यता��ा�त�होती�है।�
2. य�द�कोई�राजनी�तक�पाट��लोकसभा�के�आम�चुनाव�म��6% सीट� �जीतती�है�, तो�उसे�रा�ीय�पाट��के��प�म��मा�यता�द��जाती�है।�

ऊपर��दए�गए�कथन��म��से�कौन�सा�सही�है�/ ह��?

a) केवल�1
b) केवल�2
c) दोन��1 और�2
d) न�तो�1 और�न�ही�2
Correct Answer: A
Your Answer:
Explanation

Solution (a)

Basic Information:

य�द��न�न�ल�खत�म��से�कोई�भी�शत��पूरी�हो�जाती�है�, तो�एक�पाट��को�एक�रा�ीय�पाट��के��प�म��मा�यता�द��जाती�है:

• अगर�यह�लोकसभा�या��वधान�सभा�के�आम�चुनाव�म���कसी�भी�चार�या�अ�धक�रा�य��म��मतदान�के�6 ��तशत�वैध�वोट�हा�सल�करता
है�; और�, इसके�अलावा�, यह��कसी�भी�रा�य�या�रा�य��से�लोकसभा�म��चार�सीट� �जीतता�है�; या�
• य�द�यह�आम�चुनाव�म��लोकसभा�क��दो���तशत�सीट� �जीतता�है�; और�ये�उ�मीदवार�तीन�रा�य��से�चुने�गए�ह��; या�
• य�द�इसे�चार�रा�य��म��रा�य�पाट��के��प�म��मा�यता��ा�त�है।�

QUESTION 39.
रा�य�पाट��के��प�म��एक�राजनी�तक�दल�को�मा�यता�दे ने�के�मानदं ड�के�संदभ��म���न�न�ल�खत�कथन��पर��वचार�कर�:

1. य�द�यह�संबं�धत�रा�य�के��वधान�सभा�के�आम�चुनाव�म��2% सीट� �जीतती�है।�


2. य�द�यह�रा�य�म��एक�आम�चुनाव�म��रा�य�से�लोकसभा�के��लए�चुने�गए�कुल�वैध�मत��का�6% �ा�त�करता�है।�

ऊपर��दए�गए�कथन��म��से�कौन�सा�सही�है�/ ह��?

a) केवल�1
b) केवल�2
c) दोन��1 और�2
d) न�तो�1 और�न�ही�2
Correct Answer: D
Your Answer:
Explanation

Solution (d)

Basic Information:

य�द��न�न�म��से�कोई�भी�शत��पूरी�हो�जाती�है�, तो�एक�पाट��को�रा�य�पाट��के��प�म��मा�यता�द��जाती�है:

• य�द�यह�रा�य�के��वधान�सभा�के�आम�चुनाव�म��रा�य�म��मतदान��कए�गए�वैध�मत��का�6 ��तशत��ा�त�करता�है�; और�, इसके


अलावा�, यह�संबं�धत�रा�य�क���वधानसभा�म��2 सीट� �जीतता�है�; या�

IASbaba
Web: http://ilp.iasbaba.com/ Score:
Email: ilp@iasbaba.com 0.00 / 200
Page 136
AIPTS/ILP VETERANS-
Exam Title :
2020 TE...
Email : yadavanurag075@gmail.com
Contact : 8882839768

• य�द�यह�रा�य�के��लए�एक�आम�चुनाव�म��रा�य�से�मतदान�के��लए�वैध�मत��का�6 ��तशत�संबं�धत�रा�य�से�; और�, इसके�अलावा�, य


ह�संबं�धत�रा�य�से�लोकसभा�म��1 सीट�जीतता�है�; या�
• य�द�यह��वधान�सभा�क��तीन���तशत�सीट��पर�एक�आम�चुनाव�म��रा�य�क���वधान�सभा�या�संबं�धत�3 सीट� ��वधानसभा��म��जीतता
है�, जो�भी�अ�धक�हो�; या�
• य�द�यह���येक�25 सीट��के��लए�लोकसभा�म��1 सीट�जीतता�है�या�रा�य�के��लए�एक�आम�चुनाव�म��रा�य�से�आवं�टत��कसी�भी
�नधा��रत�अंश�को�संबं�धत�रा�य�से�जीतता�है।�

कथन��व�ेषण:

कथन�1 कथन�2

अस�य� अस�य�

रा�य�से�लोक�सभा�म��या�रा�य�क���वधान�सभा�म���मले�कुल�वैध
इस�मापदं ड�के�तहत�एक�राजनी�तक�दल�को��वधान�सभा�म��3
मत��का�8% राजनी�तक�दल�को��ा�त�करना�होता�है।�यह�शत��2
% सीट� �जीतनी�होती�ह�।�
011 म��जोड़ी�गई�थी।�

QUESTION 40.
�न�न�ल�खत�कथन�यु�म��पर��वचार�कर�:

रा�य�रा�य�पाट��का�नाम�

1. महारा��महारा�वाद��गोमांतक�( MAG)
2. असम�ऑल�इं�डया�यूनाइटे ड�डेमो�े�टक��ंट�( AUDF)
3. ह�रयाणा�इं�डयन�नेशनल�लोकदल�

ऊपर��दए�गए�कथन��म��से�कौन�सा�सही�है�/ ह��?

a) केवल�1 और�2
b) केवल�1 और�3
c) केवल�2 और�3
d) 1, 2 और�3
Correct Answer: C
Your Answer:
Explanation

Solution (c)

कथन��व�ेषण:

कथन�1 कथन�2 कथन�3

IASbaba
Web: http://ilp.iasbaba.com/ Score:
Email: ilp@iasbaba.com 0.00 / 200
Page 137
AIPTS/ILP VETERANS-
Exam Title :
2020 TE...
Email : yadavanurag075@gmail.com
Contact : 8882839768

अस�य� स�य� स�य�

महारा�वाद��गोमांतक�( MAG) गोवा ऑल�इं�डया�यूनाइटे ड�डेमो�े�टक��ंट�( AU


इं�डयन�नेशनल�लोकदल�ह�रयाणा�क��एक
क��एक�रा�य�पाट��है��जसका��तीक DF) असम�क��रा�य�पाट��है�और�इसका
रा�य�पाट��है��जसका��च�ह�च�मा�है।�
�च�ह�शेर�है।� �तीक��च�ह�ताला-चाभी�है।�

QUESTION 41.
�न�न�ल�खत�कथन��पर��वचार�कर��

1. संसद�और�रा�य��वधानसभा��के�चुनाव�के��लए���येक��े�ीय��नवा�चन��े��हेतु�अलग-अलग�सामा�य�मतदाता�सूची�होती�है।�
2. सं�वधान�ने�चुनावी�मता�धकार�के�मामले�म����येक�नाग�रक�को�समानता��दान�क��है।�
3. प�रसीमन�आयोग��ारा�जारी�आदे श�अं�तम�हो�जाते�ह��तथा��कसी�भी��यायालय�म��चुनौती�नह��द��जा�सकती�है।�

ऊपर��दए�गए�कथन��म��से�कौन�सा�सही�है�/ ह��?

a) केवल�1
b) केवल�1 और�3
c) केवल�2 और�3
d) 1, 2 और�3
Correct Answer: C
Your Answer:
Explanation

Solution (c)

Basic Information:

सं�वधान�के�भाग�XV म��अनु�छे द�324 से�329 हमारे�दे श�म��चुनाव��णाली�के�संबंध�म���न�न�ल�खत��ावधान�ह�:

• सं�वधान�(अनु�छे द�324) दे श�म���वतं��और��न�प��चुनाव�सु�न��त�करने�के��लए�एक��वतं��चुनाव�आयोग�का��ावधान�करता�है।


संसद�, रा�य��वधानसभा��, रा�प�त�के�काया�लय�और�उप-रा�प�त�के�काया�लय�के��लए��नवा�चन�क���दशा�, �नद� शन�और�आचरण
क��श���आयोग�म���न�हत�है।�
• लोकसभा�और�रा�य��वधानसभा��के�चुनाव�वय�क�मता�धकार�के�आधार�पर�होने�ह�।�1988 के�61 व��संशोधन�अ�ध�नयम�ने
मतदान�क��आयु�21 से�घटाकर�18 वष��कर�द��है।�यह�28 माच��1989 को�लागू��आ।�

कथन��व�ेषण:

कथन�1 कथन�2 कथन�3

अस�य� स�य� स�य�

IASbaba
Web: http://ilp.iasbaba.com/ Score:
Email: ilp@iasbaba.com 0.00 / 200
Page 138
AIPTS/ILP VETERANS-
Exam Title :
2020 TE...
Email : yadavanurag075@gmail.com
Contact : 8882839768

सं�वधान�घोषणा�करता�है��क��नवा�चन
केवल�धम��, जा�त�, जा�त�, �ल�ग�या�उनम��से
�े���के�प�रसीमन�या�ऐसे��नवा�चन��े��
�कसी�के�आधार�पर��कसी�����को
संसद�और�रा�य��वधानसभा��के�चुनाव को�सीट��के�आवंटन�से�संबं�धत��कसी�भी
मतदाता�सूची�म��शा�मल�करने�के��लए
के��लए���येक��े�ीय��नवा�चन��े��के कानून�क��वैधता�पर��कसी�भी��यायालय
अयो�य�नह��हो�सकता�है।�इस��कार�, सं�व
�लए�केवल�एक�समान�मतदाता�सूची�होनी म�����नह��उठाया�जा�सकता�है।
धान�ने���येक�नाग�रक�को�चुनावी
चा�हए।� नतीजतन�, प�रसीमन�आयोग��ारा�जारी
मता�धकार�के�मामले�म��समानता��दान�क�
आदे श�अं�तम�हो�जाते�ह��और��कसी�भी
है।�
�यायालय�म��चुनौती�नह��द��जा�सकती।�

QUESTION 42.
भारत�के�चुनावी�तं��( election machinery) के�संदभ��म���न�न�ल�खत�कथन��पर��वचार�कर�:

1. भारत�के�रा�प�त�मु�य�चुनाव�आयु��और�चुनाव�आयु���क���नयु���करते�ह�।�
2. मु�य��नवा�चन�अ�धकारी�क���नयु���भारत�के�चुनाव�आयोग��ारा�संबं�धत�रा�य�के�रा�यपाल�के�परामश��से�क��जाती�है।�
3. �जला��नवा�चन�अ�धकारी�क���नयु���रा�य�के�मु�य��नवा�चन�अ�धकारी��ारा�क��जाती�है।�

ऊपर��दए�गए�कथन��म��से�कौन�सा�सही�है�/ ह��?

a) केवल�1
b) केवल�1 और�3
c) केवल�2
d) 1, 2 और�3
Correct Answer: A
Your Answer:
Explanation

Solution (a)

Basic Information:

• भारत�के�चुनावी�तं��म��भारतीय��नवा�चन�आयोग�( ECI), मु�य��नवा�चन�अ�धकारी�( CEO), �जला��नवा�चन�अ�धकारी�( DEO),


�नवा�चन�अ�धकारी�( RO), �नवा�चक�र�ज���करण�अ�धकारी�( ERO), पीठासीन�अ�धकारी�और�पय�वे�क�शा�मल�ह�।�
• भारत�के�सं�वधान�के�अनु�छे द�324 के�तहत�, भारतीय�चुनाव�आयोग�लोकसभा�और�रा�य��वधानसभा��म��चुनाव�कराने�के��लए
अधी�ण�, �नद� शन�और��नयं�ण�क��श���के�साथ��न�हत�है।�
• �कसी�रा�य�/ संघ�रा�य��े��के�मु�य��नवा�चन�अ�धकारी�को�चुनाव�आयोग�के�सम��अधी�ण�, �नद� शन�और��नयं�ण�के�अधीन�रा�य
/ संघ�रा�य��े��म��चुनाव�काय��क���नगरानी�के��लए�अ�धकृत��कया�जाता�है।�
• मु�य��नवा�चन�अ�धकारी�के�अधी�ण�, �नद� शन�और��नयं�ण�के�अधीन�, �जला��नवा�चन�अ�धकारी�एक��जले�के�चुनाव�काय��का
पय�वे�ण�करता�है।�
• भारत��नवा�चन�आयोग�रा�य�सरकार�/ संघ�रा�य��े���शासन�के�परामश��से���येक��वधानसभा�और�संसद�य��नवा�चन��े���के��लए
सरकार�के�एक�अ�धकारी�या��थानीय�अ�धकारी�को��रट�न�ग�अ�धकारी�के��प�म��ना�मत�या�ना�मत�करता�है।�
• भारतीय�चुनाव�आयोग�, रा�य�/ क��शा�सत��दे श�सरकार�के�परामश��से�, सरकार�के�एक�अ�धकारी�या��थानीय�अ�धका�रय��को
�नवा�चक�पंजीकरण�अ�धकारी��नयु��करता�है।�
• �जला��नवा�चन�अ�धकारी�पीठासीन�अ�धका�रय��और�मतदान�अ�धका�रय��क���नयु���करता�है।�क���शा�सत��दे श��के�मामले�म��, ऐसी
�नयु��यां��रट�न�ग�ऑ�फसर��ारा�क��जाती�ह�।�
• भारतीय��नवा�चन�आयोग�संसद�य�और��वधानसभा��नवा�चन��े���के��लए�सरकार�के�अ�धका�रय��को�पय�वे�क��के��प�म��ना�मत
करता�है।�

कथन��व�ेषण:

IASbaba
Web: http://ilp.iasbaba.com/ Score:
Email: ilp@iasbaba.com 0.00 / 200
Page 139
AIPTS/ILP VETERANS-
Exam Title :
2020 TE...
Email : yadavanurag075@gmail.com
Contact : 8882839768

कथन�1 कथन�2 कथन�3

स�य� अस�य� अस�य�

वत�मान�म��, भारतीय�चुनाव�आयोग�एक
भारत��नवा�चन�आयोग�उस�रा�य�सरकार�/
तीन�सद�यीय��नकाय�है�, �जसम��एक भारत��नवा�चन�आयोग�रा�य�सरकार�के
क���शा�सत��दे श��शासन�के�परामश��से
मु�य�चुनाव�आयु��और�दो�चुनाव�आयु� परामश��से�रा�य�सरकार�के�एक�अ�धकारी
रा�य�/ क���शा�सत��दे श�के�एक�अ�धकारी
होते�ह�।�भारत�के�रा�प�त�मु�य�चुनाव को��जला��नवा�चन�अ�धकारी�ना�मत�या
को�मु�य��नवा�चन�अ�धकारी�के��प�म�
आयु��और�चुनाव�आयु���क���नयु�� �न�द���करता�है।�
ना�मत�या���त��त�करता�है।�
करते�ह�।�

QUESTION 43.
�न�न�ल�खत�कथन��पर��वचार�कर�:

1. �रट�न�ग�अ�धकारी�( Returning Officer) संबं�धत�संसद�य�या��वधानसभा��े��म��चुनाव�के�संचालन�के��लए�उ�रदायी�होता�है।�


2. पीठासीन�अ�धकारी�( Presiding Officer) एक�संसद�य�/ �वधानसभा��े��के��लए�मतदाता�सूची�तैयार�करने�के��लए�उ�रदायी
होता�है।�

ऊपर��दए�गए�कथन��म��से�कौन�सा�सही�है�/ ह��?

a) केवल�1
b) केवल�2
c) दोन��1 और�2
d) न�तो�1 और�न�ही�2
Correct Answer: A
Your Answer:
Explanation

Solution (a)

Basic Information:

• भारतीय��नवा�चन�आयोग�रा�य�सरकार�/ संघ�रा�य��े���शासन�के�परामश��से���येक��वधानसभा�और�संसद�य��नवा�चन��े���के��लए
सरकार�के�एक�अ�धकारी�या��थानीय�अ�धकारी�को��रट�न�ग�अ�धकारी�के��प�म��ना�मत�या�ना�मत�करता�है।�
• �नवा�चक�र�ज���करण�अ�धकारी�एक�संसद�य�/ �वधानसभा��े��के��लए�मतदाता�सूची�तैयार�करने�के��लए�उ�रदायी�होता�है।�
• इसके�अलावा�, भारतीय��नवा�चन�आयोग�भी�मतदाता�सूची�तैयार�करने�/ संशोधन�के�मामले�म��अपने�काय��के��दश�न�म���नवा�चक
र�ज���करण�अ�धकारी�क��सहायता�के��लए�एक�या�एक�से�अ�धक�सहायक��नवा�चक�र�ज���करण�अ�धका�रय��क���नयु���करता�है।
• �जला��नवा�चन�अ�धकारी�पीठासीन�अ�धका�रय��और�मतदान�अ�धका�रय��क���नयु���करता�है।�क���शा�सत��दे श��के�मामले�म��, ऐसी
�नयु��यां��रट�न�ग�ऑ�फसर��ारा�क��जाती�ह�।�

कथन��व�ेषण:

कथन�1 कथन�2

IASbaba
Web: http://ilp.iasbaba.com/ Score:
Email: ilp@iasbaba.com 0.00 / 200
Page 140
AIPTS/ILP VETERANS-
Exam Title :
2020 TE...
Email : yadavanurag075@gmail.com
Contact : 8882839768

स�य� अस�य�

वह�संसद�य�या��वधानसभा��े��म��चुनाव�के�संचालन�के��लए
मतदान�अ�धका�रय��क��सहायता�से�पीठासीन�अ�धकारी�मतदान
�ज�मेदार�है।�इसके�अलावा�, भारत�का�चुनाव�आयोग��रट�न�ग
क���पर�मतदान�करवाता�है।��नवा�चक�र�ज���करण�अ�धकारी
ऑ�फसर�क��सहायता�के��लए��वधानसभा�और�संसद�य��े���म�
एक�संसद�य�/ �वधानसभा��े��के��लए�मतदाता�सूची�तैयार�करने
से���येक�के��लए�एक�या�एक�से�अ�धक�सहायक��रट�न�ग
के��लए�उ�रदायी�होता�है।�
अ�धकारी��नयु��करता�है�

QUESTION 44.
चुनाव����या�के�बारे�म���न�न�ल�खत�कथन��पर��वचार�कर�:

1. भारतीय�चुनाव�आयोग��ारा�चुनाव�काय��म�क��घोषणा�के�तुरंत�बाद�आदश��आचार�सं�हता�लागू�हो�जाती�है।�
2. संबं�धत�पीठासीन�अ�धकारी�अपने��नवा��चत��नवा�चन��े��के��लए�प�रणाम�घो�षत�करता�है।�

ऊपर��दए�गए�कथन��म��से�कौन�सा�सही�है�/ ह��?

a) केवल�1
b) केवल�2
c) दोन��1 और�2
d) न�तो�1 और�न�ही�2
Correct Answer: A
Your Answer:
Explanation

Solution (a)

Basic Information:

जब�पांच�वष��क��सीमा�समा�त�होती�है�, या��वधा�यका�को�भंग�कर��दया�जाता�है�और�नए�चुनाव��को�आरंभ��कया�जाता�है�, तो�चुनाव�आयोग


चुनाव�कराने�के��लए�तं��को�लागू�करता�है।�

• सं�वधान�म��कहा�गया�है��क��वघ�टत�लोकसभा�के�अं�तम�स��और�नए�सदन�के�वापस�आने�के�बीच�छह�महीने�से�अ�धक�का�समय�नह�
हो�सकता�है�, इस�लए�चुनाव�पहले�ही�संप��हो�जाना�चा�हए।�
• चुनाव��क��औपचा�रक����या�एक�सदन�के�सद�य��के��नवा�चन�के��लए�मतदाता�को�सू�चत�करने�वाली�अ�धसूचना�या�अ�धसूचना�
के�साथ�शु��होती�है।�
• अ�धसूचना�जारी�होते�ही�, उ�मीदवार�उन��नवा�चन��े���म��अपना�नामांकन�दा�खल�करना�शु��कर�सकते�ह��, जहाँ�से�वे�चुनाव�लड़ना
चाहते�ह�।�इनक��जांच�संबं�धत��नवा�चन��े��के��रट�न�ग�अ�धकारी��ारा�क��जाती�है।�
• वैध��प�से�नामां�कत�उ�मीदवार�जांच�क��तारीख�से�दो��दन��के�भीतर���त�पधा��से�हट�सकते�ह��
• चुनाव�क��वा�त�वक�तारीख�से�पहले�राजनी�तक�चुनाव��चार�के��लए�उ�मीदवार��को�कम�से�कम�दो�स�ताह��मलते�ह�।�

कथन��व�ेषण:

कथन�1 कथन�2

IASbaba
Web: http://ilp.iasbaba.com/ Score:
Email: ilp@iasbaba.com 0.00 / 200
Page 141
AIPTS/ILP VETERANS-
Exam Title :
2020 TE...
Email : yadavanurag075@gmail.com
Contact : 8882839768

स�य� अस�य�

औपचा�रक����या�म��ग�त��नधा��रत�होने�से�कुछ�स�ताह�पहले
आयोग�आमतौर�पर�एक��मुख��ेस�कॉ���स�म��चुनाव��के मतगणना�के��लए�एक�अलग�तारीख�तय�क��जाती�है�तथा
काय��म��क��घोषणा�करता�है।�उ�मीदवार��और�राजनी�तक संबं�धत��नवा�चन�अ�धकारी��ारा���येक��नवा�चन��े��के��लए
दल��के�माग�दश�न�के��लए�आदश��आचार�सं�हता�ऐसी�घोषणा�के प�रणाम�घो�षत��कए�जाते�ह�।�
तुरंत�बाद�लागू�हो�जाती�है�

QUESTION 45.
चुनाव�लड़ने�वाले�उ�मीदवार��क��शपथ�या�पु���के�संदभ��म���न�न�ल�खत�कथन��पर��वचार�कर�:

1. उ�मीदवार�अपनी�उ�मीदवारी�क��जांच�क���त�थ�के�बाद�शपथ�या�पु���कर�सकता�है।�
2. एक�उ�मीदवार�को��नवा�चन�अ�धकारी�और��नवा�चन��े��के��लए�सहायक��नवा�चन�अ�धकारी�के�सम��मु�य��प�से�शपथ�या�पु��
करनी�होती�है।�
3. य�द��कसी�उ�मीदवार�को�जेल�म��या��नरोधा�मक��हरासत�म��रखा�जाता�है�, तो�जेल��के�अधी�क�को�शपथ��दलाने�के��लए�अ�धकृत
�कया�जाता�है।�

ऊपर��दए�गए�कथन��म��से�कौन�सा�सही�है�/ ह��?

a) केवल�1 और�2
b) केवल�1 और�3
c) केवल�2 और�3
d) 1, 2 और�3
Correct Answer: C
Your Answer:
Explanation

Solution (c)

Basic Information:

• चुनाव�आयोग��ारा�अ�धकृत�अ�धकारी�के�सम��उ�मीदवार�को�शपथ�या���त�ा�लेना�आव�यक�है।�
• �कसी��वशेष�चुनाव�के��लए�, अ�धकृत�����मु�य��प�से��नवा�चन�अ�धकारी�और��नवा�चन��े��के��लए�सहायक��नवा�चन�अ�धकारी
होते�ह�।�
• �कसी��वशेष�चुनाव�के��लए�, अ�धकृत�����मु�य��प�से��नवा�चन�अ�धकारी�और��नवा�चन��े��के��लए�सहायक��नवा�चन�अ�धकारी
होते�ह�।��कसी�अ�यथ��को�जेल�म��या��नवारक�बंद��के�तहत�जेल�म��रखने�के�मामले�म��, जेल�के�अधी�क�या��नरोध��श�वर�के�कमांड�ट
�जसम��वह�सी�मत�है�या�ऐसी�नजरबंद��के�तहत�शपथ�लेने�के��लए�अ�धकृत�है।�
• �कसी�उ�मीदवार�को�बीमारी�या��कसी�अ�य�कारण�से�अ�पताल�या�अ�य�जगह��पर��ब�तर�तक�सी�मत�रहने�के�मामले�म��, अ�पताल�के
�भारी��च�क�सा�अधी�क�या�उस�पर�उप��थत��च�क�सा�अ�धकारी�समान��प�से�अ�धकृत�होते�ह�।�
• य�द�कोई�उ�मीदवार�भारत�से�बाहर�है�, तो�भारतीय�राज�त�या�उ�चायु��या�उसके��ारा�अ�धकृत�राजन�यक�कांसुलर�भी�शपथ�/
��त�ा��दला�सकते�ह�।�
• उ�मीदवार�, ����को�अपने�नामांकन�प����तुत�करने�के�तुरंत�बाद�शपथ�या���त�ा�लेने�क��आव�यकता�होती�है�।�

QUESTION 46.
�न�न�ल�खत�म��से�कौन�चुनाव�और�मतदान��वहार�म��मी�डया�क��सकारा�मक�भू�मका�नह��है�?

a) सूचना��सार�

IASbaba
Web: http://ilp.iasbaba.com/ Score:
Email: ilp@iasbaba.com 0.00 / 200
Page 142
AIPTS/ILP VETERANS-
Exam Title :
2020 TE...
Email : yadavanurag075@gmail.com
Contact : 8882839768

b) पेड��यूज�
c) आदश��आचार�सं�हता�और�अ�य�कानून��का��वत�न�
d) मतदाता��श�ा�और�भागीदारी�
Correct Answer: B
Your Answer:
Explanation

Solution (b)

Basic Information:

• चुनाव�के�संबंध�म��सूचना��सार�, �वशेष��प�से�चुनाव�क�����या�के�दौरान�सभी��हतधारक��के��लए�अ�यंत�मह�वपूण��है।�चुनाव�क�
घोषणा�से�लेकर�नामांकन�, छानबीन�, अ�भयान�, सुर�ा��व�था�, मतदान�, मतगणना�, प�रणाम��क��घोषणा�, आ�द�, इन�सभी�म�
�ापक��सार�क��आव�यकता�होती�है।�
• यह�एमसीसी�के�उ�लंघन�जैसे��क��वभाजनकारी�या�घृणा�पद�भाषण��या�मतदाता��को��भा�वत�करने�के�उ�े �य�से�चलाए�गए
अ�भयान��म��अस�या�पत�आरोप��को�भी�उजागर�कर�सकता�है।�मी�डया��ारा�सू�चत�क��गई��ह�सा�को�चुनाव�आयोग��ारा�औपचा�रक
�शकायत��के�समान�डील��कया�जाता�है।�
• मतदाता�जाग�कता�और�भागीदारी�के�मह�वपूण���े��म��मी�डया�से�ब�त�बड़ी�और���तब��भागीदारी�क��गुंजाइश�है।�यह�चुनाव
आयोग�- मी�डया�संबंध�के�सबसे�आशाजनक��े���म��से�एक�है।�

QUESTION 47.
�न�न�ल�खत�म��से�कौन�सा�कथन�जन��त�न�ध�व�अ�ध�नयम�, 1951 म���ावधान�ह��/ ह�:

1. इसम��केवल�संसद�और�रा�य��वधानसभा��क��सद�यता�के��लए�यो�यता�के��ावधान�शा�मल�ह��
2. यह�लोक�सभा�तथा�रा�य��क���वधानसभा��और��वधान�प�रषद��म��सीट��के�आवंटन�का��ावधान�करती�है।�
3. इसम�����आचरण�और�चुनावी�अपराध�शा�मल�ह�।�

सही�कूट�का�चयन�कर�:

a) केवल�1
b) केवल�2 और�3
c) केवल�3
d) 1, 2 और�3
Correct Answer: C
Your Answer:
Explanation

Solution (c)

Basic Information:

• जन���त�न�ध�व�अ�ध�नयम�, 1950 म��चुनाव�से�संबं�धत�सभी��ावधान�शा�मल�नह��थे�, ब��क�केवल�सीट��के�आवंटन�के��लए�और


�नवा�चन��े���के�प�रसीमन�के��लए�, लोक�सभा�और�रा�य��क���वधानसभा��के��नवा�चन�के��लए�, इस�तरह�के�चुनाव�के��लए
मतदाता�क��यो�यता�और�मतदाता�सूची�क��तैयारी�पर��ावधान��कया�गया�था।�
• �ापक��ावधान��को��दान�करने�के��लए�, जन���त�न�ध�व�अ�ध�नयम�, 1951 अ�ध�नय�मत��कया�गया�था।�

मोटे �तौर�पर�, जन���त�न�ध�व�अ�ध�नयम�, 1950 म���न�न�ल�खत�चुनावी�मामल��से�संबं�धत��ावधान�ह�:

• आम�चुनाव�क��अ�धसूचना�
• चुनाव�के�संचालन�के��लए��शास�नक�मशीनरी�
• राजनी�तक�दल��का�पंजीकरण�

IASbaba
Web: http://ilp.iasbaba.com/ Score:
Email: ilp@iasbaba.com 0.00 / 200
Page 143
AIPTS/ILP VETERANS-
Exam Title :
2020 TE...
Email : yadavanurag075@gmail.com
Contact : 8882839768

• चुनाव�का�संचालन�
• मा�यता��ा�त�राजनी�तक�दल��के�उ�मीदवार��को�कुछ�साम�ी�क��मु�त�आपू�त��
• चुनाव�को�लेकर��ववाद�
• ���आचरण�और�चुनावी�अपराध�

कथन��व�ेषण:

कथन�1 कथन�2 कथन�3

अस�य� अस�य� स�य�

इसम��संसद�और�रा�य��वधानसभा��क�
यह�जन���त�न�ध�व�कानून�, 1950 से इसम�����आचरण�और�चुनावी�अपराध�
सद�यता�के��लए�यो�यता�और�अयो�यता
एक��ावधान�है� क���ापक�कवरेज�है।�
दोन��शा�मल�ह�।�

QUESTION 48.
�दनेश�गो�वामी�स�म�त��कससे�संबं�धत�थी:

a) दलबदल��वरोधी��ावधान�
b) चुनावी�सुधार�
c) मौ�लक�अ�धकार��से��नद� शा�मक��स�ांत��को��ाथ�मकता।�
d) मौ�लक�कत���
Correct Answer: B
Your Answer:
Explanation

Solution (b)

Basic Information:

1990 म��, वी�पी��स�ह�के�नेतृ�व�वाली�रा�ीय�मोचा��सरकार�ने�त�कालीन�कानून�मं�ी��दनेश�गो�वामी�क��अ�य�ता�म��चुनावी�सुधार��के��लए


एक�स�म�त��नयु��क��थी।�

स�म�त�को�चुनावी��णाली�का��व�तार�से�अ�ययन�करने�और�इसके�भीतर�क�मय��को��र�करने�के�उपाय�सुझाने�के��लए�कहा�गया�था।�

�दनेश�गो�वामी�स�म�त�क��कुछ��सफा�रश�:

• उ�मीदवार��के�नाम��क��सूची�
• रा�ीय�स�मान�अ�ध�नयम�का�अपमान�करने�के��लए�अयो�यता�
• शराब�क���ब���पर���तबंध�
• �कसी�संसद�य�या��वधानसभा��े��म��उ�मीदवार�का�नामांकन���तावक�के��प�म���नवा�चन��े��के�10 पंजीकृत�मतदाता���ारा��लया
जाना�चा�हए�, य�द�उ�मीदवार��कसी�मा�यता��ा�त�राजनी�तक�पाट���ारा��ायो�जत�नह��है।��कसी�मा�यता��ा�त�राजनी�तक�दल��ारा
�ायो�जत�उ�मीदवार�के�मामले�म��, केवल�एक���तावक�क��आव�यकता�होती�है।�
• एक�उ�मीदवार�क��मृ�यु�के�मामले�म��चुनाव�वा�त�वक�मतदान�से�पहले�एक�चुनाव�लड़ने�वाले�उ�मीदवार�क��मृ�यु�पर���तवाद�नह�
होगा।�हालां�क�, य�द�मृतक�उ�मीदवार��कसी�मा�यता��ा�त�राजनी�तक�दल�से�संबं�धत�है�, तो�संबं�धत�पाट��को�सात��दन��के�भीतर
�कसी�अ�य�उ�मीदवार�को���ता�वत�करने�का��वक�प��दया�जाएगा।�

IASbaba
Web: http://ilp.iasbaba.com/ Score:
Email: ilp@iasbaba.com 0.00 / 200
Page 144
AIPTS/ILP VETERANS-
Exam Title :
2020 TE...
Email : yadavanurag075@gmail.com
Contact : 8882839768

• संसद�के��कसी�भी�सदन�या�रा�य��वधानमंडल�म���र���होने�के�छह�महीने�के�भीतर�उपचुनाव�होने�ह�।�
• मतदान��दवस�पर�कम�चा�रय��को�अवकाश�दे ना�

QUESTION 49.
‘ ए��जट�पोल�’ के�संबंध�म���न�न�ल�खत�म��से�कौन�सा�कथन�सही�है�?

a) ‘ ए��जट�पोल�’ एक�ऐसा�श�द�है��जसका�इ�तेमाल�मतदाता��के�चुनाव�के�बाद�के�सव��ण�को�सू�चत�करने�के��लए��कया�जाता�है�, �ज
सके�प��म��उ�ह�ने�अपने�मता�धकार�का��योग��कया�है�
b) ‘ ए��जट�पोल�’ और�‘ ओ�प�नयन�पोल�’ एक�और�समान�ह��
c) ‘ ए��जट�पोल�’ वह�उपकरण�है��जसके�मा�यम�से�मतदान�के�प�रणाम��क��सट�क�भ�व�यवाणी�क��जा�सकती�है�
d) ‘ ए��जट�पोल�’ एक��शास�नक�उपकरण�है��जसे�मु�य�चुनाव�आयु��ने���त�पण�( impersonation) को�रोकने�के��लए�हाल�ही
म��बनाया�है�
Correct Answer: A
Your Answer:
Explanation

Solution (a)

Basic Information:

• " ए��जट-पोल�" एक�चुनाव�सव��ण�है��क�चुनाव�म��मतदाता��ने�कैसे�वोट��दया�है�या��कसी�चुनाव�म���कसी�राजनी�तक�दल�या


उ�मीदवार�क��पहचान�के�संबंध�म��सभी�मतदाता��ने�कैसा��दश�न��कया�है।�
• जन���त�न�ध�व�अ�ध�नयम�, 1951 क��धारा�126A: इसम��ए��जट�पोल�के�संचालन�पर�रोक�लगाई�गई�है�और�इसम��उ�ले�खत
अव�ध�के�दौरान�उनके�प�रणाम�का��सार�, अथा�त�, सभी�रा�य��और�क���शा�सत��दे श��म��पहले�चरण�म��मतदान�शु��होने�के�समय�से
आधे�घंटे�पहले�और�अं�तम�चरण�के��लए�मतदान�बंद�होने�के��नधा��रत�समय�से�आधे�घंटे�बाद�ही��कया�जा�सकता�है।�
• 2009 के�एक��ावधान�के�अनुसार�, ए��जट�पोल�आयो�जत�करना�और�ए��जट�पोल�के�नतीजे��का�शत�करना�लोकसभा�और�रा�य
�वधानसभा��के�चुनाव�के�दौरान��न�ष��होगा।�इस��कार�, कोई�भी������कसी�भी�ए��जट�पोल�का�संचालन�नह��करेगा�और����ट
या�इले��ॉ�नक�मी�डया�के�मा�यम�से��का�शत�या��चा�रत�करेगा�या��कसी�अ�य�तरीके�से��चा�रत�करेगा�, इस�संबंध�म��चुनाव�आयोग
�ारा�अ�धसू�चत�अव�ध�के�दौरान�ही��कसी�भी�ए��जट�पोल�का�प�रणाम�जारी��कया�जा�सकता�है।�

QUESTION 50.
दल-बदल�के�आधार�पर�अयो�यता�के��खलाफ�अपवाद��के�बारे�म���न�न�ल�खत�कथन��पर��वचार�कर�:

1. य�द�कोई�सद�य��कसी�अ�य�पाट��के�साथ�पाट��के��वलय�के�प�रणाम�व�प�अपनी�पाट��से�बाहर�जाता�है।�
2. �वधायक�दल�के�एक��तहाई�सद�य���ारा��वभाजन�के�मामले�म�।�

उपरो��कथन��म��से�कौन�सा�गलत�है�/ ह��?

a) केवल�1
b) केवल�2
c) दोन��1 और�2
d) न�तो�1 और�न�ही�2
Correct Answer: B
Your Answer:
Explanation

Solution (b)

Basic Information:

IASbaba
Web: http://ilp.iasbaba.com/ Score:
Email: ilp@iasbaba.com 0.00 / 200
Page 145
AIPTS/ILP VETERANS-
Exam Title :
2020 TE...
Email : yadavanurag075@gmail.com
Contact : 8882839768

दलबदल�के�आधार�पर�अयो�यता��न�न�ल�खत�दो�मामल��म��लागू�नह��होती�है:

• य�द�कोई�सद�य��कसी�अ�य�पाट��के�साथ�पाट��के��वलय�के�प�रणाम�व�प�अपनी�पाट��से�बाहर�जाता�है।�एक��वलय�तब�होता�है�जब
पाट��के�दो-�तहाई�सद�य�इस�तरह�के��वलय�के��लए�सहमत�हो�जाते�ह�।�
• य�द�कोई�सद�य�, सदन�के�पीठासीन�अ�धकारी�के��प�म��चुने�जाने�के�बाद�, �वे�छा�से�अपनी�पाट��क��सद�यता�छोड़�दे ता�है�या�उस
काया�लय�को�समा�त�करने�के�बाद��फर�से�इसम��शा�मल�हो�जाता�है।�यह�छू ट�इस�काया�लय�क��ग�रमा�और��न�प�ता�को�दे खते��ए
�दान�क��गई�है।�
• सदन�के�पीठासीन�अ�धकारी��ारा�दलबदल�से�उ�प��अयो�यता�के�बारे�म��कोई�भी����तय��कया�जाता�है।�
• एक�सदन�के�पीठासीन�अ�धकारी�को�दसव��अनुसूची�के��ावधान��को��भावी�करने�के��लए��नयम�बनाने�का�अ�धकार�है।�ऐसे�सभी
�नयम��को�30 �दन��के�अंदर�सदन�के�सम��रखा�जाना�चा�हए।�

कथन��व�ेषण:

कथन�1 कथन�2

स�य� अस�य�

�वधायक�दल�के�एक-�तहाई�सद�य���ारा��वभाजन�के�मामले�म�
अयो�यता�से�छू ट�के�संबंध�म��दसव��अनुसूची�के��ावधान�को�200
एक��वलय�तब�होता�है�जब�पाट��के�दो-�तहाई�सद�य�इस�तरह
3 के�91 व��संशोधन�अ�ध�नयम��ारा�हटा��दया�गया�है।�इसका
के��वलय�के��लए�सहमत�हो�जाते�ह�।�
अथ��है��क�दो�षय��को��वभाजन�के�आधार�पर�अ�धक�सुर�ा�नह�
है।�

QUESTION 51.
भारत�म��चुनाव�आयोग�के�संबंध�म��, नीचे��दए�गए�कथन��म��से�कौन�सा�/ से�सही�है�/ ह��?

1. यह�एक�अ�खल�भारतीय��नकाय�है�, �जसे�सीधे�भारत�के�सं�वधान��ारा��था�पत��कया�गया�है।�
2. यह�संसद�, रा�य��म��रा�य��वधानसभा��, पंचायत��और�नगरपा�लका��के��लए��वतं��और��न�प��चुनाव�सु�न��त�करता�है।�

सही��वक�प�चुन�:

a) केवल�1
b) केवल�2
c) दोन��1 और�2
d) न�तो�1 और�न�ही�2
Correct Answer: A
Your Answer:
Explanation

Solution (a)

कथन��व�ेषण:

कथन�1 कथन�2

IASbaba
Web: http://ilp.iasbaba.com/ Score:
Email: ilp@iasbaba.com 0.00 / 200
Page 146
AIPTS/ILP VETERANS-
Exam Title :
2020 TE...
Email : yadavanurag075@gmail.com
Contact : 8882839768

स�य� अस�य�

सं�वधान�के�अनु�छे द�324 म��यह��ावधान�है��क�संसद�, रा�य


�वधानसभा��, भारत�के�रा�प�त�के�काया�लय�और�भारत�के
चुनाव�आयोग�दे श�के��वतं��और��न�प��चुनाव�सु�न��त�करने उपरा�प�त�के�चुनाव�के��लए��नवा�चन�क���दशा�और��नयं�ण
के��लए�सीधे�भारत�के�सं�वधान��ारा��था�पत�एक��थायी�और चुनाव�आयोग�म���न�हत�होगा।�
एक��वतं���नकाय�है।�
यहां�यह��यान�रखना�चा�हए��क�चुनाव�आयोग�का�रा�य��क�
यह�इस�अथ��म��भी�एक�अ�खल�भारतीय��नकाय�है��क�यह�क�� पंचायत��और�नगरपा�लका��के�चुनाव��से�कोई�संबंध�नह��है।�
सरकार�और�रा�य�सरकार��दोन��के��लए�समान�है।�
इसके��लए�, भारत�का�सं�वधान�एक�अलग�रा�य�चुनाव�आयोग
�दान�करता�है।�

QUESTION 52.
भाषाई�अ�पसं�यक��के��लए��वशेष�अ�धकारी�( Special Officer for Linguistic Minorities) के�बारे�म���न�न�ल�खत�म��से�कौन
सा�कथन�सही�है�/ ह��?

1. वह�रा�प�त��ारा��नयु���कया�जाता�है।�
2. सं�वधान�भाषाई�अ�पसं�यक��के��लए��वशेष�अ�धकारी�क��यो�यता�, काय�काल�, वेतन�और�भ�े�, सेवा�शत��और�हटाने�क�����या
को��न�द���नह��करता�है।�

उ�चत��वक�प�चुन�:

a) केवल�1
b) केवल�2
c) दोन��1 और�2
d) न�तो�1 और�न�ही�2
Correct Answer: C
Your Answer:
Explanation

Solution (c)

Basic information:

मूल��प�से�, भारत�के�सं�वधान�ने�भाषाई�अ�पसं�यक��के��लए��वशेष�अ�धकारी�के�संबंध�म��कोई��ावधान�नह���कया�था।�

1956 के�सातव��संवैधा�नक�संशोधन�अ�ध�नयम�ने�सं�वधान�के�भाग�XVII म��एक�नया�अनु�छे द�350-b जोड़ा�, �जसम���न�न�ल�खत


�ावधान�शा�मल�थे�-

• भाषाई�अ�पसं�यक��के��लए�एक��वशेष�अ�धकारी�होना�चा�हए।�उ�ह��भारत�के�रा�प�त��ारा��नयु���कया�जाना�है।�
• सं�वधान�के�तहत�भाषाई�अ�पसं�यक��के��लए��दान��कए�गए�सुर�ा�उपाय��से�संबं�धत�सभी�मामल��क��जांच�करना��वशेष�अ�धकारी
का�कत���होगा।�
• वह�उन�मामल��पर�रा�प�त�को��रपोट� �करेगा�जैसे��क�रा�प�त��नद� श�कर�सकते�ह�।�रा�प�त�को�ऐसी�सभी��रपोट��को�संसद�के���येक
सदन�के�सम��रखना�चा�हए�और�संबं�धत�रा�य��क��सरकार��को�भेजना�चा�हए।�

यहां�यह��यान��दया�जाना�चा�हए��क�सं�वधान�भाषाई�अ�पसं�यक��के��लए��वशेष�अ�धकारी�क��यो�यता�, काय�काल�, वेतन�और�भ�े�, सेवा


शत��और�हटाने�क�����या�को��न�द���नह��करता�है।�

IASbaba
Web: http://ilp.iasbaba.com/ Score:
Email: ilp@iasbaba.com 0.00 / 200
Page 147
AIPTS/ILP VETERANS-
Exam Title :
2020 TE...
Email : yadavanurag075@gmail.com
Contact : 8882839768

कथन��व�ेषण:

कथन�1 कथन�2

स�य� अस�य�

सं�वधान�भाषाई�अ�पसं�यक��के��लए��वशेष�अ�धकारी�क�
भाषाई�अ�पसं�यक��के��लए��वशेष�अ�धकारी�क���नयु��
यो�यता�, काय�काल�, वेतन�और�भ�े�, सेवा�शत��और�हटाने�क�
भारत�के�रा�प�त��ारा�क��जाती�है।�
���या�को��न�द���नह��करता�है।�

QUESTION 53.
मु�य�चुनाव�आयु��( CEC) और�अ�य�चुनाव�आयु���( EC) के�बारे�म���न�न�ल�खत�कथन��पर��वचार�कर�।�सही�कथन�को�पहचान�:

a) वे�समान���थ�त�का�आनंद�लेते�ह��तथा�भारत�के�सव��च��यायालय�के��यायाधीश��के�समान�वेतन�और�भ�े��ा�त�करते�ह�।�
b) CEC और�अ�य�EC के�बीच�राय�के�अंतर�के�मामले�म��, मामला�CEC �ारा�तय��कया�जाता�है।�
c) उ�ह��अपने�काया�लय�से�उसी�तरीके�से�और�उसी�आधार�पर�हटाया�जा�सकता�है�, जैसे�सव��च��यायालय�के��यायाधीश�को।�
d) वे�रा�प�त�के��सादपय�त�अपने�पद�पर�बने�रहते�ह�।�
Correct Answer: A
Your Answer:
Explanation

Solution (a)

कथन��व�ेषण:

�वक�प�( a) �वक�प�( b) �वक�प�( c) �वक�प�( d)

स�य� अस�य� अस�य� अस�य�

IASbaba
Web: http://ilp.iasbaba.com/ Score:
Email: ilp@iasbaba.com 0.00 / 200
Page 148
AIPTS/ILP VETERANS-
Exam Title :
2020 TE...
Email : yadavanurag075@gmail.com
Contact : 8882839768

मु�य�चुनाव�आयु��को
काय�काल�क��सुर�ा��दान�क� CEC को�रा�प�त��ारा��वशेष
जाती�है।� ब�मत�के�साथ�संसद�के�दोन�
सदन���ारा�उस��भाव�, या�तो
मु�य�चुनाव�आयु��और�दो
उ�ह��सु�ीम�कोट� �के��यायाधीश ���वहार�या�अ�मता�के�आधार
अ�य�चुनाव�आयु���के�पास मु�य�चुनाव�आयु��और�/ या
के�समान�उसी�तरह�और�उसी पर�पा�रत��कए�गए�एक���ताव
समान�अ�धकार�होते�ह��तथा दो�अ�य�चुनाव�आयु���के�बीच
आधार�पर�ही�उनके�काया�लय�से के�आधार�पर�हटाया�जा�सकता
समान�वेतन�, भ�े�और�अ�य मतभेद�के�मामले�म��, आयोग
हटाया�जा�सकता�है।� है।�
अनुलाभ��ा�त�करते�ह��, जो��क �ारा�ब�मत�से�मामला�तय
सव��च��यायालय�के��यायाधीश �कया�जाता�है।� हालां�क�, मु�य�चुनाव�आयु� इस��कार�, वह�रा�प�त�के
के�समान�होते�ह�।� क���सफा�रश�के��बना��कसी�भी �सादपय�त�अपना�पद�नह�
अ�य�चुनाव�आयु��या�एक संभालता�है�, हालां�क�वह�उसके
�े�ीय�आयु��को�पद�से�हटाया �ारा��नयु���कया�जाता�है।�
नह��जा�सकता�है।�

QUESTION 54.
�न�न�ल�खत�कथन��पर��वचार�कर��तथा�गलत�कथन�क��पहचान�कर�:

a) सं�वधान�ने�चुनाव�आयोग�के�सद�य��क��यो�यता��नधा��रत�नह��क��है।�
b) सं�वधान�ने�चुनाव�आयोग�के�सद�य��के�काय�काल�को��न�द���नह���कया�है।�
c) सं�वधान�ने�सेवा�नवृ��चुनाव�आयु���को�सरकार��ारा��कसी�और��नयु���से�वं�चत�नह���कया�है।�
d) चुनाव�आयोग�ने�अपनी��थापना�के�ठ�क�बाद�से�एक�ब�प�ीय��नकाय�के��प�म��काय���कया�है।�
Correct Answer: D
Your Answer:
Explanation

Solution (d)

Basic information:

हालां�क�सं�वधान�ने�चुनाव�आयोग�क���वतं�ता�और��न�प�ता�को�सुर��त�रखने�और�सु�न��त�करने�क��मांग�क��है�, ले�कन�कुछ�खा�मय��को
नोट��कया�जा�सकता�है।�(जैसा��क�नीचे�व����व�ेषण�ता�लका�म���दया�गया�है)

कथन��व�ेषण:

�वक�प�( a) �वक�प�( b) �वक�प�( c) �वक�प�( d)

स�य� स�य� स�य� अस�य�

IASbaba
Web: http://ilp.iasbaba.com/ Score:
Email: ilp@iasbaba.com 0.00 / 200
Page 149
AIPTS/ILP VETERANS-
Exam Title :
2020 TE...
Email : yadavanurag075@gmail.com
Contact : 8882839768

1950 म��इसक���थापना�के
बाद�से�, और�15 अ�टू बर�19
89 तक�चुनाव�आयोग�ने�मु�य
चुनाव�आयु��से��मलकर�एकल
सं�वधान�ने�चुनाव
सद�य��नकाय�के��प�म��काय�
आयोग�के�सद�य�
सं�वधान�ने�सेवा�नवृ��चुनाव �कया।�
क��यो�यता सं�वधान�ने�चुनाव�आयोग�के
आयु���को�सरकार��ारा��कसी
(कानूनी�, शै��क�, सद�य��के�काय�काल�को��न�द��
और��नयु���से�वं�चत�नह���कया 16 अ�टू बर�1989 को�, रा�
�शास�नक�या नह���कया�है।�
है।� प�त�ने�21 से�18 वष��तक�क�
�या�यक) �नधा��रत मतदान�आयु�कम�करने�के
नह��क��है।� कारण�चुनाव�आयोग�के�बढ़े ��ए
काय��का�सामना�करने�के��लए
दो�और�चुनाव�आयु���क�
�नयु���क�।�

QUESTION 55.
महा�यायवाद��( Attorney general) के�संदभ��म��, नीचे�से�गलत�कथन�क��पहचान�कर�:

a) वह�भारत�सरकार�क��अनुम�त�से�आपरा�धक�मुकदम��म��अ�भयु���क��र�ा�कर�सकता�है।�
b) वह�उन�मामल��म��एक�सलाह�दे �सकता�है�, जो�भारत�सरकार�के��लए�सलाह�दे ने�या�पेश�करने�के��लए�कहे�जाते�ह�।�
c) उ�ह��भारत�सरकार�क��अनुम�त�के��बना��कसी�कंपनी�या��नगम�म���नदे शक�के��प�म���नयु���को��वीकार�नह��करना�चा�हए।�
d) वह�सरकारी�नौकर��क���ेणी�म��नह��आता�है।�
Correct Answer: B
Your Answer:
Explanation

Solution (b)

Explanation:

कत���क���कसी�भी�ज�टलता�और�संघष��से�बचने�के��लए�महा�यायवाद���ारा��न�न�सीमा��का�पालन��कया�जाता�है:

1. उसे�भारत�सरकार�के��खलाफ�सं���त�सलाह�या�परामश��नह��दे ना�चा�हए।�
2. उ�ह��ऐसे�मामल��म��सलाह�नह��दे नी�चा�हए�, �जनम��भारत�सरकार�के��लए�सलाह�दे ने�या���तुत�होने�के��लए�कहा�जाता�है।�
3. उसे�भारत�सरकार�क��अनुम�त�के��बना�आपरा�धक�मुकदम��म��अ�भयु���का�बचाव�नह��करना�चा�हए।�
4. उ�ह��भारत�सरकार�क��अनुम�त�के��बना��कसी�कंपनी�या��नगम�म���नदे शक�के��प�म���नयु���को��वीकार�नह��करना�चा�हए।�

हालां�क�, महा�यायवाद��सरकार�के��लए�पूण�का�लक�वक�ल�नह��ह�।�वह�सरकारी�नौकर��क���ेणी�म��नह��आता�है।�इसके�अलावा�, वह��नजी


कानूनी��ै��टस�से��वमु��नह��है।�

QUESTION 56.
रा�य�लोक�सेवा�आयोग�के�बारे�म���न�न�ल�खत�कथन��पर��वचार�कर�:

1. सं�वधान�आयोग�क��साम�य��( strength) को��न�द���नह��करता�है�ले�कन�इस�मामले�को�रा�यपाल�के��ववेक�पर�छोड़��दया�है।�


2. आयोग�के�अ�य��और�सद�य�छह�वष��क��अव�ध�के��लए�या�65 वष��क��आयु��ा�त�करने�तक�, जो�भी�पहले�हो�, पद�धारण�करते�ह�।

ऊपर��दए�गए�कथन��म��से�कौन�सा�सही�है�/ ह��?

a) केवल�1

IASbaba
Web: http://ilp.iasbaba.com/ Score:
Email: ilp@iasbaba.com 0.00 / 200
Page 150
AIPTS/ILP VETERANS-
Exam Title :
2020 TE...
Email : yadavanurag075@gmail.com
Contact : 8882839768

b) केवल�2
c) दोन��1 और�2
d) न�तो�1 और�न�ही�2
Correct Answer: A
Your Answer:
Explanation

Solution (a)

Basic information:

सं�वधान�के�भाग�XIV म��अनु�छे द�315 से�323 दोन��संघ�लोक�सेवा�आयोग�( UPSC) और�रा�य�लोक�सेवा�आयोग�( SPSC) से�संबं�धत


है।�

रा�य�लोक�सेवा�आयोग�म��रा�य�के�रा�यपाल��ारा��नयु��एक�अ�य��और�अ�य�सद�य�होते�ह�।�

कथन��व�ेषण:

कथन�1 कथन�2

स�य� अस�य�

सं�वधान�आयोग�क��साम�य��( strength ) को��न�द���नह� आयोग�के�अ�य��और�सद�य�छह�वष��क��अव�ध�के��लए�या�जब


करता�है�ले�कन�इस�मामले�को�रा�यपाल�के��ववेक�पर�छोड़ तक�वे�62 वष��क��आयु��ा�त�नह��कर�लेते�, जो�भी�पहले�हो
�दया�है।� (यूपीएससी�के�मामले�म��, आयु�सीमा�65 वष��है)।�

QUESTION 57.
�न�न�ल�खत�कथन��पर��वचार�कर�:

1. �व��आयोग��ारा�क��गई��सफा�रश��केवल�सलाहकार��कृ�त�क��ह��और�इस�लए�, सरकार�पर�बा�यकारी�नह��ह�।�
2. UPSC �ारा�क��गई��सफा�रश��केवल�सलाहकार��कृ�त�क��ह��और�इस�लए�, सरकार�पर�बा�यकारी�नह��ह�।�

ऊपर��दए�गए�कथन��म��से�कौन�सा�सही�है�/ ह��?

a) केवल�1
b) केवल�2
c) दोन��1 और�2
d) न�तो�1 और�न�ही�2
Correct Answer: C
Your Answer:
Explanation

Solution (c)

कथन��व�ेषण:

IASbaba
Web: http://ilp.iasbaba.com/ Score:
Email: ilp@iasbaba.com 0.00 / 200
Page 151
AIPTS/ILP VETERANS-
Exam Title :
2020 TE...
Email : yadavanurag075@gmail.com
Contact : 8882839768

कथन�1 कथन�2

स�य� स�य�

�व��आयोग��ारा�क��गई��सफा�रश��केवल�सलाहकारी��कृ�त
क��ह��और�इस�लए�, सरकार�पर�बा�यकारी�नह��ह�।� UPSC �ारा�क��गई��सफा�रश��केवल�सलाहकारी��कृ�त�क��ह�
और�इस�लए�, सरकार�पर�बा�यकारी�नह��ह�।�उस�सलाह�को
इसे��सरे�श�द��म��कह��तो�, ' यह�सं�वधान�म��कह��भी��नधा��रत �वीकार�करना�या�अ�वीकार�करना�क���सरकार�पर��नभ�र�है।�
नह��है��क�आयोग�क���सफा�रश��भारत�सरकार�के��लए
बा�यकारी�ह�गी�या�यह�लाभाथ��रा�य��के�धन�को��ा�त�करने�के आयोग�क��अनुशंसा�से��वच�लत�होने�के��लए�सरकार�क�
�लए�कानूनी�अ�धकार�को�ज�म�दे गी।�उ�ह��आयोग��ारा�क� जवाबदे ही�एकमा��संसद�क����त�जवाबदे ही�है।�
पेशकश��सफा�रशी�कही�गई�ह�।�'

QUESTION 58.
चुनाव�आयोग�के�संबंध�म���न�न�ल�खत�म��से�कौन�सा�कथन�सही�है�/ ह��?

1. चुनाव�आयु���क��सं�या�रा�प�त��ारा��नधा��रत�क��जाती�है।�
2. चुनाव�आयु���क��सेवा�और�काय�काल�क��शत��भी�रा�प�त��ारा��नधा��रत�क��जाती�ह�।�

सही��वक�प�चुन�:

a) केवल�1
b) केवल�2
c) दोन��1 और�2
d) न�तो�1 और�न�ही�2
Correct Answer: C
Your Answer:
Explanation

Solution (c)

कथन��व�ेषण:

कथन�1 कथन�2

स�य� स�य�

IASbaba
Web: http://ilp.iasbaba.com/ Score:
Email: ilp@iasbaba.com 0.00 / 200
Page 152
AIPTS/ILP VETERANS-
Exam Title :
2020 TE...
Email : yadavanurag075@gmail.com
Contact : 8882839768

चुनाव�आयोग�म��मु�य�चुनाव�आयु��और�अ�य�चुनाव�आयु�� सं�वधान�का�अनु�छे द�324 यह��ावधान�करता�है��क�चुनाव


क��उतनी�सं�या�होती�है�, य�द�कोई�हो�, जो�रा�प�त�समय- आयु���और��े�ीय�आयु���क��सेवा�और�काय�काल�क��शत�
समय�पर�तय�कर�सकते�ह�।� रा�प�त��ारा��नधा��रत�क��जाएंगी।�

QUESTION 59.
UPSC क���वतं�ता�के�संबंध�म��नीचे��दए�गए�कथन��पर��वचार�कर��

1. UPSC के�अ�य��या�सद�य�को�रा�प�त��ारा�पद�से�केवल�सं�वधान�म��व�ण�त�तरीके�से�और�उस�आधार�पर�हटाया�जा�सकता�है।�
2. UPSC के�अ�य��और�सद�य��के�संपूण���य��को�भारत�के�समे�कत�कोष�पर�भा�रत��कया�गया�है।�

ऊपर��दए�गए�कथन��म��से�कौन�सा�सही�है�/ ह��?

a) केवल�1
b) केवल�2
c) दोन��1 और�2
d) न�तो�1 और�न�ही�2
Correct Answer: C
Your Answer:
Explanation

Solution (c)

यूपीएससी�क���वतं�ता�( Independence of the UPSC )

सं�वधान�ने�यूपीएससी�के��वतं��और��न�प��कामकाज�को�सुर��त�रखने�और�सु�न��त�करने�के��लए��न�न�ल�खत��ावधान��कए�ह��-

1. UPSC के�अ�य��या�सद�य�को�रा�प�त��ारा�पद�से�केवल�सं�वधान�म��व�ण�त�तरीके�से�और�उस�आधार�पर�हटाया�जा�सकता�है।
इस�लए�, वे�काय�काल�क��सुर�ा�का�आनंद�लेते�ह�।�
2. अ�य��या�सद�य�क��सेवा�क��शत��रा�प�त��ारा��नधा��रत�क��जाती�ह��तथा�उनक���नयु���के�बाद�उनक��हा�न�के��लए�प�रवत�न�नह��हो
सकते�ह�।�
3. UPSC के�अ�य��और�सद�य��के�वेतन�, भ�े�और�प�शन�स�हत�संपूण���य�भारत�के�समे�कत�कोष�पर�भा�रत�होता�है।�इस��कार�, वे
संसद�के�मतदान�के�अधीन�नह��ह�।�

QUESTION 60.
UPSC �ारा��कए�गए�काय��के�संदभ��म��, नीचे��दए�गए�कथन��म��से�कौन�सा�/ से�सही�है�/ ह��?

1. य�द�ऐसा�करने�के��लए�दो�या�दो�से�अ�धक�रा�य��से�अनुरोध��कया�जाता�है�, तो�UPSC �कसी�रा�य�क��सभी�या�कोई�आव�यकता�


को�पूरा�करता�है।�
2. UPSC चुनावी��व�था�से�संबं�धत��ववाद��क��जांच�के��लए�अ�धका�रय��क���नयु���कर�सकता�है।�

उ�चत��वक�प�चुन�:

a) केवल�1
b) केवल�2
c) दोन��1 और�2
d) न�तो�1 और�न�ही�2
Correct Answer: D
Your Answer:

IASbaba
Web: http://ilp.iasbaba.com/ Score:
Email: ilp@iasbaba.com 0.00 / 200
Page 153
AIPTS/ILP VETERANS-
Exam Title :
2020 TE...
Email : yadavanurag075@gmail.com
Contact : 8882839768

Explanation

Solution (d)

कथन��व�ेषण:

कथन�1 कथन�2

अस�य� अस�य�

यूपीएससी�रा�य��क��सहायता�करता�है�(य�द�ऐसा�करने�के��लए
दो�या�दो�से�अ�धक�रा�य���ारा�अनुरोध��कया�गया�है) �कसी�भी
सेवा��के��लए�संयु��भत��क��योजना��के�संचालन�के��लए�,
�जनके��लए��वशेष�यो�यता�रखने�वाले�उ�मीदवार��क� चुनाव�आयोग�(यूपीएससी�नह�) चुनावी��व�था�से�संबं�धत
आव�यकता�होती�है।� �ववाद��क��जांच�के��लए�अ�धका�रय��क���नयु���कर�सकता�है।�
हालाँ�क�, UPSC रा�य�के�रा�यपाल�के�अनुरोध�पर�और
भारत�के�रा�प�त�के�अनुमोदन�के�साथ��कसी�भी�रा�य�क��सभी
या�कोई�आव�यकता��को�पूरा�करता�है।�

QUESTION 61.
नीचे��दए�गए�कथन��पर��वचार�कर�:

1. रा�प�त�मु�य�चुनाव�आयु��( CEC) और�अ�य�चुनाव�आयु���दोन��क���नयु���करता�है।�


2. रा�प�त�के�परामश��के�बाद�ही�CEC, �े�ीय�आयु���क���नयु���कर�सकता�है।�

ऊपर��दए�गए�कथन��म��से�कौन�सा�सही�है�/ ह��?

a) केवल�1
b) केवल�2
c) दोन��1 और�2
d) न�तो�1 और�न�ही�2
Correct Answer: A
Your Answer:
Explanation

Solution (a)

कथन��व�ेषण:

कथन�1 कथन�2

IASbaba
Web: http://ilp.iasbaba.com/ Score:
Email: ilp@iasbaba.com 0.00 / 200
Page 154
AIPTS/ILP VETERANS-
Exam Title :
2020 TE...
Email : yadavanurag075@gmail.com
Contact : 8882839768

स�य� अस�य�

अनु�छे द�324 यह�भी��दान�करता�है��क�रा�प�त�चुनाव�आयोग


सं�वधान�के�अनु�छे द�324 म��यह��ावधान�है��क�मु�य�चुनाव
के�साथ�परामश��के�बाद�ऐसे��े�ीय�आयु���क���नयु���कर�सकता
आयु��और�अ�य�चुनाव�आयु���क���नयु���रा�प�त��ारा�क�
है�, �जसे�वह�चुनाव�आयोग�क��सहायता�के��लए�आव�यक
जाएगी।�
समझता�है।�

QUESTION 62.
भारत�का�सं�वधान�भारत�म��राजकोषीय�संघवाद�के�संतुलन�च��के��प�म���न�न�ल�खत�म��से��कस��नकाय�क��प�रक�पना�करता�है�?

a) �नयं�क�और�महालेखा�परी�क�
b) �व��आयोग�
c) नी�त�आयोग�
d) क���य�सतक�ता�आयोग�
Correct Answer: B
Your Answer:
Explanation

Solution (b)

Explanation:

भारत�का�सं�वधान�भारत�म��राजकोषीय�संघवाद�के�संतुलन�च��के��प�म���व��आयोग�क��प�रक�पना�करता�है।�

QUESTION 63.
रा�ीय�अनुसू�चत�जा�त�आयोग�के�बारे�म���न�न�ल�खत�कथन��पर��वचार�कर�:

1. यह�आव�यक�होने�पर�रा�प�त�को�एक��रपोट� ���तुत�कर�सकता�है।�
2. इसम���वयं�क�����या�को��व�नय�मत�करने�क��श����न�हत�है।�

ऊपर��दए�गए�कथन��म��से�कौन�सा�सही�है�/ ह��?

a) केवल�1
b) केवल�2
c) दोन��1 और�2
d) न�तो�1 और�न�ही�2
Correct Answer: C
Your Answer:
Explanation

Solution (c)

कथन��व�ेषण:

IASbaba
Web: http://ilp.iasbaba.com/ Score:
Email: ilp@iasbaba.com 0.00 / 200
Page 155
AIPTS/ILP VETERANS-
Exam Title :
2020 TE...
Email : yadavanurag075@gmail.com
Contact : 8882839768

कथन�1 कथन�2

स�य� स�य�

रा�ीय�अनुसू�चत�जा�त�आयोग�रा�प�त�को�एक�वा�ष�क��रपोट�
��तुत�करता�है।�यह�आव�यक�होने�पर�भी��रपोट� ���तुत�कर
सकता�है।�
आयोग�म��अपनी����या�को��व�नय�मत�करने�क��श����न�हत
रा�प�त�ऐसी�सभी��रपोट��को�संसद�के�सम��रखता�है�, साथ होती�है।�
ही�, एक��ापन�म��आयोग��ारा�क��गई��सफा�रश��पर�क��गई
कार�वाई�के�बारे�म��बताता�है।��ापन�म��ऐसी��कसी�भी��सफा�रश
को�न�मानने�के�कारण�भी�होने�चा�हए।�

QUESTION 64.
�न�न�ल�खत�कथन��पर��वचार�कर��और�पहचान���क�कौन�सा�कथन�स�य�है�/ ह��?

a) य�द�चुनाव�केवल�संसद�के��लए�आयो�जत��कए�जा�रहे�ह��, तो��य�पूरी�तरह�से�क���सरकार��ारा�वहन��कया�जाता�है�, जब�क�केवल


रा�य��वधानमंडल�के��लए�होने�वाले�चुनाव��के��लए�, �य�पूरी�तरह�से�संबं�धत�रा�य��ारा�वहन��कया�जाता�है।�
b) संसद�और�रा�य��वधानमंडल�के��लए�एक�साथ�चुनाव�के�मामले�म��, �य�को�क���और�रा�य�सरकार��के�बीच�समान��प�से�साझा��कया
जाता�है।�
c) (a) और�(b) दोन��
d) कोई�नह��
Correct Answer: C
Your Answer:
Explanation

Solution (c)

चुनाव�आयोग�का�बजट�और��य�

नई��द�ली�म��चुनाव�आयोग�का�एक�अलग�स�चवालय�है�, �जसम��, एक�पदानु�म�म��लगभग�300 अ�धकारी�शा�मल�ह�।�स�चवालय�के�पास


एक��वतं��बजट�है�, �जसे�सीधे�आयोग�और�क���सरकार�के��व��मं�ालय�के�बीच�परामश��से�अं�तम��प��दया�जाता�है।�क���सरकार�आम�तौर
पर�अपने�बजट�के��लए�आयोग�क���सफा�रश��को��वीकार�करती�है।�

कथन��व�ेषण:

कथन�1 कथन�2

स�य� स�य�

IASbaba
Web: http://ilp.iasbaba.com/ Score:
Email: ilp@iasbaba.com 0.00 / 200
Page 156
AIPTS/ILP VETERANS-
Exam Title :
2020 TE...
Email : yadavanurag075@gmail.com
Contact : 8882839768

संसद�और�रा�य��वधानमंडल�के��लए�एक�साथ�चुनाव�के�मामले
म��, �य�को�क���और�रा�य�सरकार��के�बीच�समान��प�से�साझा
य�द�चुनाव�केवल�संसद�के��लए�आयो�जत��कए�जा�रहे�ह��, तो �कया�जाता�है।�
�य�पूरी�तरह�से�क���सरकार��ारा�वहन��कया�जाता�है�, जब
�या�आप�जानते�ह��?
�क�केवल�रा�य��वधानमंडल�के��लए�होने�वाले�चुनाव��के��लए�,
�य�पूरी�तरह�से�संबं�धत�रा�य��ारा�वहन��कया�जाता�है।� कै�पटल�इ��वपम�ट�के��लए�, मतदाता�सूची�क��तैयारी�से�संबं�धत
�य�और�मतदाता�पहचान�प��के��लए�योजना�का��य�भी�समान
�प�से�साझा��कया�जाता�है।�

QUESTION 65.
नीचे��दए�गए�कथन��म��से�कौन�सा�/ से�, चुनाव�आयोग�के�सलाहकारी��े�ा�धकार�और�समी�ा�काय��के�संदभ��म��सही�है�/ ह��?

1. एक�बार�चुनाव�पूरा�होने�और�प�रणाम�घो�षत�होने�के�बाद�, आयोग��वयं��कसी�भी�प�रणाम�क��समी�ा�नह��कर�सकता�है।�
2. सं�वधान�के�तहत�, आयोग�को�संसद�और�रा�य��वधानसभा��के�सद�य��के�चुनाव�के�बाद�अयो�य�ठहराए�जाने�के�मामले�म�
सलाहकारी��े�ा�धकार�है।�

उ�चत��वक�प�चुन�:

a) केवल�1
b) केवल�2
c) दोन��1 और�2
d) न�तो�1 और�न�ही�2
Correct Answer: C
Your Answer:
Explanation

Solution (c)

कथन��व�ेषण:

कथन�1 कथन�2

स�य� स�य�

IASbaba
Web: http://ilp.iasbaba.com/ Score:
Email: ilp@iasbaba.com 0.00 / 200
Page 157
AIPTS/ILP VETERANS-
Exam Title :
2020 TE...
Email : yadavanurag075@gmail.com
Contact : 8882839768

चुनाव�आयोग�क���या�यक�समी�ा�

चुनाव�आयोग�का�सलाहकारी��े�ा�धकार� उ�चत�या�चका��के��ारा�आयोग�के��नण�य��को�उ�च
�यायालय�और�भारत�के�सव��च��यायालय�म��चुनौती�द��जा
सं�वधान�के�तहत�, आयोग�को�संसद�और�रा�य सकती�है।�
�वधानसभा��के�सद�य��के��नवा�चन�के��लए�अयो�य
ठहराए�जाने�के�मामले�म��सलाहकारी��े�ा�धकार�है।� लंबे�समय�तक�चलने�वाले�क�व�शन��और�कई��या�यक
घोषणा��के�बाद�, एक�बार�चुनाव��क��वा�त�वक����या
इसके�अलावा�, य�द�सव��च��यायालय�और�उ�च शु��हो�जाने�के�बाद�, �यायपा�लका�चुनाव�के�वा�त�वक
�यायालय��के�सम��आने�वाले�चुनाव��म�����य��के संचालन�म��ह�त�ेप�नह��करती�है।�
मामल��को����आचरण�का�दोषी�पाया�जाता�है�, जो�इस
���पर�अपनी�राय�के��लए�आयोग�को�संद�भ�त�करते�ह���क एक�बार�चुनाव�पूरा�होने�और�प�रणाम�घो�षत�होने�के�बाद�,
�या�ऐसे�����को�अयो�य�घो�षत��कया�जाएगा�और�य�द आयोग��कसी�भी�प�रणाम�क��समी�ा�नह��कर�सकता�है।�यह
हां�, तो��कस�अव�ध�के��लए�।� केवल�एक�चुनाव�या�चका�क�����या�के�मा�यम�से�समी�ा�क�
जा�सकती�है�, �जसे�संसद�और�रा�य��वधानसभा��के�चुनाव�
ऐसे�सभी�मामल��म��आयोग�क��राय�रा�प�त�या�रा�यपाल के�संबंध�म��उ�च��यायालय�के�सम��दायर��कया�जा�सकता�है।
के��लए�बा�यकारी�है�, जैसा��क�मामला�हो�सकता�है�, �जन
के��ारा�इस�तरह�क��राय�मांगी�गयी�है।� रा�प�त�और�उपरा�प�त�के�काया�लय��के��लए�चुनाव�के�संबंध
म��, ऐसी�या�चकाएं�केवल�उ�चतम��यायालय�के�सम��दायर
क��जा�सकती�ह�।�

QUESTION 66.
नीचे��दए�गए�कथन��पर��वचार�कर�:

1. रा�प�त�पद�, सेवा��और�मामल��को�UPSC के�दायरे�से�बाहर�कर�सकता�है।�


2. रा�प�त��कसी�भी��ा�धकरण�, कारपोरेट��नकाय�या�साव�ज�नक�सं�थान�के�का�म�क��णाली�को�UPSC के�अ�धकार��े��म��रख�सकते
ह�।�

ऊपर��दए�गए�कथन��म��से�कौन�सा�सही�है�/ ह��?

a) केवल�1
b) केवल�2
c) दोन��1 और�2
d) न�तो�1 और�न�ही�2
Correct Answer: A
Your Answer:
Explanation

Solution (a)

कथन��व�ेषण:

कथन�1 कथन�2

स�य� अस�य�

IASbaba
Web: http://ilp.iasbaba.com/ Score:
Email: ilp@iasbaba.com 0.00 / 200
Page 158
AIPTS/ILP VETERANS-
Exam Title :
2020 TE...
Email : yadavanurag075@gmail.com
Contact : 8882839768

रा�प�त�पद��, सेवा��और�मामल��को�यूपीएससी�के�दायरे�से
संसद��कसी�भी��ा�धकरण�, कारपोरेट��नकाय�या�साव�ज�नक
बाहर�कर�सकता�है।�
सं�थान�के�का�म�क��णाली�को�UPSC के�अ�धकार��े��म��रख
सं�वधान�म��कहा�गया�है��क�रा�प�त�, अ�खल�भारतीय�सेवा� सकती�है।�
और�क���य�सेवा��और�पद��के�संबंध�म��उन��नयम��को��न�द��
�सरे�श�द��म��, UPSC के��े�ा�धकार�को�संसद��ारा�बनाए�गए
कर�सकते�ह���जनम��यूपीएससी�के��लए�परामश���कया�जाना
अ�ध�नयम��ारा�बढ़ाया�जा�सकता�है।�
आव�यक�नह��होगा।�

QUESTION 67.
UPSC के�काया��मक��े�ा�धकार�( functional jurisdiction) के�बाहर��न�न�ल�खत�म��से�कौन�सा�मामला�रखा�जाता�है�?

1. �कसी�भी��पछड़े�वग��के�नाग�रक��के�प��म���नयु��य��या�पद��का�आर�ण�करते�समय�UPSC से�परामश��नह���कया�जाता�है।�
2. सेवा��और�पद��पर��नयु��यां�करने�म��अनुसू�चत�जा�त�और�अनुसू�चत�जनजा�त�के�दाव��को��यान�म��रखते��ए�UPSC से�परामश�
नह���कया�जाता�है।�

उ�चत�उ�र�चुन�:

a) केवल�1
b) केवल�2
c) दोन��1 और�2
d) न�तो�1 और�न�ही�2
Correct Answer: C
Your Answer:
Explanation

Solution (c)

Explanation:

यूपीएससी�के�काया��मक��े�ा�धकार�के�बाहर��न�न�ल�खत�मामले�रखे�गए�ह�।��सरे�श�द��म��, UPSC �ारा��न�न�ल�खत�मामल��पर�सलाह


नह��ली�जाती�है:

(a) �कसी�भी��पछड़े�वग��के�नाग�रक��के�प��म���नयु��य��या�पद��का�आर�ण�करते�समय।�

(b) सेवा��और�पद��पर��नयु��याँ�करने�म��अनुसू�चत�जा�तय��और�अनुसू�चत�जनजा�तय��के�दाव��को��यान�म��रखते��ए।�

(c) आयोग��या��याया�धकरण��क��अ�य�ता�या�सद�यता�के�चयन�के�संबंध�म��, उ�चतम�राजन�यक��कृ�त�और�समूह�सी�और�समूह�डी


सेवा��के�एक�पद�के��लए।�

(d) य�द��कसी�����को��नयु��करने�क��संभावना�नह��है�, तो��कसी�पद�पर�अ�थायी�या�काय�वाहक��नयु���के��लए�चयन�के�संबंध�म�।�

QUESTION 68.
�न�न�ल�खत�म��से�कौन��कसी�कर�या�शु�क�(अनु�छे द�279 के�तहत) क��शु���ा��त�क��जाँच�और��मा�णत�करता�है�?

a) कैग�( CAG)
b) �व��मं�ी�
c) अ�य��, �व��आयोग�
d) अ�य��, नी�त�आयोग�
Correct Answer: A

IASbaba
Web: http://ilp.iasbaba.com/ Score:
Email: ilp@iasbaba.com 0.00 / 200
Page 159
AIPTS/ILP VETERANS-
Exam Title :
2020 TE...
Email : yadavanurag075@gmail.com
Contact : 8882839768

Your Answer:
Explanation

Solution (a)

Explanation:

भारत�के��नयं�क�और�महालेखा�परी�क�( CAG) �कसी�भी�कर�या�शु�क�क��शु���ा��त�का�पता�लगाता�है�तथा�उसे��मा�णत�करता�है


(अनु�छे द�279) ।�उसका��माण�प��अं�तम�है।�‘ शु���ा��त�’ का�अथ��है�एक�कर�क��आय�या�एक�शु�क�से�शु�क�सं�ह�क��लागत�को�घटा
�दया�जाता�है।�

QUESTION 69.
भाषाई�अ�पसं�यक��के��लए�आयु��के�बारे�म���न�न�ल�खत�कथन��पर��वचार�कर�:

1. उनक���नयु���भाषाई�अ�पसं�यक��के��लए��वशेष�अ�धकारी��ारा�क��जाती�है।�
2. क���य��तर�पर�, आयु��अ�पसं�यक�मामल��के�मं�ालय�के�अंतग�त�आता�है।�

ऊपर��दए�गए�कथन��म��से�कौन�सा�सही�है�/ ह��?

a) केवल�1
b) केवल�2
c) दोन��1 और�2
d) न�तो�1 और�न�ही�2
Correct Answer: B
Your Answer:
Explanation

Solution (b)

कथन��व�ेषण:

कथन�1 कथन�2

अस�य� स�य�

सं�वधान�के�अनु�छे द�350-B के��ावधान�के�अनुसरण�म��,


1957 म��भाषाई�अ�पसं�यक��के��लए��वशेष�अ�धकारी�का क���य��तर�पर�, आयु��अ�पसं�यक�मामल��के�मं�ालय�के
काया�लय�बनाया�गया�था।�उ�ह��भारत�के�रा�प�त��ारा��नयु� अंतग�त�आता�है।�
�कया�जाता�है।�
इस�लए�, वह�क���य�अ�पसं�यक�मामल��के�मं�ी�के�मा�यम�से
उ�ह��भाषाई�अ�पसं�यक��के��लए�आयु��के��प�म��भी�ना�मत रा�प�त�को�वा�ष�क��रपोट� �या�अ�य��रपोट� ���तुत�करता�है।�
�कया�गया�है।�

QUESTION 70.

IASbaba
Web: http://ilp.iasbaba.com/ Score:
Email: ilp@iasbaba.com 0.00 / 200
Page 160
AIPTS/ILP VETERANS-
Exam Title :
2020 TE...
Email : yadavanurag075@gmail.com
Contact : 8882839768

भारत�म��सरकार�क��लोकतां��क��णाली�के��पलर�( bulwark) के��प�म���न�न�ल�खत�म��से�कौन�सा�/ से�माना�जाता�है�?

1. �नयं�क�और�महालेखा�परी�क�
2. संघ�लोक�सेवा�आयोग�
3. चुनाव�आयोग�
4. सु�ीम�कोट� �

उ�चत�उ�र�का�चयन�कर�:

a) केवल�1
b) केवल�4
c) 1 और�2
d) 1, 2, 3 और�4
Correct Answer: D
Your Answer:
Explanation

Solution (d)

Explanation:

डॉ. बी�आर�अ�बेडकर�के�अनुसार�, �नयं�क�और�महालेखा�परी�क�( CAG) को�भारत�के�सं�वधान�के�तहत�सबसे�मह�वपूण��अ�धकारी�माना


गया�है।�

CAG भारत�म��सरकार�क��लोकतां��क��णाली�क��बु�नयाद��म��से�एक�है�; अ�य�सु�ीम�कोट� �, चुनाव�आयोग�और�संघ�लोक�सेवा�आयोग�ह�।�

QUESTION 71.
�नयं�क�और�महालेखा�परी�क�( CAG) के�बारे�म���न�न�ल�खत�कथन��पर��वचार�कर�।�

1. वह�भारत�के�रा�प�त��ारा�अपने�हाथ�और�मुहर�( hand and seal) के�तहत�एक�वारंट��ारा��नयु���कया�जाता�है।�


2. उनका�वेतन�और�अ�य�सेवा�शत��रा�प�त��ारा��नधा��रत�क��जाती�ह�।�
3. वह�रा�प�त�को�उस�खाते�के��प�के�संबंध�म��सलाह�दे ता�है��जसम��क���और�रा�य��के�खाते�रखे�जाएंगे।�

ऊपर��दए�गए�कथन��म��से�कौन�सा�सही�है�/ ह��?

a) 1 और�2
b) 1 और�3
c) केवल�1
d) 1, 2 और�3
Correct Answer: B
Your Answer:
Explanation

Solution (b)

कथन��व�ेषण:

कथन�1 कथन�2 कथन�3

IASbaba
Web: http://ilp.iasbaba.com/ Score:
Email: ilp@iasbaba.com 0.00 / 200
Page 161
AIPTS/ILP VETERANS-
Exam Title :
2020 TE...
Email : yadavanurag075@gmail.com
Contact : 8882839768

स�य� अस�य� स�य�

उनका�वेतन�और�अ�य�सेवा�शत��संसद��ारा
�नधा��रत�क��जाती�ह�।�(रा�प�त��ारा�नह�)

सं�वधान�(अनु�छे द�149) संसद�को�संघ


CAG को�भारत�के�रा�प�त��ारा�अपने और�रा�य��और��कसी�अ�य��ा�धकरण�या कैग�रा�प�त�को�उस�खाते�के��प�के
हाथ�और�मुहर�( his hand and seal �नकाय�के�खात��के�संबंध�म��कैग�के�कत��� संबंध�म��सलाह�दे ता�है��जसम��क���और
) के�तहत�वारंट��ारा��नयु���कया�जाता और�श��य��को�संर��त�करने�के��लए रा�य��के�खाते�रखे�जाएंगे।�(अनु�छे द�1
है।� अ�धकृत�करता�है।� 50) ।

तदनुसार�, संसद�ने�कैग�(कत���, श��यां


और�सेवा�क��शत�) अ�ध�नयम�, 1971 ला
गू��कया�है।�

QUESTION 72.
�न�न�ल�खत�म��से�कौन�चुनाव�आयोग�क��श��य��और�काय��का��ह�सा�है�?

1. यह�पूरे�दे श�म���नवा�चन��े���के��े�ीय�भाग��( territorial areas) को��नधा��रत�करता�है।�


2. यह�राजनी�तक�दल��को�चुनाव��च�ह�आवं�टत�करता�है�तथा�उ�ह��चुनाव��च�ह�के�ऐसे�आवंटन�से�संबं�धत��ववाद��को��नपटाने�के��लए
एक�अदालत�के��प�म��काय��करता�है।�
3. यह�चुनाव��के�समय�म��रे�डयो�और�ट�वी�पर�राजनी�तक�दल��क��नी�तय��के��चार�के��लए�एक�रो�टर�( roster) तैयार�कर�सकता�है।�

सही��वक�प�चुन�:

a) केवल�1
b) केवल�2
c) 1 और�2
d) 1, 2 और�3
Correct Answer: D
Your Answer:
Explanation

Solution (d)

Explanation:

चुनाव�आयोग�क��कुछ�श��यां�और�काय��नीचे��दए�गए�ह��-

1. यह�संसद�के�प�रसीमन�आयोग�अ�ध�नयम�के�आधार�पर�पूरे�दे श�म���नवा�चन��े���के��े�ीय��े���( territorial areas ) को


�नधा��रत�करता�है।�
2. यह�समय-समय�पर��नवा�चक�नामावली�तैयार�करता�है�तथा�सभी�पा��मतदाता��को�पंजीकृत�करता�है।�
3. यह�चुनाव��क��तारीख��और�काय��म��को�सू�चत�करता�है�तथा�नामांकन�प���क��जांच�करता�है।�
4. यह�राजनी�तक�दल��को�मा�यता��दान�करता�है�और�उ�ह��चुनाव��च�ह�आवं�टत�करता�है।�
5. यह�राजनी�तक�दल��को�मा�यता�दे ने�और�उ�ह��चुनाव��च�ह�आवं�टत�करने�से�संबं�धत��ववाद��को��नपटाने�के��लए�एक��यायालय�के
�प�म��काय��करता�है।�
6. यह�अ�धका�रय��को�चुनावी��व�था�से�संबं�धत��ववाद��क��पूछताछ�के��लए��नयु��करता�है।�
7. यह�चुनाव�के�समय�पा�ट�य��और�उ�मीदवार���ारा�पालन��कए�जाने�वाले�आचार�सं�हता�का��नधा�रण�करता�है।�

IASbaba
Web: http://ilp.iasbaba.com/ Score:
Email: ilp@iasbaba.com 0.00 / 200
Page 162
AIPTS/ILP VETERANS-
Exam Title :
2020 TE...
Email : yadavanurag075@gmail.com
Contact : 8882839768

8. यह�चुनाव��के�समय�म��रे�डयो�और�ट�वी�पर�राजनी�तक�दल��क��नी�तय��के��चार�के��लए�एक�रो�टर�तैयार�करता�है।�
9. यह�संसद�के�सद�य��क��अयो�यता�से�संबं�धत�मामल��पर�रा�प�त�को�सलाह�दे ता�है।�
10. यह�रा�य��वधा�यका�के�सद�य��क��अयो�यता�से�संबं�धत�मामल��पर�रा�यपाल�को�सलाह�दे ता�है।�

QUESTION 73.
�न�न�ल�खत�कथन��पर��वचार�कर�:

1. CAG भारत�के�समे�कत��न�ध�, ��येक�रा�य�के�समे�कत��न�ध�और���येक��वधानसभा��े��वाले�क���शा�सत��दे श�क��समे�कत��न�ध


से�सभी��य�से�संबं�धत�खात��का�ऑ�डट�करता�है।�
2. CAG भारत�के�आक��मकता�कोष�और�भारत�के�साव�ज�नक�खाते�के�साथ-साथ���येक�रा�य�के�आक��मक��न�ध�और���येक�रा�य
के�साव�ज�नक�खाते�से�सभी��य�का�ऑ�डट�करता�है।�

ऊपर��दए�गए�कथन��म��से�कौन�सा�सही�है�/ ह��?

a) केवल�1
b) केवल�2
c) दोन��1 और�2
d) न�तो�1 और�न�ही�2
Correct Answer: C
Your Answer:
Explanation

Solution (c)

कथन��व�ेषण:

कथन�1 कथन�2

स�य� स�य�

CAG भारत�के�समे�कत��न�ध�, ��येक�रा�य�के�समे�कत��न�ध CAG भारत�के�आक��मकता�कोष�और�भारत�के�साव�ज�नक


और���येक��वधानसभा��े��वाले�क���शा�सत��दे श�क��समे�कत खाते�के�साथ-साथ���येक�रा�य�के�आक��मक��न�ध�और���येक
�न�ध�से�सभी��य�से�संबं�धत�खात��का�ऑ�डट�करता�है।� रा�य�के�साव�ज�नक�खाते�से�सभी��य�का�ऑ�डट�करता�है।�

अ�त�र��जानकारी:

• कैग�क���सरकार�और�रा�य�सरकार��के��कसी�भी��वभाग��ारा�रखे�गए�सभी��े �ड�ग�, �व�नमा�ण�, लाभ�और�हा�न�खात��, बैल�स�शीट


और�अ�य�सहायक�खात��का�ऑ�डट�करता�है।�
• कैग�क���और���येक�रा�य�क���ा��तय��और��य�का�ऑ�डट�करता�है�ता�क�वह��वयं�को�संतु��कर�सके��क�उसे�मू�यांकन�और�सं�ह
तथा�राज�व�के�समु�चत�आवंटन�के��लए�एक��भावी�जाँच�को�सुर��त�करने�के��लए�उस��नयम�और����या�को�तैयार��कया�गया�है।�
• कैग��न�न�ल�खत�क���ा��तय��और��य�का�ऑ�डट�करता�है:

(a) सभी��नकाय��और��ा�धकरण���जसे�क���या�रा�य�के�राज�व�से�अ�धक��व�पो�षत��कया�गया�है�;

(b) सरकारी�कंप�नयाँ�; तथा�

IASbaba
Web: http://ilp.iasbaba.com/ Score:
Email: ilp@iasbaba.com 0.00 / 200
Page 163
AIPTS/ILP VETERANS-
Exam Title :
2020 TE...
Email : yadavanurag075@gmail.com
Contact : 8882839768

(c) अ�य��नगम��और��नकाय��, जब�संबं�धत�कानून���ारा�आव�यक�हो।�

• कैग�ऋण�, डू बते�धन�, जमा�, अ��म�, �नलं�बत�खात��और��ेषण��वसाय�से�संबं�धत�क���और�रा�य�सरकार��के�सभी�लेनदे न�का


ऑ�डट�करता�है।�
• वह�रा�प�त�क���वीकृ�त�के�साथ�या�रा�प�त��ारा�आव�यक�होने�पर��ा��तय��, �टॉक�खाते�और�अ�य�का�ऑ�डट�भी�करता�है।�
• वह�रा�प�त�या�रा�यपाल��ारा�अनुरोध��कए�जाने�पर��कसी�अ�य��ा�धकरण�के�खात��का�ऑ�डट�करता�है।�उदाहरण�के��लए�, �थानी
य��नकाय��का�ऑ�डट।�

QUESTION 74.
CAG के�बारे�म���न�न�ल�खत�कथन��पर��वचार�कर��और�गलत�कथन�क��पहचान�कर�:

a) CAG संसद�का�एक�एज�ट�है।�
b) CAG केवल�संसद�के���त�उ�रदायी�है।�
c) CAG क��ओर�से�कानूनी�, �नयामक�और�औ�च�य�लेखा�परी�ा�अ�नवाय��है।�
d) भारत�का�सं�वधान�CAG को��नयं�क�और�महालेखा�परी�क�के��प�म��बताता�है।�
Correct Answer: C
Your Answer:
Explanation

Solution (c)

कथन��व�ेषण:

�वक�प�( a) और�( b) �वक�प�( c) �वक�प�( d)

स�य� अस�य� स�य�

भारत�का�सं�वधान�कैग�को��नयं�क�और
कैग�संसद�का�एज�ट�है�तथा�संसद�क��ओर�से
महालेखा�परी�क�के��प�म��बताता�है।�
�य�का�ऑ�डट�करता�है।� कैग�क��ओर�से�केवल�कानूनी�और
�व�नयामक�य�ऑ�डट�अ�नवाय��है�, जब�क हालां�क�, �वहार�म��, कैग�एक�लेखा
इस�लए�, वह�केवल�संसद�के���त�उ�रदायी औ�च�य�संबंधी�ऑ�डट��ववेकाधीन�है।� परी�क�क��भू�मका�को�पूरा�कर�रहा�है�,
है।�
�नयं�क�क��नह�।�

QUESTION 75.
�न�न�ल�खत�म��से��कसे�रा�प�त��ारा�सव��च��यायालय�के��यायाधीश�के�समान�उसी�तरीके�और�आधार�पर�हटाया�जा�सकता�है�?

1. �नयं�क�और�महालेखा�परी�क�
2. मु�य�चुनाव�आयु��
3. उ�च��यायालय�के��यायाधीश�

उ�चत��वक�प�चुन�:

a) केवल�1

IASbaba
Web: http://ilp.iasbaba.com/ Score:
Email: ilp@iasbaba.com 0.00 / 200
Page 164
AIPTS/ILP VETERANS-
Exam Title :
2020 TE...
Email : yadavanurag075@gmail.com
Contact : 8882839768

b) केवल�2
c) 1 और�2
d) 1, 2 और�3
Correct Answer: D
Your Answer:
Explanation

Solution (d)

Explanation:

सभी��दए�गए�पद�- CAG, CEC और�उ�च��यायालय�के��यायाधीश�- रा�प�त��ारा�सव��च��यायालय�के��यायाधीश�के��प�म��और�उसी


आधार�पर�पद�से�हटाए�जा�सकते�ह�।�

�सरे�श�द��म��, उ�ह��रा�प�त��ारा��वशेष�ब�मत�के�साथ�संसद�के�दोन��सदन���ारा�उस��भाव�, या�तो����वहार�या�अ�मता�के�आधार�पर


पा�रत��कए�गए�एक���ताव�के�आधार�पर�हटाया�जा�सकता�है।�

QUESTION 76.
�न�न�ल�खत�म��से�कौन�नी�त�आयोग�काय��णाली�के��लए�माग�दश�क��स�ांत��का��ह�सा�ह��?

1. अ��योदय�( Antyodaya)
2. समावेश�( Inclusion)
3. सततता�( Sustainability)
4. जनसां��यक�य�लाभांश�

सही��वक�प�चुन�:

a) 1 और�2
b) 1, 3 और�4
c) 2 और�3
d) 1, 2, 3 और�4
Correct Answer: D
Your Answer:
Explanation

Solution (d)

Explanation:

उपरो��काय��को�करने�म��, नी�त�आयोग��न�न�ल�खत��स�ांत���ारा��नद� �शत�है�

पं�डत�द�नदयाल�उपा�याय�के�' अं�योदय�' के��वचार�के�अनुसार�, गरीब��, हा�शए�पर�रहने�वाले�और�द�लत��क�


1. अ��योदय�
सेवा�और�उ�थान�को��ाथ�मकता�द� ।�

कमजोर�और�हा�शए�पर�खड़े�वग��, सभी��कार�के��ल�ग�, �े��, धम��, जा�त�या�वग��क��पहचान-आधा�रत


2. समावेश�
असमानता��का��नवारण।�

IASbaba
Web: http://ilp.iasbaba.com/ Score:
Email: ilp@iasbaba.com 0.00 / 200
Page 165
AIPTS/ILP VETERANS-
Exam Title :
2020 TE...
Email : yadavanurag075@gmail.com
Contact : 8882839768

हमारे�गांव��को�हमारी�नै�तकता�, सं�कृ�त�और�जी�वका�के�आधार�पर�जीवन�श���और�ऊजा��को��वक�सत
3. �ाम�
���या�म��एक�कृत�कर�।�

4. जनसां��यक�य हमारी�सबसे�बड़ी�संप���, भारत�के�लोग��म��; उ�पादक�आजी�वका�के�अवसर��के�मा�यम�से�, �श�ा�और


लाभांश� कौशल�, और�उनके�सश��करण�के�मा�यम�से�, उनके��वकास�पर��यान�क���त�करके।�

�वकास����या�को�जन-संचा�लत����या�म��बदलना�, जागृत�और�सहभागी�नाग�रकता�बनाना�- सुशासन�का


5. जन�भागीदारी�
चालक�है।�

शासन�क��एक�खुली�, पारदश��, जवाबदे ह�, स��य�और�उ�े �यपूण��शैली�का�पोषण�कर��, आउटले�से�आउटपुट


6. शासन�
से�आउटकम�तक�( from Outlay to Output to Outcome ) �यान�क���त�कर�।�

पया�वरण�के���त�हमारी��ाचीन�परंपरा�पर�आधा�रत�, हमारी�योजना�और��वकासा�मक����या�के�मूल�म�
7. सततता�
सततता�बनाए�रख�।�

QUESTION 77.
�न�न�ल�खत�म��से��कसे�दे श�का�सव��च�कानून�अ�धकारी�माना�जाता�है�?

a) भारत�के�मु�य��यायाधीश�
b) महा�यायवाद��
c) महा�धव�ा�
d) रा�प�त�
Correct Answer: B
Your Answer:
Explanation

Solution (b)

Explanation:

सं�वधान�(अनु�छे द�76) म��भारत�के��लए�महा�यायवाद��के�काया�लय�के��लए��ावधान��कया�गया�है।�वे�दे श�के�सव��च�कानून�अ�धकारी�ह�।�

सं�वधान�(अनु�छे द�165) म��रा�य��के��लए�महा�धव�ा�के�काया�लय�के��लए��ावधान��कया�गया�है।�वे�रा�य�के�सव��च�कानून�अ�धकारी�ह�।


इस��कार�वह�भारत�के�महा�यायवाद��से�मेल�खाता�है।�

QUESTION 78.
महा�धव�ा�( Advocate General) के�संदभ��म���न�न�ल�खत�कथन��पर��वचार�कर��और�गलत�कथन�क��पहचान�कर�:

a) वह�रा�प�त��ारा��नयु���कया�जाता�है�और�रा�प�त�के��सादपय�त�पद�धारण�करता�है।�
b) महा�धव�ा�के�काया�लय�का�काय�काल�सं�वधान��ारा��नधा��रत�नह��है।�
c) सं�वधान��ारा�महा�धव�ा�का�पा�र��मक�तय�नह��है।�

IASbaba
Web: http://ilp.iasbaba.com/ Score:
Email: ilp@iasbaba.com 0.00 / 200
Page 166
AIPTS/ILP VETERANS-
Exam Title :
2020 TE...
Email : yadavanurag075@gmail.com
Contact : 8882839768

d) कोई�नह��
Correct Answer: A
Your Answer:
Explanation

Solution (a)

Explanation:

सं�वधान�(अनु�छे द�165) ने�रा�य��के��लए�महा�धव�ा�के�काया�लय�के��लए��ावधान��कया�है।�

वह�रा�यपाल��ारा��नयु���कया�जाता�है।�वह�रा�यपाल�के��सादपय�त�पद�धारण�करता�है।�इसका�मतलब�है�

�क�वह��कसी�भी�समय�रा�यपाल��ारा�हटाया�जा�सकता�है।�वह�रा�यपाल�को�अपना�इ�तीफा�स�पकर�अपना�पद�भी�छोड़�सकते�ह�।�इस�लए�,
कथन�( a) गलत�है।�

QUESTION 79.
�न�न�ल�खत�म��से�कौन�रा�प�त�के��सादपय�त�( pleasure) अपना�पद�धारण�नह��करता�है�?

1. �नयं�क�और�महालेखा�परी�क�
2. मु�य�चुनाव�आयु��
3. मं��य��( Ministers)
4. महा�यायवाद��
5. एक�उ�च��यायालय�के��यायाधीश�

उ�चत��वक�प�चुन�:

a) केवल�1 और�2
b) केवल�1, 2 और�5
c) 1, 2, 3 और�5
d) 1, 2, 3, 4 और�5
Correct Answer: B
Your Answer:
Explanation

Solution (b)

Explanation:

एक�उ�च��यायालय�के��यायाधीश�, �नयं�क�एवं�महालेखा�परी�क�और�मु�य�चुनाव�आयु��को�काय�काल�क��सुर�ा��दान�क��गयी�है।�उ�ह�
रा�प�त��ारा�पद�से�केवल�सं�वधान�म��व�ण�त�तरीके�से�और�उन�आधार��पर�हटाया�जा�सकता�है।�इसका�अथ��यह�है��क�वे�रा�प�त�के
�सादपय�त�अपना�काया�लय�नह��रखते�ह��, हालां�क�वे�उनके��ारा��नयु���कए�जाते�ह�।�

सं�वधान�म��कोई�आधार�व�ण�त�नह��है�, �जस�पर�रा�प�त��ारा�रा�यपाल�को�हटाया�जा�सकता�है।�

अनु�छे द�75 म��कहा�गया�है��क�मं�ी�रा�प�त�के��सादपय�त�पद�धारण�करते�ह��, �जसका�अथ��है��क�रा�प�त��कसी�मं�ी�को�तब�भी�हटा�सकते


ह��जब�मं��प�रषद�को�लोकसभा�का��व�ास��ा�त�हो।�हालाँ�क�, रा�प�त��धानमं�ी�क��सलाह�पर�ही��कसी�मं�ी�को�हटाता�है।�

महा�यायवाद��(एजी) को�रा�प�त��ारा��नयु���कया�जाता�है।�वह�रा�प�त�के��सादपय�त�पद�धारण�करता�है।�इसका�अथ��यह�है��क�उसे��कसी
भी�समय�रा�प�त��ारा�हटाया�जा�सकता�है।�

IASbaba
Web: http://ilp.iasbaba.com/ Score:
Email: ilp@iasbaba.com 0.00 / 200
Page 167
AIPTS/ILP VETERANS-
Exam Title :
2020 TE...
Email : yadavanurag075@gmail.com
Contact : 8882839768

एक�रा�यपाल�पांच�साल�के�काय�काल�के��लए�पद�धारण�करता�है�, �जस��दन�से�वह�अपने�काया�लय�म���वेश�करता�है।�हालां�क�, पांच�साल


का�यह�काय�काल�रा�प�त�के��सादपय�त�के�अधीन�है।�

रा�यपाल�के�पास�काय�काल�क��कोई�सुर�ा�नह��होती�है�और�न�ही�कोई��न��त�काय�काल�होता�है।�उ�ह���कसी�भी�समय�रा�प�त��ारा�हटाया
जा�सकता�है।�

QUESTION 80.
भारत�के�सॉ�ल�सटर�जनरल�के�संबंध�म���न�न�ल�खत�कथन��पर��वचार�कर�:

1. सॉ�ल�सटर�जनरल�, अटॉन��जनरल�को�अपनी�आ�धका�रक��ज�मेदा�रय��क��पू�त��म��सहायता�करता�है।�
2. सं�वधान�म��भारत�के�सॉ�ल�सटर�जनरल�के�बारे�म��कह��नह���लखा�गया�है।�
3. कै�बनेट�क���नयु���स�म�त�( ACC) भारत�के�सॉ�ल�सटर�जनरल�क���नयु���करती�है।�

ऊपर��दए�गए�कथन��म��से�कौन�सा�सही�है�/ ह��?

a) केवल�1
b) केवल�1 और�2
c) केवल�2 और�3
d) 1, 2 और�3
Correct Answer: B
Your Answer:
Explanation

Solution (b)

कथन��व�ेषण:

कथन�1 कथन�2 कथन�3

स�य� स�य� अस�य�

कै�बनेट�क���नयु���स�म�त�(एसीसी)
जैसे�भारत�के��लए�अटॉन��जनरल�, सॉ�ल�स
�नयु���क���सफा�रश�करती�है�और
टर�जनरल�और�अ�त�र��सॉ�ल�सटर�जनरल
रा�प�त�आ�धका�रक�तौर�पर�सॉ�ल�सटर
भारत�का�सॉ�ल�सटर�जनरल�भारत�के सरकार�को�सलाह�दे ते�ह��तथा�कानून
जनरल��नयु��करता�है।�
�लए�अटॉन��जनरल�के�अधीन�थ�होता�है।� अ�धकारी�(�नयम�और�शत�) �नयम�, 1972
के�संदभ��म��भारत�संघ�क��ओर�से�पेश�होते सॉ�ल�सटर�जनरल�, अ�त�र��सॉ�ल�सटर
वह�दे श�का��सरा�कानून�अ�धकारी�होता�है ह�।� जनरल�क���नयु���का���ताव�आम�तौर
, अटॉन��जनरल�क��सहायता�करता�है�,
पर�, कानूनी�मामल��के��वभाग�म��संयु�
और�भारत�के��लए�चार�अ�त�र� हालां�क�, भारत�के��लए�अटॉन��जनरल�के
स�चव�/ कानून�स�चव�के��तर�पर�ले�जाया
सॉ�ल�सटर�जनरल��ारा��वयं�सहायता पद�के��वपरीत�, जो��क�भारत�के�सं�वधान
जाता�है�तथा�कानून�और��याय�मं�ी�क�
�ा�त�करता�है।� के�अनु�छे द�76 के�तहत�एक�संवैधा�नक�पद
मंजूरी��ा�त�करने�के�बाद�, ��ताव�एसीसी
है�, सॉ�ल�सटर�जनरल�और�अ�त�र�
के�पास�जाता�है��फर�रा�प�त�के�पास
सॉ�ल�सटर�जनरल�के�पद�केवल�वैधा�नक�ह�।
जाता�है।�

IASbaba
Web: http://ilp.iasbaba.com/ Score:
Email: ilp@iasbaba.com 0.00 / 200
Page 168
AIPTS/ILP VETERANS-
Exam Title :
2020 TE...
Email : yadavanurag075@gmail.com
Contact : 8882839768

�या�आप�जानते�ह��?

• वत�मान�म��, भारत�के�सॉ�ल�सटर�जनरल�तुषार�मेहता�ह�।�

QUESTION 81.
नीचे��दए�गए�कथन��म��से�कौन�सा�UPSC के�बारे�म��स�य�नह��है�?

1. UPSC सेवा��के�वग�करण�, वेतन�और�सेवा�शत��, कैडर��बंधन�, ��श�ण�, और�इसी�तरह�के�अ�य�से�संबं��नह��है।�


2. UPSC भारत�म��एक�क���य�का�म�क�एज�सी�है�तथा�इसे�अ�सर�भारत�म��' मे�रट��स�टम�का�रखवाला�' (watch-dog of merit
system) माना�जाता�है।�

उ�चत��वक�प�चुन�:

a) केवल�1
b) केवल�2
c) दोन��1 और�2
d) न�तो�1 और�न�ही�2
Correct Answer: B
Your Answer:
Explanation

Solution (b)

कथन��व�ेषण:

कथन�1 कथन�2

स�य� अस�य�

UPSC अ�खल�भारतीय�सेवा��और�क���य�सेवा�-�ुप�ए
और�समूह�बी�म��भत��से�संब��है�तथा�पदो��त�और
अनुशासना�मक�मामल��पर�परामश��दे ने�के��लए�सरकार�को भारत�म��UPSC ‘ मे�रट��स�टम�का�रखवाला�’ होने�क��क�पना
सलाह�दे ता�है।�यह�सेवा��के�वग�करण�, वेतन�और�सेवा�शत� करता�है।�
, कैडर��बंधन�, ��श�ण�, आ�द�से�संब��नह��है।�
UPSC केवल�एक�क���य�भत��एज�सी�है�, जब�क�का�म�क�और
इन�मामल��को�का�म�क�और���श�ण��वभाग��ारा��नयं��त ��श�ण��वभाग�भारत�म��क���य�का�म�क�एज�सी�है।�
�कया�जाता�है�- जो�का�म�क�, लोक��शकायत�और�प�शन
मं�ालय�के�तीन��वभाग��म��से�एक�है�

QUESTION 82.
महा�यायवाद��( AG) के�बारे�म���न�न�ल�खत�म��से�कौन�सा�कथन�सही�है�/ ह��?

1. उसे�संसद�के�दोन��सदन��क��काय�वाही�म��बोलने�और�भाग�लेने�का�अ�धकार�है।�
2. उसे�संसद�क��संयु��बैठक�म��बोलने�और�भाग�लेने�का�अ�धकार�है।�
3. वह�उन�सभी��वशेषा�धकार��और���तर�ा��का�आनंद�लेता�है�जो�संसद�के�सद�य�के��लए�उपल�ध�ह�।�

IASbaba
Web: http://ilp.iasbaba.com/ Score:
Email: ilp@iasbaba.com 0.00 / 200
Page 169
AIPTS/ILP VETERANS-
Exam Title :
2020 TE...
Email : yadavanurag075@gmail.com
Contact : 8882839768

उ�चत��वक�प�चुन�:

a) केवल�1
b) केवल�1 और�2
c) केवल�2 और�3
d) 1, 2 और�3
Correct Answer: D
Your Answer:
Explanation

Solution (d)

कथन��व�ेषण:

कथन�( 1) और�( 2) कथन�( 3)

स�य� स�य�

अपने�आ�धका�रक�कत����के��दश�न�म��, महा�यायवाद��को
बोलने�और�संसद�के�दोन��सदन��या�उनके�संयु��बैठक�क�
महा�यायवाद��उन�सभी��वशेषा�धकार��और���तर�ा��का�आनंद
काय�वाही�म��भाग�लेने�का�अ�धकार�है�तथा�संसद�क��कोई�भी
लेता�है�जो�संसद�के�सद�य�के��लए�उपल�ध�ह�।�
स�म�त��जसम��वह�सद�य�ना�मत��कया�जा�सकता�है�, ले�कन
उसे�वोट�दे ने�का�अ�धकार�नह��है।�

QUESTION 83.
�न�न�ल�खत�कत����पर��वचार�कर�:

1. ऐसे�कानूनी�मामल��पर�रा�य�क��सरकार�को�सलाह�दे ना�जो�रा�यपाल��ारा�उसे�संद�भ�त��कया�जाता�है।�
2. एक�कानूनी�च�र��के�ऐसे�अ�य�कत����को��नभाने�के��लए�जो�रा�यपाल��ारा�उसे�स�पे�जाते�ह�।�
3. अपने�आ�धका�रक�कत����के��दश�न�म��, वह�रा�य�के�कानून�के��कसी�भी��यायालय�के�सम��उप��थत�होने�का�हकदार�है।�

उपरो��कत����से�संबं�धत�ह��-

a) महा�यायवाद��
b) महा�धव�ा�
c) सॉ�ल�सटर�जनरल�
d) ��यात��याय�वद�
Correct Answer: B
Your Answer:
Explanation

Solution (b)

Explanation:

IASbaba
Web: http://ilp.iasbaba.com/ Score:
Email: ilp@iasbaba.com 0.00 / 200
Page 170
AIPTS/ILP VETERANS-
Exam Title :
2020 TE...
Email : yadavanurag075@gmail.com
Contact : 8882839768

रा�य�म��सरकार�के�मु�य�कानून�अ�धकारी�के��प�म��, महा�धव�ा�के�कत����म���न�न�ल�खत�शा�मल�ह�:

1. ऐसे�कानूनी�मामल��पर�रा�य�क��सरकार�को�सलाह�दे ना�जो�रा�यपाल��ारा�उसे�संद�भ�त��कया�जाता�है।�
2. एक�कानूनी�च�र��के�ऐसे�अ�य�कत����को��नभाने�के��लए�जो�रा�यपाल��ारा�उसे�स�पे�जाते�ह�।�
3. सं�वधान�या��कसी�अ�य�कानून��ारा�उसे��दए�गए�काय��का��नव�हन�करने�के��लए।�

अपने�आ�धका�रक�कत����के��दश�न�म��, महा�धव�ा�रा�य�के�भीतर��कसी�भी�कानून�क��अदालत�के�सम��उप��थत�होने�का�हकदार�है।�

QUESTION 84.
नी�त�आयोग�के�संदभ��म��, नीचे��दए�गए�कथन��म��से�कौन-सा�/ से�सही�है�/ ह��?

1. योजना�आयोग�क��तरह�, यह�भी�न�तो�एक�संवैधा�नक��नकाय�है�और�न�ही�एक�वैधा�नक��नकाय�है।�
2. यह�भारत�सरकार�क���मुख�' �थ�क�ट� क�' है।�

सही�उ�र�चुन�:

a) केवल�1
b) केवल�2
c) दोन��1 और�2
d) न�तो�1 और�न�ही�2
Correct Answer: C
Your Answer:
Explanation

Solution (c)

कथन��व�ेषण:

कथन�1 कथन�2

स�य� स�य�

नी�त�आयोग�भी�, योजना�आयोग�क��तरह�, भारत�सरकार


(यानी�, क���य�मं��मंडल) के�एक�काय�कारी�संक�प��ारा�बनाया नी�त�आयोग�भारत�सरकार�क���मुख�' �थ�क�ट� क�' है�, जो
गया�था।�इस�लए�, यह�न�तो�एक�संवैधा�नक��नकाय�है�और�न�ही �दशा�मक�और�नी�तगत�दोन��इनपुट��दान�करती�है।�भारत
एक�वैधा�नक��नकाय�है।��सरे�श�द��म��, यह�एक�गैर-संवैधा�नक सरकार�के��लए�रणनी�तक�और�द�घ�का�लक�नी�तय��और
या�अ�त�र�-संवैधा�नक��नकाय�है�(यानी�, सं�वधान��ारा�नह� काय��म��को��डजाइन�करते��ए�, नी�त�आयोग�क���और�रा�य�
बनाया�गया�है) और�एक�गैर-सां�व�धक��नकाय�(संसद�के�एक को��ासं�गक�तकनीक��सलाह�भी��दान�करता�है।�
अ�ध�नयम��ारा��न�म�त�नह�) है।�

QUESTION 85.
नी�त�आयोग�क��शासन�प�रषद�( Governing Council) म��शा�मल�ह��-

1. सभी�रा�य��के�मु�यमं�ी�
2. �वधानसभा��वाले�क���शा�सत��दे श��के�मु�यमं�ी�

IASbaba
Web: http://ilp.iasbaba.com/ Score:
Email: ilp@iasbaba.com 0.00 / 200
Page 171
AIPTS/ILP VETERANS-
Exam Title :
2020 TE...
Email : yadavanurag075@gmail.com
Contact : 8882839768

3. सभी�रा�य��और�क���शा�सत��दे श��के�रा�यपाल�

उपयु��कूट�चुन�:

a) केवल�1
b) केवल�1 और�2
c) केवल�2 और�3
d) 1, 2 और�3
Correct Answer: B
Your Answer:
Explanation

Solution (b)

नी�त�आयोग�क��संरचना�इस��कार�है:

अ�य�� भारत�के��धान�मं�ी�

(a) सभी�रा�य��के�मु�यमं�ी�

शासन�प�रषद� (b) �वधानसभा��(यानी�, �द�ली�और�पु�चेरी) के�साथ�क���शा�सत��दे श��के�मु�यमं�ी�

(c) अ�य�क���शा�सत��दे श��के�उप-रा�यपाल�

ये��धान�मं�ी��ारा�बुलायी�जाती�ह��और�इस��े��म��रा�य��के�मु�यमं�ी�और�क���शा�सत��दे श��के�उप-
रा�यपाल�शा�मल�होते�ह�।�
�े�ीय�प�रषद� �
इनक��अ�य�ता�नी�त�आयोग�के�अ�य��या�उनके�नॉ�मनी�करते�ह�।�

�वशेष��, जानकर�और��ासं�गक�डोमेन��ान�के�साथ�अ�यासरत�लोग��को��वशेष�आमं��त�सद�य�के��प�म�
�वशेष��नमं�ण�
�धानमं�ी��ारा�ना�मत��कया�जाता�है।�

IASbaba
Web: http://ilp.iasbaba.com/ Score:
Email: ilp@iasbaba.com 0.00 / 200
Page 172
AIPTS/ILP VETERANS-
Exam Title :
2020 TE...
Email : yadavanurag075@gmail.com
Contact : 8882839768

इसम���धानमं�ी�अ�य��के��प�म��तथा�इनके�अलावा�शा�मल�ह�:

(i) उपा�य�: वह��धानमं�ी��ारा��नयु���कया�जाता�है।�उ�ह��कै�बनेट�मं�ी�का�दजा���ा�त�होता�है।�

(ii) सद�य: पूण�का�लक।�उ�ह��रा�य�मं�ी�का�दजा���ा�त�होता�है।�

(iii) अंशका�लक�सद�य: अ�णी��व��व�ालय��, शोध�संगठन��और�अ�य�संबं�धत�सं�थान��से�अ�धकतम�2 ।


पूण�का�लक
अंशका�लक�सद�य�एक�रोटे शन�पर�ह�गे।�
संगठना�मक�ढांचा�
(iv) पदे न�सद�य: �धान�मं�ी��ारा�ना�मत��कए�जाने�वाले�क���य�मं��प�रषद�के�अ�धकतम�4 सद�य।�

(v) मु�य�काय�कारी�अ�धकारी: उ�ह��भारत�सरकार�के�स�चव�के�पद�पर�एक��न��त�काय�काल�के��लए��धान


मं�ी��ारा��नयु���कया�जाता�है।�

(vi) स�चवालय: जैसा�आव�यक�समझा�जाए।�

QUESTION 86.
�न�न�ल�खत�कथन��पर��वचार�कर�:

1. कैग�रा�प�त�को�तीन�ऑ�डट��रपोट� �दे ता�है- �व�नयोग�खात��पर�ऑ�डट��रपोट� �, �व��खात��पर�ऑ�डट��रपोट� �और�साव�ज�नक�उप�म�


पर�ऑ�डट��रपोट� ।�
2. भारत�म��, काय�पा�लका�सरकारी�खजाने�से�केवल�कैग�क��मंजूरी�के�साथ�पैसे��नकाल�सकती�है।�

ऊपर��दए�गए�कथन��म��से�कौन�सा�सही�है�/ ह��?

a) केवल�1
b) केवल�2
c) दोन��1 और�2
d) न�तो�1 और�न�ही�2
Correct Answer: A
Your Answer:
Explanation

Solution (a)

कथन��व�ेषण:

कथन�1 कथन�2

स�य� अस�य�

IASbaba
Web: http://ilp.iasbaba.com/ Score:
Email: ilp@iasbaba.com 0.00 / 200
Page 173
AIPTS/ILP VETERANS-
Exam Title :
2020 TE...
Email : yadavanurag075@gmail.com
Contact : 8882839768

समे�कत��न�ध�से�धन�के�मु�े�पर�कैग�का�कोई��नयं�ण�नह��है�और
कैग��ारा�रा�प�त�को�तीन�ऑ�डट��रपोट� �द��जाती�है- �व�नयोग कई��वभाग�कैग�के��व�श���ा�धकरण�के��बना�चेक�जारी��कये
खात��पर�ऑ�डट��रपोट� �, �व��खात��पर�ऑ�डट��रपोट� �और धन��नकालने�के��लए�अ�धकृत�ह��, जो�केवल�ऑ�डट�चरण�से
साव�ज�नक�उप�म��पर�ऑ�डट��रपोट� ।� संब��ह��, जब�क��य�पहले�ही�हो�चुका�हो।�

रा�प�त�इन��रपोट��को�संसद�के�दोन��सदन��के�सम��रखता�है। भारत�का�कैग�पूरी�तरह�से���टे न�के�कैग�से��भ��है�, �जसम�


इसके�बाद�, लोक�लेखा�स�म�त�उनक��जांच�करती�है�तथा �नयं�क�और�महालेखा�परी�क�दोन��क��श��याँ�होती�ह�।��सरे
संसद�को�अपने��न�कष��क���रपोट� �दे ती�है।� श�द��म��, ��टे न�म��, काय�पा�लका�सरकारी�खजाने�से�केवल�कैग
के�अनुमोदन�से�धन��नकाल�सकती�है।�

QUESTION 87.
नी�त�आयोग�के��भावी��शासन�के��लए�सात��तंभ�म��से�नीचे��दए�गए�कौन�से�ह��?

1. सभी�पहलु��म��म�हला��को�सश��बनाना।�
2. युवा��के��लए�अवसर�क��समानता।�
3. �ौ�ो�गक��के�उपयोग�के�मा�यम�से�पारद�श�ता।�
4. आतंकवाद�का�मुकाबला।�

उ�चत��वक�प�चुन�:

a) 1 और�2
b) 2 और�4
c) 1, 2 और�3
d) 1, 2, 3 और�4
Correct Answer: C
Your Answer:
Explanation

Solution (c)

Explanation:

नी�त�आयोग��भावी�शासन�के��न�न�ल�खत�सात��तंभ��पर�आधा�रत�है:

1. जन-क���त�एज�डा�जो�समाज�के�साथ-साथ����य��क��आकां�ा��को�पूरा�करता�है।�
2. नाग�रक��क��ज�रत��का�अनुमान�लगाने�और�उसका���तउ�र�दे ने�म��स��य।�
3. जनसहभा�गता�, नाग�रक��क��भागीदारी�से।�
4. सभी�पहलु��म��म�हला��को�सश��बनाना।�
5. एससी�, एसट��, ओबीसी�और�अ�पसं�यक��पर��वशेष��यान�दे ने�के�साथ�सभी�समूह��को�शा�मल�करना।�
6. युवा��के��लए�अवसर�क��समानता।�
7. सरकार�को���यमान�और�उ�रदायी�बनाने�के��लए��ौ�ो�गक��के�उपयोग�के�मा�यम�से�पारद�श�ता।�

QUESTION 88.
रा�ीय��म�अथ�शा���अनुसंधान�एवं��वकास�सं�थान�( NILERD) के�बारे�म���न�न�ल�खत�कथन��पर��वचार�कर�:

1. यह�नी�त�आयोग�का�एक�अधीन�थ�काया�लय�है।�
2. यह�मु�य��प�से�नी�त�आयोग�क��अनुदान�सहायता��ारा��व��पो�षत�है।�
3. इं�ट��ूट�ऑफ�ए�लाइड�मैनपावर��रसच��( IAMR), �जसे�1962 म��सोसायट��पंजीकरण�अ�ध�नयम�, 1860 के�तहत��था�पत
�कया�गया�था�, का�नाम�बदलकर�NILERD कर��दया�गया�है।�

IASbaba
Web: http://ilp.iasbaba.com/ Score:
Email: ilp@iasbaba.com 0.00 / 200
Page 174
AIPTS/ILP VETERANS-
Exam Title :
2020 TE...
Email : yadavanurag075@gmail.com
Contact : 8882839768

ऊपर��दए�गए�कथन��म��से�कौन�सा�सही�है�/ ह��?

a) केवल�1
b) केवल�1 और�2
c) केवल�2 और�3
d) 1, 2 और�3
Correct Answer: D
Your Answer:
Explanation

Solution (d)

Basic information:

रा�ीय��म�अथ�शा���अनुसंधान�एवं��वकास�सं�थान�( NILERD) ( पूव��म��इं�ट��ूट�ऑफ�ए�लाइड�मैनपावर��रसच�) नी�त�आयोग�का�एक


अधीन�थ�काया�लय�है।�

इं�ट��ूट�ऑफ�ए�लाइड�मैनपावर��रसच��( IAMR) क���थापना�1962 म��सोसाइट�ज�र�ज��े शन�ए�ट�1860 के�तहत�क��गई�थी�, जो�एक


ऐसे�सं�थान�के��प�म��काम�करेगा�जो��वचार��के�समाशोधन-गृह�के��प�म��काय��करेगा�तथा�प�र�े�य��नयोजन�और�एक�करण�को�बढ़ावा�दे ने�के
�लए�मानव�पूंजी��वकास�पर�नी�त�अनुसंधान�का�काय��करेगा।�सं�थान�का�मु�य�उ�े �य�अनुसंधान�, �श�ा�और���श�ण�, परामश��, आ�द�के
मा�यम�से�मानव�संसाधन��क���कृ�त�, �वशेषता��और�उपयोग�के�बारे�म���ान�को�आगे�बढ़ाना�है।�

IAMR को�9 जून�2014 को�रा�ीय��म�अथ�शा���अनुसंधान�एवं��वकास�सं�थान�( NILERD) के��प�म���फर�से�नाम��दया�गया�है।�NILE


RD को�मु�य��प�से�नी�त�आयोग�(पूव��योजना�आयोग) से�अनुदान��ा�त�करता�है�, तथा�अनुबं�धत�अनुसंधान�प�रयोजना��, और��श�ा
और���श�ण�ग�त�व�धय��से�अपने��वयं�के�सृ�जत�राज�व��ारा�काय��करता�है।�NILERD का�मु�य�उ�े �य�एक�सं�थागत�ढांचे�को��वक�सत
करने�और�एक��व��थत�मानव�संसाधन��नयोजन�अनुसंधान����या�को�संचा�लत�करने�म��स�म��वक�सत�करना�है।�

QUESTION 89.
रा�ीय�मानवा�धकार�आयोग�के�बारे�म���न�न�ल�खत�म��से�कौन�सा�कथन�स�य�है�/ नह��है�?

1. यह�एक�वैधा�नक��नकाय�है��जसे�2006 म��संसद��ारा�अ�ध�नय�मत�कानून�के�तहत��था�पत��कया�गया�था।�
2. यह�एक�ब�-सद�यीय��नकाय�है��जसम��एक�अ�य��और�चार�सद�य�होते�ह�।�
3. यह��व-सं�ान�( suo motu) के�आधार�पर�मानवा�धकार��के��कसी�भी�उ�लंघन�क��जांच�कर�सकता�है।�

उपयु���वक�प�चुन�:

a) केवल�1
b) केवल�1 और�2
c) केवल�2 और�3
d) 1, 2 और�3
Correct Answer: A
Your Answer:
Explanation

Solution (c)

कथन��व�ेषण:

IASbaba
Web: http://ilp.iasbaba.com/ Score:
Email: ilp@iasbaba.com 0.00 / 200
Page 175
AIPTS/ILP VETERANS-
Exam Title :
2020 TE...
Email : yadavanurag075@gmail.com
Contact : 8882839768

कथन�1 कथन�2 कथन�3

अस�य� स�य� स�य�

एनएचआरसी��कसी�लोक
सेवक��ारा�इस�तरह�के
आयोग�एक�ब�-सद�यीय��नकाय�है��जसम� उ�लंघन�क��रोकथाम�म�
एक�अ�य��और�चार�सद�य�होते�ह�।� मानवा�धकार��या�लापरवाही�के
�कसी�भी�उ�लंघन�के�बारे�म�
अ�य��को�भारत�का�एक�सेवा�नवृ��मु�य पूछताछ�कर�सकता�है�, या�तो
�यायाधीश�होना�चा�हए�, तथा�सद�य��को मुकदमा�कर�सकता�है�या�इसे
सव��च��यायालय�के�एक�सेवारत�या ��तुत�करने�के��लए�या
रा�ीय�मानवा�धकार�आयोग�एक�वैधा�नक
सेवा�नवृ���यायाधीश�, मानवा�धकार��के अदालत�के�आदे श�पर�जाँच
(और�संवैधा�नक�नह�) �नकाय�है।�यह�19
संबंध�म���ान�या��ावहा�रक�अनुभव�रखने कर�सकता�है।�
93 म��संसद��ारा�बनाए�गए�एक�कानून
वाले�दो����य��क��सेवा�या�उ�च��यायालय
के�तहत��था�पत��कया�गया�था�, अथा�त्�,
का�सेवा�नवृ��मु�य��यायाधीश�होना आयोग�दे श�म��मानव�अ�धकार�
मानवा�धकार�संर�ण�अ�ध�नयम�,
चा�हए।� का��हरी�है�, अथा�त्�, सं�वधान
1993 ।�यह�अ�ध�नयम�2006 म�
�ारा�गारंट�कृत�����क�
संशो�धत��कया�गया�था।� इन�पूण�का�लक�सद�य��के�अलावा�, आयोग जीवन�, �वतं�ता�, समानता
म��चार�पदे न�सद�य�भी�होते�ह��- रा�ीय और�ग�रमा�से�संबं�धत
अ�पसं�यक�आयोग�के�अ�य��, रा�ीय अ�धकार�या�अंतररा�ीय
अनुसू�चत�जा�त�आयोग�के�अ�य��, रा�ीय वाचा��म��स���हत�अ�धकार।�
अनुसू�चत�जनजा�त�आयोग�के�अ�य��और
रा�ीय�म�हला�आयोग�के�अ�य�।� और�जो�भारत�म���यायालय�
�ारा�लागू�करने�यो�य�अ�धकार
है।�

QUESTION 90.
�न�न�ल�खत�कथन��पर��वचार�कर�:

1. रा�ीय�मानवा�धकार�आयोग�के�अ�य��क���नयु���रा�प�त��ारा�क��जाती�है।�
2. रा�ीय�मानवा�धकार�आयोग�के�सद�य��को��धानमं�ी�क��अ�य�ता�और�लोकसभा�अ�य��, रा�यसभा�के�उपा�य��, संसद�के�दोन�
सदन��म���वप��के�नेता�और�क���य�गृह�मं�ी�वाली�छह�सद�यीय�स�म�त��ारा��नयु���कया�जाता�है।�

ऊपर��दए�गए�कथन��म��से�कौन�सा�सही�है�/ ह��?

a) केवल�1
b) केवल�2
c) दोन��1 और�2
d) न�तो�1 और�न�ही�2
Correct Answer: A
Your Answer:
Explanation

Solution (a)

IASbaba
Web: http://ilp.iasbaba.com/ Score:
Email: ilp@iasbaba.com 0.00 / 200
Page 176
AIPTS/ILP VETERANS-
Exam Title :
2020 TE...
Email : yadavanurag075@gmail.com
Contact : 8882839768

कथन��व�ेषण:

कथन�1 कथन�2

स�य� अस�य�

अ�य��और�सद�य��क���नयु���रा�प�त��ारा��धान�मं�ी�क�
अ�य�ता�वाली�छः�सद�यीय�स�म�त�क���सफा�रश��पर�क��जाती
है��जसम���धान�मं�ी�, लोकसभा�अ�य��, रा�यसभा�के�उपा�य�
, संसद�के�दोन��सदन��म���वप��के�नेता�और�के���य�गृह�मं�ी
NHRC के�अ�य��को�रा�प�त��ारा��नयु���कया�जाता�है।� शा�मल�होते�ह�।�

इसके�अलावा�, सु�ीम�कोट� �के�एक��स�ट�ग�जज�या��कसी�हाई


कोट� �के��स�ट�ग�चीफ�ज��टस�क���नयु���भारत�के�मु�य
�यायाधीश�के�परामश��के�बाद�ही�क��जा�सकती�है।�

QUESTION 91.
�न�न�ल�खत�म��से�कौन�पाँच�वष��क��अव�ध�के��लए�या�जब�तक�वे�65 वष��क��आयु��ा�त�नह��कर�लेते�ह��, पद�धारण�करते�ह��?

a) NHRC के�अ�य��और�सद�य�
b) SHRC के�अ�य��और�सद�य�
c) CAT के�अ�य��और�सद�य�
d) इनम��से�कोई�भी�नह��
Correct Answer: D
Your Answer:
Explanation

Solution (d)

Explanation:

रा�ीय�मानवा�धकार�आयोग�एक�ब�-सद�यीय��नकाय�है��जसम��एक�अ�य��और�चार�सद�य�होते�ह�।�अ�य��और�सद�य�पाँच�वष��क��अव�ध
तक�या�70 वष��क��आयु��ा�त�करने�तक�, जो�भी�पहले�हो�, के��लए�पद�धारण�करते�ह�।�

रा�य�मानवा�धकार�आयोग�एक�ब�-सद�यीय��नकाय�है��जसम��एक�अ�य��और�दो�सद�य�होते�ह�।�अ�य��और�सद�य�पांच�साल�के�काय�काल
के��लए�या�70 वष��क��आयु��ा�त�करने�तक�, जो�भी�पहले�हो�, पद�धारण�करते�ह�।�

CAT एक�ब�-सद�यीय��नकाय�है��जसम��एक�अ�य��और�सद�य�होते�ह�।�वे�पांच�साल�क��अव�ध�के��लए�या�जब�तक��क�वे�65 वष��क��आयु


�ा�त�नह��कर�लेते�, अ�य��के�मामले�म��तथा�सद�य��के�मामले�म��62 साल�तक�, जो�भी�पहले�हो।�

QUESTION 92.
सं�वधान�ने�सरकार�को��न�न�ल�खत�म��से��कस�को�सेवा�समा��त�के�बाद�आगे�क���नयु���के��लए�रोका�नह��है�?

1. सेवा�नवृ��चुनाव�आयु��
2. CAG

IASbaba
Web: http://ilp.iasbaba.com/ Score:
Email: ilp@iasbaba.com 0.00 / 200
Page 177
AIPTS/ILP VETERANS-
Exam Title :
2020 TE...
Email : yadavanurag075@gmail.com
Contact : 8882839768

3. NHRC और�SHRC के�सद�य�

सही�उ�र�चुन�:

a) केवल�1
b) 1 और�2
c) 1, 2 और�3
d) कोई�नह��
Correct Answer: A
Your Answer:
Explanation

Solution (a)

कथन��व�ेषण:

कथन�1 कथन�2 कथन�3

स�य� अस�य� अस�य�

कैग�आगे�अ�य�पद�पाने�के��लए�यो�य�नह� उनके�काय�काल�के�बाद�, NHRC या�SH


सं�वधान�ने�सेवा�नवृ��चुनाव�आयु���को
होता�है�, जो�भारत�सरकार�या��कसी RC के�अ�य��और�सद�य�क���या�रा�य
सरकार��ारा��कसी�भी�अ�य��नयु���से
रा�य�के�अधीन�हो�, जब�वह�अपना सरकार�के�तहत�आगे�के�रोजगार�के��लए
वं�चत�नह���कया�है।�
काया�लय�से�सेवा�नवृ��होता�है।� पा��नह��होते�ह�।�

QUESTION 93.
NHRC और�SHRC के�बारे�म���न�न�ल�खत�कथन��पर��वचार�कर�:

1. NHRC के�अ�य��या�सद�य�के�वेतन�, भ�े�और�अ�य�शत��क���सरकार��ारा��नधा��रत�क��जाती�ह�।�


2. SHRC के�अ�य��या�सद�य�के�वेतन�, भ�े�और�अ�य�शत��का��नधा�रण�रा�य�सरकार��ारा��कया�जाता�है।�

ऊपर��दए�गए�कथन��म��से�कौन�सा�सही�है�/ ह��?

a) केवल�1
b) केवल�2
c) दोन��1 और�2
d) न�तो�1 और�न�ही�2
Correct Answer: C
Your Answer:
Explanation

Solution (c)

कथन��व�ेषण:

IASbaba
Web: http://ilp.iasbaba.com/ Score:
Email: ilp@iasbaba.com 0.00 / 200
Page 178
AIPTS/ILP VETERANS-
Exam Title :
2020 TE...
Email : yadavanurag075@gmail.com
Contact : 8882839768

कथन�1 कथन�2

स�य� स�य�

एनएचआरसी�के�अ�य��या�सद�य�के�वेतन�, भ�े�और�अ�य रा�य�सरकार��ारा�एसएचआरसी�के�अ�य��या�सद�य�के�वेतन�,


शत��क���सरकार��ारा��नधा��रत�क��जाती�ह�।� भ�े�और�अ�य�शत��का��नधा�रण��कया�जाता�है।�

QUESTION 94.
रा�ीय�मानवा�धकार�आयोग�के�संदभ��म��, गलत�कथन�क��पहचान�कर�:

a) इसम��अपनी����या�को��व�नय�मत�करने�क��श����न�हत�है।�
b) इसम���स�वल��यायालय�क��सभी�श��याँ�होती�ह��तथा�इसक��काय�वाही�म��एक��या�यक�च�र��होता�है।�
c) यह�क���और�रा�य�सरकार��या��कसी�अ�य�अधीन�थ��ा�धकारी�से�सूचना�या��रपोट� �के��लए�मांग�कर�सकता�है।�
d) मानवा�धकार��के�उ�लंघन�क���शकायत��क��जांच�के��लए�जांच�कम�चा�रय��का�अपना�संघटक�( nucleus) नह��है।�
Correct Answer: D
Your Answer:
Explanation

Solution (d)

कथन��व�ेषण:

कथन�( a) कथन�( b) कथन�( c) कथन�( d)

स�य� स�य� स�य� अस�य�

IASbaba
Web: http://ilp.iasbaba.com/ Score:
Email: ilp@iasbaba.com 0.00 / 200
Page 179
AIPTS/ILP VETERANS-
Exam Title :
2020 TE...
Email : yadavanurag075@gmail.com
Contact : 8882839768

NHRC के�पास�मानव�अ�धकार�
के�उ�लंघन�क���शकायत��क��जांच
के��लए�जांच�कम�चा�रय��का
अपना�संघटक�( nucleus ) है।

इसके�अलावा�, क���सरकार�या
NHRC क���और�रा�य
NHRC के�पास�द�वानी�अदालत �कसी�रा�य�सरकार�क���कसी�भी
NHRC म��अपनी����या�को सरकार��या��कसी�अ�य
क��सभी�श��यां�ह��और�इसक� अ�धकारी�या�जांच�एज�सी�क�
�व�नय�मत�करने�क��श�� अधीन�थ��ा�धकारी�से�सूचना
काय�वाही�म��एक��या�यक�च�र� सेवा��का�उपयोग�करने�का
�न�हत�होती�है।� या��रपोट� �के��लए�मांग�कर
है।� अ�धकार�है।�
सकता�है।�
इसने�मानव�अ�धकार��के�उ�लंघन
के�बारे�म��पहली�सूचना�के�साथ
गैर-सरकारी�संगठन��के�साथ
�भावी�सहयोग�भी��था�पत��कया
है।�

QUESTION 95.
NHRC या�SHRC के�बारे�म���न�न�ल�खत�कथन��पर��वचार�कर�:

1. इसम��मानवा�धकार��के�उ�लंघन�करने�वाल��को�दं �डत�करने�क��, न�ही�पी�ड़त�को�मौ��क�राहत�स�हत�कोई�राहत�दे ने�के��लए�कोई


श���नह��है।�
2. आयोग�के�काय���कृ�त�म��मु�य��प�से��सफा�रशी�ह�।�

ऊपर��दए�गए�कथन��म��से�कौन�सा�गलत�है�/ ह��?

a) केवल�1
b) केवल�2
c) दोन��1 और�2
d) न�तो�1 और�न�ही�2
Correct Answer: D
Your Answer:
Explanation

Solution (d)

कथन��व�ेषण:

कथन�1 कथन�2

स�य� स�य�

IASbaba
Web: http://ilp.iasbaba.com/ Score:
Email: ilp@iasbaba.com 0.00 / 200
Page 180
AIPTS/ILP VETERANS-
Exam Title :
2020 TE...
Email : yadavanurag075@gmail.com
Contact : 8882839768

NHRC या�SHRC के�पास�मानवा�धकार��के


NHRC या�SHRC के�काय���कृ�त�म��मु�य��प�से
उ�लंघनकता���को�दं �डत�करने�क��कोई�श���नह��है�, न�ही
अनुशंसा�मक�ह�।�
पी�ड़त�को�मौ��क�राहत�स�हत�कोई�राहत�दे ने�क��श���है।�

QUESTION 96.
�न�न�ल�खत�म��से�कौन�रा�प�त�को�अपनी�वा�ष�क�या��वशेष��रपोट� ���तुत�नह��करता�है�?

a) कैग�( CAG)
b) भाषाई�अ�पसं�यक��के��लए�आयु��
c) रा�ीय�मानवा�धकार�आयोग�
d) कोई�नह��
Correct Answer: C
Your Answer:
Explanation

Solution (c)

Explanation:

रा�ीय�मानवा�धकार�आयोग�अपनी�वा�ष�क�या��वशेष��रपोट��को�क���सरकार�और�संबं�धत�रा�य�सरकार�को���तुत�करता�है।�ये��रपोट� �संबं�धत
�वधानसभा��के�सम��रखी�जाती�है�, साथ�ही�, आयोग�क���सफा�रश��और�ऐसी��कसी�भी��सफा�रश�को�न�मानने�के�कारण��पर�एक��ापन
भी�स�पा�जाता�है।�

CAG और�भाषाई�अ�पसं�यक��के��लए�आयु��अपनी��रपोट� �रा�प�त�को�स�पते�ह�।�

QUESTION 97.
लोकायु��और�उपलोकायु���कसके��ारा��नयु��होते�ह��-

a) मु�यमं�ी�
b) रा�यपाल�
c) उ�च��यायालय�के��यायाधीश�
d) उ�च��यायालय�के�मु�य��यायाधीश�
Correct Answer: B
Your Answer:
Explanation

Solution (b)

Explanation:

लोकायु��और�उपलोकायु��को�रा�य�के�रा�यपाल��ारा��नयु���कया�जाता�है।��नयु��करते�समय�, अ�धकांश�रा�य��म��रा�यपाल�(क) रा�य


उ�च��यायालय�के�मु�य��यायाधीश�, और�(ख) रा�य��वधान�सभा�म���वप��के�नेता�से�सलाह�लेते�ह�।�

QUESTION 98.
क���य�सूचना�आयोग�( CIC) के�बारे�म���न�न�ल�खत�कथन��पर��वचार�कर�:

1. वह�क���सरकार�, रा�य�सरकार�और�क���शा�सत��दे श��के�अधीन�काया�लय��, �व�ीय�सं�थान��, साव�ज�नक�उप�म��आ�द�से�संबं�धत


�शकायत��और�अपील��क��सुनवाई�कर�सकता�है।�

IASbaba
Web: http://ilp.iasbaba.com/ Score:
Email: ilp@iasbaba.com 0.00 / 200
Page 181
AIPTS/ILP VETERANS-
Exam Title :
2020 TE...
Email : yadavanurag075@gmail.com
Contact : 8882839768

2. वह�5 वष��क��अव�ध�तक�या�65 वष��क��आयु��ा�त�करने�तक�, जो�भी�पहले�हो�, पद�धारण�कर�सकता�है।�

ऊपर��दए�गए�कथन��म��से�कौन�सा�सही�है�/ ह��?

a) केवल�1
b) केवल�2
c) दोन��1 और�2
d) न�तो�1 और�न�ही�2
Correct Answer: D
Your Answer:
Explanation

Solution (d)

Basic information:

क���य�सूचना�आयोग�( CIC) का�गठन�सूचना�का�अ�धकार�अ�ध�नयम�( 2005) के��ावधान��के�तहत�एक�आ�धका�रक�राजप��अ�धसूचना


के�मा�यम�से��कया�गया�था।�

कथन��व�ेषण:

कथन�1 कथन�2

अस�य� अस�य�

CIC क���सरकार�और�क���शा�सत��दे श��के�अंतग�त�आने�वाले मु�य�सूचना�आयु��और�एक�सूचना�आयु��का�काय�काल�3


काया�लय��, �व�ीय�सं�थान��, साव�ज�नक��े��के�उप�म� वष��या�जब�तक�वे�65 वष��क��आयु��ा�त�नह��कर�लेते�, जो
आ�द�से�संबं�धत��शकायत��और�अपील��को�दे खता�है।� भी�पहले�हो�, होता�है।�

QUESTION 99.
UPSC के��े�ा�धकार�को��कसके��ारा�बढ़ाया�जा�सकता�है�-

a) गृह�मं�ालय�
b) का�म�क�मं�ालय�
c) रा�प�त�
d) संसद�
Correct Answer: D
Your Answer: Unanswered
Explanation

Solution (d)

Explanation:

UPSC के��े�ा�धकार�को�संसद��ारा�बनाए�गए�अ�ध�नयम��ारा�बढ़ाया�जा�सकता�है।�

QUESTION 100.

IASbaba
Web: http://ilp.iasbaba.com/ Score:
Email: ilp@iasbaba.com 0.00 / 200
Page 182
AIPTS/ILP VETERANS-
Exam Title :
2020 TE...
Email : yadavanurag075@gmail.com
Contact : 8882839768

मानवा�धकार��यायालय��( Human Rights Courts) के�संबंध�म���न�न�ल�खत�कथन��पर��वचार�कर�:

1. मानवा�धकार�अ�ध�नयम�क��सुर�ा�के��लए���येक��जले�म��मानवा�धकार��यायालय�क���थापना�क��गयी�है।�
2. मानवा�धकार��यायालय�रा�य�सरकार��ारा�केवल�उस�रा�य�के�उ�च��यायालय�के�मु�य��यायाधीश�क��सहम�त�से��था�पत�क��जा
सकते�ह�।�

ऊपर��दए�गए�कथन��म��से�कौन�सा�सही�है�/ ह��?

a) केवल�1
b) केवल�2
c) दोन��1 और�2
d) न�तो�1 और�न�ही�2
Correct Answer: C
Your Answer:
Explanation

Solution (c)

कथन��व�ेषण:

कथन�1 कथन�2

स�य� स�य�

मानवा�धकार�संर�ण�अ�ध�नयम�( 1993) म��मानवा�धकार��के ये��यायालय�रा�य�सरकार��ारा�केवल�उस�रा�य�के�उ�च


उ�लंघन�के��व�रत�मुकदमे�के��लए���येक��जले�म��मानवा�धकार �यायालय�के�मु�य��यायाधीश�क��सहम�त�से��था�पत�क��जा
�यायालय�क���थापना�का�भी��ावधान�है।� सकते�ह�।�

अ�त�र��जानकारी:

• ��येक�मानवा�धकार��यायालय�के��लए�, रा�य�सरकार�एक�सरकारी�वक�ल�को��न�द���करती�है�या�एक��वशेष�सरकारी�वक�ल�के��प
म��एक�वक�ल�(�जसने�सात�साल�से��ै��टस��कया�है) को��नयु��करती�है।�

IASbaba
Web: http://ilp.iasbaba.com/ Score:
Email: ilp@iasbaba.com 0.00 / 200
Page 183

You might also like